{"qid": 0, "query": "Would the One Ring even work for anyone but Sauron?", "score": 462, "views": 76690, "answer_pids": [130975, 130977, 130982, 130991, 130997, 130998, 131019, 131021, 131026, 135480, 139194, 143870, 145486, 182816], "question_author": ""} {"qid": 1, "query": "Could I fake death to stop being Force-choked?", "score": 452, "views": 35798, "answer_pids": [152301, 152331, 152343, 152442, 169727], "question_author": "EtienneT"} {"qid": 2, "query": "Who or what was Tom Bombadil?", "score": 434, "views": 99836, "answer_pids": [109883, 109890, 111423, 111459, 123072, 123190, 123866, 126082, 130382, 136722, 143233, 143357, 147092, 164706, 176593], "question_author": "Jaydles"} {"qid": 3, "query": "Why dont Muggle-born wizards use Muggle technology to fight Death Eaters?", "score": 378, "views": 150656, "answer_pids": [110334, 110335, 110339, 110340, 110342, 110607, 112222, 116566, 117010, 118889, 119859, 120072, 121325, 127544, 127555, 133235, 133554, 134605, 135862, 140159, 142895, 156516, 157720, 167062, 174912, 181003, 182596, 187012, 191960, 192588, 207111], "question_author": "Rowland Shaw"} {"qid": 4, "query": "Story where the number 3 is the monster?", "score": 352, "views": 21404, "answer_pids": [153672], "question_author": "Brian"} {"qid": 5, "query": "Why didnt Gandalf or Frodo Fly to Mount Doom?", "score": 333, "views": 130782, "answer_pids": [110194, 110197, 110199, 110204, 110219, 111192, 112001, 113065, 116405, 124755, 125721, 127353, 131043, 134664, 138370, 155507, 163753, 170348, 179085], "question_author": "Rowland Shaw"} {"qid": 6, "query": "How long was Bill Murrays character (Phil Connors) supposed to be in a time loop in the film Groundhog Day?", "score": 309, "views": 60660, "answer_pids": [112245, 112248, 114565, 159985], "question_author": "jessegavin"} {"qid": 7, "query": "In what order should the Star Wars movies be watched?", "score": 294, "views": 1431470, "answer_pids": [109846, 109848, 109849, 109882, 110075, 110504, 111812, 112282, 115381, 118513, 128580, 137577, 156965, 157271, 171787], "question_author": "arathorn"} {"qid": 8, "query": "Why does everyone think that the Millennium Falcon is a piece of junk?", "score": 257, "views": 50918, "answer_pids": [157225, 157226, 157227, 157239, 157240, 157248, 157349, 157360, 157461, 157814, 189538], "question_author": "Rowland Shaw"} {"qid": 9, "query": "Which 2015 technologies were correctly predicted by Back to the Future II?", "score": 249, "views": 26961, "answer_pids": [150094, 150095, 150118, 153525], "question_author": "Michael McCarty"} {"qid": 10, "query": "Are all stormtroopers as poor soldiers as the ones in the movies?", "score": 240, "views": 28127, "answer_pids": [111038, 111039, 111040, 111041, 111042, 111044, 111071, 115047, 141919, 160916, 170235, 176085], "question_author": "lilserf"} {"qid": 11, "query": "How did Dumbledore, or anyone, know Lily had sacrificed herself for Harry?", "score": 217, "views": 38011, "answer_pids": [135969, 135970, 135972, 135980, 136016, 140038, 153205, 153206, 153267, 167024, 192823, 195985], "question_author": "Jaydles"} {"qid": 12, "query": "Why did C.S. Lewis use Turkish Delight as temptation for Edmund?", "score": 211, "views": 35504, "answer_pids": [187420, 187429, 187515, 193531, 198915], "question_author": "Rowland Shaw"} {"qid": 13, "query": "Whose underwear is this?", "score": 208, "views": 28712, "answer_pids": [139068, 139069], "question_author": "Grant Palin"} {"qid": 14, "query": "Did Tolkien really explicitly consider Sam the true hero of The Lord of the Rings?", "score": 205, "views": 234824, "answer_pids": [112183, 144181, 147521, 207423], "question_author": ""} {"qid": 15, "query": "Are E.T. and Star Wars in the same universe?", "score": 201, "views": 67713, "answer_pids": [112362, 115517, 115983], "question_author": "jessegavin"} {"qid": 16, "query": "Hogwarts: So why arent the kids doing it?", "score": 200, "views": 48851, "answer_pids": [136653, 136654, 136655, 136662, 136680, 136684, 136715, 173516, 185728], "question_author": "Michael McCarty"} {"qid": 17, "query": "Was Yoda fully sane on Dagobah?", "score": 197, "views": 72713, "answer_pids": [112249, 112250, 112251, 163590, 170874], "question_author": "pupeno"} {"qid": 18, "query": "Why does the Death Star have a trash compactor?", "score": 195, "views": 34779, "answer_pids": [186001, 186002, 186009, 186011, 186068, 186072, 186095], "question_author": "JeremyFromEarth"} {"qid": 19, "query": "Was the Millennium Falcon too slow?", "score": 193, "views": 34620, "answer_pids": [109824, 109825, 109826, 127580, 128162, 137450, 150060, 167492], "question_author": "Bernhard Hofmann"} {"qid": 20, "query": "Why did Douglas Adams pick 42 as the ultimate answer?", "score": 188, "views": 44053, "answer_pids": [109475, 123983, 208093], "question_author": "Kendall Hopkins"} {"qid": 21, "query": "Is the basic premise of humans as a power source in The Matrix reasonable?", "score": 179, "views": 63735, "answer_pids": [109737, 109739, 109740, 109741, 109744, 109754, 109770, 109898, 110957, 115711, 115812, 120890, 123512, 130414, 132765, 132796, 137382, 144317, 163118, 167755], "question_author": "Mark Rogers"} {"qid": 22, "query": "Does Batman use Linux?", "score": 177, "views": 33519, "answer_pids": [134280, 134285, 134299], "question_author": ""} {"qid": 23, "query": "What is the origin of the Riker Chair Maneuver?", "score": 173, "views": 102588, "answer_pids": [126051, 195410], "question_author": "Michael McCarty"} {"qid": 24, "query": "What does Gandalf mean by Secret Fire, Flame of Anor and Flame of Ud\u00fbn?", "score": 171, "views": 175726, "answer_pids": [122873, 123189, 128382, 152999, 153104], "question_author": "jessegavin"} {"qid": 25, "query": "What order should Asimovs Foundation series be read in?", "score": 170, "views": 504924, "answer_pids": [110195, 110196, 110200, 110202, 110203, 110215, 110471, 110481, 119177, 120237, 121787, 129283, 157899, 164614, 184831], "question_author": "Pulkit Sinha"} {"qid": 26, "query": "What did Yoda do on Dagobah?", "score": 169, "views": 13862, "answer_pids": [153944, 153963], "question_author": ""} {"qid": 27, "query": "How can Scrooge McDuck dive into money without hurting himself?", "score": 168, "views": 29614, "answer_pids": [182621, 182648], "question_author": "Michael McCarty"} {"qid": 28, "query": "Was Aragorns claim to the throne legitimate?", "score": 168, "views": 27411, "answer_pids": [158409, 158410, 158411, 158440, 158943, 187051], "question_author": "MatthewMartin"} {"qid": 29, "query": "Why didnt the basilisk bite in Chamber of Secrets destroy the Horcrux?", "score": 167, "views": 92680, "answer_pids": [113563, 113564, 114173, 143691, 146383, 171501, 187228], "question_author": "Quickhorn"} {"qid": 30, "query": "If Luke Skywalker didn\u2019t want to be found, why did he leave a map?", "score": 165, "views": 45682, "answer_pids": [157449, 157454, 157456, 157489], "question_author": "jedihawk"} {"qid": 31, "query": "Which ship can go faster, the Millennium Falcon or the USS Enterprise-D?", "score": 164, "views": 35725, "answer_pids": [174654, 174692, 174700, 174713], "question_author": "Reid"} {"qid": 32, "query": "Where did Harrys parents get all of their money from?", "score": 163, "views": 131213, "answer_pids": [110664, 112226, 112239, 120067, 120069, 127397, 132711, 152613, 152614, 175343], "question_author": ""} {"qid": 33, "query": "Why did the Matrix simulate 1999 instead of a pre-computer year?", "score": 163, "views": 35616, "answer_pids": [137342, 137343, 137344, 137346, 137347, 137349, 137359, 137369, 137381, 137387, 137392, 137553, 156502], "question_author": "pupeno"} {"qid": 34, "query": "Why are ships in Star Wars so ridiculously easy to steal?", "score": 163, "views": 33302, "answer_pids": [161883, 161896, 161905, 161910, 161911, 162000], "question_author": ""} {"qid": 35, "query": "Why didnt Sauron become invisible while wearing the One Ring?", "score": 162, "views": 94007, "answer_pids": [115816, 115817, 115818, 115821, 115844, 122276], "question_author": "wdypdx22"} {"qid": 36, "query": "In-universe, is Aslan actually Jesus?", "score": 160, "views": 28655, "answer_pids": [159507, 159509, 159674, 191684], "question_author": ""} {"qid": 37, "query": "Why are there so many 47s?", "score": 159, "views": 22281, "answer_pids": [150666], "question_author": "Craig Walker"} {"qid": 38, "query": "Which finger was BB-8 sticking up?", "score": 158, "views": 20085, "answer_pids": [158341, 158342, 158452], "question_author": "Goran Jovic"} {"qid": 39, "query": "Why did Darth Vader die in Return of the Jedi?", "score": 155, "views": 352105, "answer_pids": [110528, 110530, 110534, 110535, 111146, 124301, 127062, 129408, 129576, 132028, 134775, 138554, 138607, 144365], "question_author": "John Fiala"} {"qid": 40, "query": "Does Hobbes ever do anything that Calvin himself could not do?", "score": 155, "views": 18871, "answer_pids": [175206, 175214, 175217, 175278, 187139], "question_author": ""} {"qid": 41, "query": "If only the Sith deal in absolutes, why does Obi-Wan say it that way?", "score": 152, "views": 219309, "answer_pids": [113229, 117447, 120896, 121951, 126893, 126895, 127390], "question_author": ""} {"qid": 42, "query": "Is there an official Marvel Cinematic Universe viewing order?", "score": 149, "views": 460932, "answer_pids": [111364, 111365, 129799, 137829], "question_author": ""} {"qid": 43, "query": "In Civil War, why did Tony Stark and Steve Rogers choose sides that didnt reflect their attitudes?", "score": 149, "views": 32531, "answer_pids": [189312, 189324, 189325, 189326, 189327, 189336, 189360, 189417], "question_author": "Dr G"} {"qid": 44, "query": "Was Darth Vader REALLY a Sith?", "score": 148, "views": 35585, "answer_pids": [112998, 113010, 113013, 113064, 123229, 131259, 151107, 151121, 167323, 171541], "question_author": ""} {"qid": 45, "query": "How does Picard choose a warp speed?", "score": 148, "views": 26226, "answer_pids": [112176, 112208, 132269], "question_author": "C. Ross"} {"qid": 46, "query": "Given a magical world, why is the Quibbler ridiculous?", "score": 148, "views": 19025, "answer_pids": [175762, 175767, 175770, 175775, 175777, 175781, 175801], "question_author": "gkrogers"} {"qid": 47, "query": "In The Matrix Revolutions How Does Neo Stop the Machine Weapons in the Real World?", "score": 144, "views": 51533, "answer_pids": [115089, 115090, 115227, 118037, 118117, 121298, 128733, 148943, 148972, 153391], "question_author": "ICodeForCoffee"} {"qid": 48, "query": "What did Padm\u00e9 die of?", "score": 143, "views": 112881, "answer_pids": [142186, 142187, 142196, 142199, 142206, 142209, 142291, 142356], "question_author": "Mark Rogers"} {"qid": 49, "query": "What is the significance of Sarumans Ring?", "score": 141, "views": 28046, "answer_pids": [110209, 110211, 110213, 110226, 110801, 120302, 191539, 197379, 199499], "question_author": "Dan McClain"} {"qid": 50, "query": "Why would Snape set his office password to Dumbledore?", "score": 140, "views": 31781, "answer_pids": [170850, 170854, 170855, 170930, 178752], "question_author": "user19"} {"qid": 51, "query": "Why is Shelob considered evil?", "score": 140, "views": 30796, "answer_pids": [195088, 195089, 195093, 195108, 195114, 195120, 195126, 195127, 195132, 195158, 195184], "question_author": "Rowland Shaw"} {"qid": 52, "query": "When and why did the Enterprise get balls?", "score": 140, "views": 18104, "answer_pids": [149293, 149294, 162224], "question_author": ""} {"qid": 53, "query": "Who created the Secret Stairs as a way into Mordor and for what purpose?", "score": 139, "views": 26467, "answer_pids": [170308, 170309, 170353, 170379], "question_author": ""} {"qid": 54, "query": "What do people in Westeros eat during Winter?", "score": 137, "views": 18938, "answer_pids": [165294, 165296, 165307, 173197], "question_author": null} {"qid": 55, "query": "Why was Hermione not in Ravenclaw?", "score": 135, "views": 119891, "answer_pids": [134197, 134199, 134231, 136750, 136754], "question_author": "MatthewMartin"} {"qid": 56, "query": "Why does the Party allow playing chess in Orwells 1984?", "score": 135, "views": 32303, "answer_pids": [189193, 189194, 189200, 189201, 189202, 189229], "question_author": "Toon Krijthe"} {"qid": 57, "query": "How much older was Hermione after her third year at Hogwarts?", "score": 135, "views": 23268, "answer_pids": [112685, 112686, 112693], "question_author": "David G"} {"qid": 58, "query": "How many times was Obi-Wan less than honest with Luke?", "score": 135, "views": 17248, "answer_pids": [156998, 157027, 157050, 160572], "question_author": "user19"} {"qid": 59, "query": "If diary Tom Riddle had succeeded, would there have been two Voldemorts?", "score": 134, "views": 57588, "answer_pids": [112787, 112788, 112789, 112804, 123660, 128416, 128601], "question_author": "Pr\u00e4riewolf"} {"qid": 60, "query": "Why doesnt the Discworld run out of water?", "score": 134, "views": 19955, "answer_pids": [166373, 166381, 166405, 166476], "question_author": ""} {"qid": 61, "query": "Why is The Matrix tinted green?", "score": 133, "views": 48872, "answer_pids": [164157, 164158], "question_author": "txwikinger"} {"qid": 62, "query": "Was the Cantina music deliberately off-tune or just an artifact of cheap production?", "score": 133, "views": 9329, "answer_pids": [128832], "question_author": "Grant Palin"} {"qid": 63, "query": "How did no other engineer see the design flaw of the Death Star?", "score": 132, "views": 17723, "answer_pids": [175949, 175966, 175991], "question_author": "Mnementh"} {"qid": 64, "query": "Why does everyone face the same way on the transporter?", "score": 132, "views": 15499, "answer_pids": [138389, 138392, 138394, 138406, 138417, 138434, 138435, 138544], "question_author": ""} {"qid": 65, "query": "Did Calvin ever realise that Hobbes was not real?", "score": 131, "views": 32559, "answer_pids": [175153, 175168, 175173, 175203, 175279], "question_author": "Craig Walker"} {"qid": 66, "query": "Is Slytherin Evil?", "score": 129, "views": 56569, "answer_pids": [112857, 112858, 112863, 112877, 117985, 124183, 144316, 189934], "question_author": ""} {"qid": 67, "query": "What is Marios full name?", "score": 128, "views": 45024, "answer_pids": [164741, 164742, 164748], "question_author": "lavonardo"} {"qid": 68, "query": "In Star Trek - Whats the Point of the Transporter Room?", "score": 128, "views": 24835, "answer_pids": [111811, 113491, 124414, 167012], "question_author": ""} {"qid": 69, "query": "In Back to the Future, why was the speed 88 miles per hour?", "score": 127, "views": 161467, "answer_pids": [116936, 116949, 132347, 132390, 140706, 149369], "question_author": ""} {"qid": 70, "query": "Did Palpatine intentionally lose to Mace Windu?", "score": 127, "views": 108648, "answer_pids": [114871, 114879, 115652, 123941, 124402, 126580, 132104, 133563, 135829, 153186, 154649], "question_author": "Dan McClain"} {"qid": 71, "query": "Why did the Jedi want to bring balance to the Force?", "score": 127, "views": 35664, "answer_pids": [111172, 111173, 111174, 111175, 112825, 116286, 124949, 129302, 139235], "question_author": "user19"} {"qid": 72, "query": "Why and how did the Star Trek Universe evolve to a cashless/commerce-less society?", "score": 127, "views": 16688, "answer_pids": [109693, 109694, 109696, 109699, 109701, 110059, 110066, 111684, 112000, 124900, 134662, 144091], "question_author": ""} {"qid": 73, "query": "What is the end of 2001: A Space Odyssey about?", "score": 126, "views": 115962, "answer_pids": [110314, 110315, 110316, 110319, 110321, 110326, 110330, 116978, 117030, 125632, 133307, 148677], "question_author": ""} {"qid": 74, "query": "What are Guinans powers from Star Trek: TNG?", "score": 126, "views": 53316, "answer_pids": [112095, 117193, 122510, 127703, 131260, 149187, 200535], "question_author": "Pawka"} {"qid": 75, "query": "Why would the Force become so mysterious and unrecognized after only 20 years?", "score": 126, "views": 25171, "answer_pids": [119721, 119722, 119727, 119730, 119733, 119738, 119862, 119920, 126672, 129559, 129560, 166332], "question_author": "Codemwnci"} {"qid": 76, "query": "Does any Looney Tunes character ever use any fictional brand-name product that isnt ACME?", "score": 126, "views": 12944, "answer_pids": [194888], "question_author": ""} {"qid": 77, "query": "Is Lex Luthors 40-cake theft canonical?", "score": 126, "views": 10635, "answer_pids": [193939], "question_author": "ICodeForCoffee"} {"qid": 78, "query": "Why is Sauron called the Necromancer?", "score": 125, "views": 50541, "answer_pids": [130778, 130782, 130826, 135726, 139172], "question_author": "moodygrrl"} {"qid": 79, "query": "What was the origin of Star Treks Borg?", "score": 124, "views": 55382, "answer_pids": [109519, 109877, 110028, 110265, 124469, 127923, 148993, 200434], "question_author": "triptych"} {"qid": 80, "query": "Why would killing Darth Vader or the Emperor turn Luke to the Dark Side?", "score": 124, "views": 27097, "answer_pids": [134388, 134389, 134390, 134405, 134407, 134454, 134609, 134624, 134785, 136582], "question_author": ""} {"qid": 81, "query": "When and why did orcs go green?", "score": 124, "views": 25711, "answer_pids": [180993, 180999, 181083, 181088], "question_author": "Toon Krijthe"} {"qid": 82, "query": "Why were the Elves so great at archery?", "score": 124, "views": 25505, "answer_pids": [114501, 114502, 114505, 114509, 114511, 114518, 114531, 114538, 115340, 125569, 140261], "question_author": ""} {"qid": 83, "query": "Why is JK Rowling considered bad at math?", "score": 123, "views": 42593, "answer_pids": [169361], "question_author": ""} {"qid": 84, "query": "Why doesnt Hogwarts have more teachers?", "score": 123, "views": 30469, "answer_pids": [168141, 168145, 168177, 168182, 168189, 168207, 191748], "question_author": null} {"qid": 85, "query": "What did Frodo mean by He told me the true story?", "score": 123, "views": 20950, "answer_pids": [174898, 174921], "question_author": ""} {"qid": 86, "query": "Whats the recommended reading order of the Ender Wiggin books?", "score": 122, "views": 188951, "answer_pids": [109438, 109442, 109625, 109778, 111200, 125991, 129282, 154807, 178729], "question_author": "Maniero"} {"qid": 87, "query": "Why is Tony Stark in the Iron Man suit unaffected by G-forces, inertia and blunt force trauma?", "score": 122, "views": 55518, "answer_pids": [117788, 117790, 117792, 120405, 123010, 138522], "question_author": null} {"qid": 88, "query": "Was any character alive during all of the Star Wars episodes from I to VII?", "score": 122, "views": 39338, "answer_pids": [160392, 160393, 160398, 160409, 160428, 160516, 160706, 167134, 187281, 202024, 208296], "question_author": "Wazery"} {"qid": 89, "query": "When & how did Yoda come to know about the Rule of Two?", "score": 122, "views": 23219, "answer_pids": [116413, 116416, 125510], "question_author": ""} {"qid": 90, "query": "Why would Dumbledore encourage people to call Voldemort Voldemort instead of Tom Riddle?", "score": 121, "views": 26553, "answer_pids": [126159, 126160, 126165, 126168, 126169, 126170, 126172, 142184, 188524], "question_author": "LeguRi"} {"qid": 91, "query": "Where does the archetypal image of the Grey alien come from?", "score": 121, "views": 21231, "answer_pids": [140682, 149241, 175481], "question_author": null} {"qid": 92, "query": "Can wizards create their own spells?", "score": 120, "views": 26536, "answer_pids": [111544, 111547, 111548, 111549, 111552, 111556, 126865, 135927, 138307, 141582, 146385, 147868], "question_author": "GWLlosa"} {"qid": 93, "query": "Was Harry making a big mistake with his plan for Elder Wand?", "score": 120, "views": 19632, "answer_pids": [115744, 115746, 115755, 140908, 141157, 141164, 152448, 161387, 161389, 168552], "question_author": "txwikinger"} {"qid": 94, "query": "Do the Imperials know they are evil?", "score": 119, "views": 29481, "answer_pids": [155314, 155317, 155324, 155325, 155331, 155357, 155377, 155460, 155508], "question_author": "cmcculloh"} {"qid": 95, "query": "Why did George Lucas set Star Wars in the past instead of the future?", "score": 119, "views": 27019, "answer_pids": [201788, 201789, 201826, 201829], "question_author": ""} {"qid": 96, "query": "Is Thor the only Avenger who cant die?", "score": 118, "views": 111399, "answer_pids": [116628, 116629, 116631, 116650, 116652, 119185, 125976, 128311, 128588, 137452], "question_author": "Shaihi"} {"qid": 97, "query": "Does Katniss depict proper handling of a bow?", "score": 118, "views": 17978, "answer_pids": [155047, 155048, 155064, 192264], "question_author": ""} {"qid": 98, "query": "Is the Dark Side of the Force more powerful?", "score": 117, "views": 98753, "answer_pids": [142766, 142767, 142769, 142775, 142778, 142780, 142786, 142793, 142833, 142835, 142843, 142903, 148313, 155163], "question_author": null} {"qid": 99, "query": "Story where scientist communicates with cities directly", "score": 117, "views": 4774, "answer_pids": [136721, 153311], "question_author": ""} {"qid": 100, "query": "Who is Anakin Skywalkers father?", "score": 116, "views": 458374, "answer_pids": [109900, 110000, 114142, 115783, 121011, 169484], "question_author": ""} {"qid": 101, "query": "Why does Leias accent change during A New Hope?", "score": 116, "views": 50114, "answer_pids": [110502, 110503, 110506, 110509, 110510, 152833, 164851], "question_author": "Pr\u00e4riewolf"} {"qid": 102, "query": "What was the first Sci-Fi work to feature a spaceship?", "score": 116, "views": 15314, "answer_pids": [148937, 148940, 148945, 148953, 148956, 149045, 156349, 164785, 176422, 206058], "question_author": ""} {"qid": 103, "query": "Why was Gandalf involved with the Dwarves quest to rob Smaug?", "score": 115, "views": 40881, "answer_pids": [110915, 110916, 110921, 110924, 181172], "question_author": "Dan McClain"} {"qid": 104, "query": "Is there a Vulcan funeral blessing?", "score": 115, "views": 13793, "answer_pids": [144331, 144332, 144336, 144337, 144349, 144371, 144438, 147769, 162483], "question_author": ""} {"qid": 105, "query": "Was Rorschachs Pagliacci joke a real joke?", "score": 114, "views": 208772, "answer_pids": [164618, 164638, 164731, 182974, 207394], "question_author": ""} {"qid": 106, "query": "What plot points are missing in the Harry Potter movies that are in the novels?", "score": 114, "views": 109644, "answer_pids": [112533, 112537, 113307, 126664, 127046, 127467, 127600, 127623, 128232, 128527, 128933, 130586, 131381, 136937, 138528, 139559], "question_author": "David Lambert"} {"qid": 107, "query": "Why Cant R2-D2 Talk?", "score": 114, "views": 19128, "answer_pids": [113308, 113312, 113317, 113319, 113328, 129122, 131230, 132425, 134338, 158393], "question_author": ""} {"qid": 108, "query": "Why isnt the system identifiable using only BB-8s map?", "score": 114, "views": 11277, "answer_pids": [157051, 157055, 157061, 157145, 157164, 157221, 157229], "question_author": ""} {"qid": 109, "query": "Which wars are Star Wars?", "score": 113, "views": 11790, "answer_pids": [151998, 151999], "question_author": null} {"qid": 110, "query": "Was ST:VOY watered down?", "score": 113, "views": 9204, "answer_pids": [154424, 154431], "question_author": "Mathias"} {"qid": 111, "query": "Is Luke Skywalker actually a Jedi?", "score": 112, "views": 27813, "answer_pids": [181445, 181446, 181472], "question_author": "ahockley"} {"qid": 112, "query": "Why arent Muggles extinct?", "score": 112, "views": 24578, "answer_pids": [168918, 168921, 168924, 168925, 168931, 168934, 168942, 168944, 168977, 168993, 169003, 169009, 169014], "question_author": "reuscam"} {"qid": 113, "query": "How many axes did Gimli bring on the quest of the Fellowship?", "score": 112, "views": 18083, "answer_pids": [147012, 147013, 147046, 147098], "question_author": "Wazery"} {"qid": 114, "query": "Are Jedi required to abstain from all sexual relations, or only from the ones involving emotional attachments?", "score": 111, "views": 43620, "answer_pids": [142817, 142818, 166366], "question_author": "Codemwnci"} {"qid": 115, "query": "Whats the story behind the Spock helmet?", "score": 111, "views": 24097, "answer_pids": [159170, 159171], "question_author": ""} {"qid": 116, "query": "How could a raw, untrained Force user be so adept at using the Force?", "score": 110, "views": 31227, "answer_pids": [156278, 156279, 156281, 156284, 156286, 156296, 156318, 167820], "question_author": ""} {"qid": 117, "query": "Why was Han Solo on Tatooine?", "score": 110, "views": 17548, "answer_pids": [137963, 138009, 163566, 163567, 163569, 201401], "question_author": ""} {"qid": 118, "query": "Could the Enterprise beam a vampire into a house she didn\u2019t have permission to enter?", "score": 110, "views": 11033, "answer_pids": [112462, 112463, 112465, 112466, 112468, 112474, 112476, 112479, 112482, 112483, 112496, 118141, 118160, 118161, 120671, 122413, 123486, 124608], "question_author": "David G"} {"qid": 119, "query": "When and how does Vader learn that Luke is his son?", "score": 109, "views": 156514, "answer_pids": [143378, 143789, 154071, 157098], "question_author": ""} {"qid": 120, "query": "Were the Smurfs the first to smurf their smurfs?", "score": 108, "views": 14312, "answer_pids": [169826, 169829, 169832, 169882, 169941, 170049, 180569], "question_author": ""} {"qid": 121, "query": "Why Do Jedi Use One Lightsaber in Combat?", "score": 107, "views": 42834, "answer_pids": [115638, 115639, 115642, 115643, 115647, 115648, 115663, 116376, 124146, 125458, 140218], "question_author": "Mark Rogers"} {"qid": 122, "query": "Which seven kings does Saruman think Gandalf wants to control?", "score": 107, "views": 25615, "answer_pids": [145790, 145794, 145800, 145812, 145827, 149171], "question_author": "lavonardo"} {"qid": 123, "query": "Did Dumbledore consider the implications of leaving a baby on a doorstep overnight?", "score": 107, "views": 15778, "answer_pids": [189070, 189086], "question_author": ""} {"qid": 124, "query": "Why didnt Yoda and Obi-Wan kill Darth Vader and the Emperor? Why wait for Skywalker?", "score": 106, "views": 71496, "answer_pids": [110250, 129605], "question_author": "Suspi"} {"qid": 125, "query": "What is a year in Westeros?", "score": 106, "views": 40682, "answer_pids": [111437, 111438, 127404, 131816, 135771, 140823], "question_author": "Noel M"} {"qid": 126, "query": "How is canonicity of derivative works determined for Star Wars?", "score": 106, "views": 10647, "answer_pids": [109621, 143202, 163591], "question_author": ""} {"qid": 127, "query": "Why was Gandalf afraid of the Balrog of Morgoth?", "score": 105, "views": 109776, "answer_pids": [115672, 115673, 115676, 115679, 125163, 139771], "question_author": ""} {"qid": 128, "query": "What is the reference Captain America understood?", "score": 105, "views": 79294, "answer_pids": [176503, 176504, 176579], "question_author": "Massimo"} {"qid": 129, "query": "Tolkien calls Sauron Morgoths Greatest Servant - But does he ever really earn that with his actions?", "score": 105, "views": 18534, "answer_pids": [144524, 144528, 144559, 144795, 146083, 148361, 211527], "question_author": "Rowland Shaw"} {"qid": 130, "query": "How do Star Trek viewscreens show perceivable depth?", "score": 105, "views": 15449, "answer_pids": [191344, 191358, 191370, 191372, 191412, 191447, 198178], "question_author": "Dan McClain"} {"qid": 131, "query": "How well-known is the theology of Middle-earth, in Middle-earth?", "score": 105, "views": 11539, "answer_pids": [131012, 131013, 131015, 131016, 131036, 131051, 144888, 178739, 199626], "question_author": "Maniero"} {"qid": 132, "query": "Is it true that The Doctor can only regenerate 12 times?", "score": 104, "views": 52216, "answer_pids": [109591, 109595, 109613, 109616, 109658, 109669, 111896, 112625, 113136, 123376, 123385, 124347, 124846, 130333, 142460, 158196, 206506], "question_author": ""} {"qid": 133, "query": "Did Eru Il\u00favatar trip Gollum?", "score": 104, "views": 42712, "answer_pids": [138986, 139007, 139009, 146163, 206405], "question_author": ""} {"qid": 134, "query": "Why do castle gates in GoT open inwards?", "score": 104, "views": 35747, "answer_pids": [178495, 178499, 178523, 178583], "question_author": "Patricia"} {"qid": 135, "query": "Why not let a dwarf carry the One Ring to Mount Doom, since their will cannot be dominated by the rings of power?", "score": 104, "views": 31913, "answer_pids": [139195, 139202, 139220, 139237, 139248, 139767], "question_author": "Dan McClain"} {"qid": 136, "query": "Did Darth Vader wear the same suit for 20+ years?", "score": 104, "views": 14082, "answer_pids": [197930, 197938, 197945], "question_author": "Mike Bohlmann"} {"qid": 137, "query": "Does no one run in Star Trek?", "score": 104, "views": 6459, "answer_pids": [123100, 179722, 180061], "question_author": "Dan McClain"} {"qid": 138, "query": "Is it possible to ruin one\u2019s career by failing the Kobayashi Maru?", "score": 103, "views": 32490, "answer_pids": [189487, 189488, 189496, 189499, 189502, 189527, 189546], "question_author": ""} {"qid": 139, "query": "Why was Darth Vader so careless?", "score": 103, "views": 26921, "answer_pids": [159184, 159185, 159186, 159192, 159195, 159211, 159230, 159265], "question_author": "Mark"} {"qid": 140, "query": "How is Shelobs name known?", "score": 103, "views": 17506, "answer_pids": [171160, 171162, 171166, 171569], "question_author": "Dan McClain"} {"qid": 141, "query": "Why does everyone call Hagrid by his last name?", "score": 103, "views": 17230, "answer_pids": [164411, 164412, 164523, 164537, 169791], "question_author": "Louis Gerbarg"} {"qid": 142, "query": "How can Data lose a chess game?", "score": 103, "views": 16315, "answer_pids": [133275, 133276, 133277, 133278, 133300, 133326, 133330, 133358, 133376, 133446, 133492, 181296], "question_author": "LessPop_MoreFizz"} {"qid": 143, "query": "Is/was General Leia Organa expected to be in the next episodes (VIII and IX) of Star Wars?", "score": 103, "views": 10079, "answer_pids": [171826, 171828, 171829, 175517, 190071], "question_author": ""} {"qid": 144, "query": "Why Does Samwise Gamgee Call Frodo Baggins Mr. Frodo?", "score": 102, "views": 71915, "answer_pids": [114877, 114880, 114884, 114918, 115927, 128691], "question_author": "mmccoo"} {"qid": 145, "query": "Why does Kylo Ren wear a mask?", "score": 102, "views": 59539, "answer_pids": [156342, 156344, 156345, 156371, 156631, 156737, 156862, 156871, 158358], "question_author": ""} {"qid": 146, "query": "Who is / was the Lord of the Rings?", "score": 102, "views": 56860, "answer_pids": [136364, 136367, 147003], "question_author": "Jack C Buel"} {"qid": 147, "query": "Why is invisibility the power granted by the One Ring?", "score": 102, "views": 30486, "answer_pids": [143103, 143126, 143128, 143129, 143137, 143207], "question_author": ""} {"qid": 148, "query": "Why do the people in MarvelVerse dislike X-Men more than other superheroes?", "score": 102, "views": 21492, "answer_pids": [138725, 138732, 138738, 138769, 149079], "question_author": ""} {"qid": 149, "query": "Is there an actual army in 1984?", "score": 102, "views": 20845, "answer_pids": [170099, 170106, 170107, 170120], "question_author": "Michiel de Mare"} {"qid": 150, "query": "Why do we hear Leias Theme during Kenobis death?", "score": 102, "views": 8468, "answer_pids": [110454, 139836], "question_author": "Chris Missal"} {"qid": 151, "query": "Did Peter Jackson ever explain why he left out the Scouring of the Shire?", "score": 101, "views": 50055, "answer_pids": [115247, 117797, 162682, 169631, 172653], "question_author": "lilserf"} {"qid": 152, "query": "Why is it that after 8,000 years, technology in Westeros has not evolved in the world of the Game of Thrones?", "score": 101, "views": 24222, "answer_pids": [119953, 119954, 119968, 119973, 125362, 125392, 125719, 126989, 126995, 133551, 140704, 140845], "question_author": "mafu"} {"qid": 153, "query": "Why does Zaphod Beeblebrox call Ford Prefect Ford when they meet on the Heart of Gold?", "score": 101, "views": 11709, "answer_pids": [164853], "question_author": ""} {"qid": 154, "query": "Is there a term for the science fiction trope where a character lists two historical things and a future thing?", "score": 100, "views": 7242, "answer_pids": [190749, 190948], "question_author": ""} {"qid": 155, "query": "Why didnt Harry feel the Cruciatus Curse at the end of Deathly Hallows?", "score": 99, "views": 59690, "answer_pids": [119080, 119087, 119124, 123716, 125470, 125746, 177541], "question_author": ""} {"qid": 156, "query": "Why was Susan treated so unkindly?", "score": 99, "views": 54579, "answer_pids": [169732, 169737, 169738, 169739, 169742, 169757, 169808, 169900], "question_author": "Noel M"} {"qid": 157, "query": "Why didnt Qui-Gon Jinn use his Jedi mind tricks to exchange his Republic credits?", "score": 99, "views": 12612, "answer_pids": [140616, 145430, 151132], "question_author": ""} {"qid": 158, "query": "Boy transported to another planet where the being who owns the Earth lives", "score": 99, "views": 7237, "answer_pids": [165740], "question_author": "Agent_9191"} {"qid": 159, "query": "Why does the paper in Battlestar Galactica have the corners cut off?", "score": 98, "views": 46795, "answer_pids": [115603, 115606, 115656, 118419, 123918, 129521, 196250], "question_author": "Paul Lammertsma"} {"qid": 160, "query": "How valuable are C-3PO and R2-D2?", "score": 98, "views": 16399, "answer_pids": [176802, 176804, 176807, 176826, 176829, 176833, 176848, 176849, 176853, 186399], "question_author": ""} {"qid": 161, "query": "Why could Quirrell tolerate Harrys touch at the beginning of Philosophers Stone?", "score": 97, "views": 90180, "answer_pids": [122076, 128671, 140063, 156321], "question_author": "lavonardo"} {"qid": 162, "query": "Why werent the Three Rings for the Elven-kings destroyed as well?", "score": 97, "views": 24795, "answer_pids": [110615, 110617, 134887, 207304], "question_author": "txwikinger"} {"qid": 163, "query": "Why was Sirius ever convicted?", "score": 97, "views": 20539, "answer_pids": [146485, 146486, 146549, 146564, 146572, 146576, 157455], "question_author": "Richard Gadsden"} {"qid": 164, "query": "Has an injured Flash ever used super-speed to walk on his hands?", "score": 97, "views": 13019, "answer_pids": [167006, 167015, 167023, 167037], "question_author": "mmccoo"} {"qid": 165, "query": "Which was the first novel set in universes where P=NP?", "score": 97, "views": 8999, "answer_pids": [109514, 109518, 109527, 110939, 111868, 111944, 113490, 159878], "question_author": ""} {"qid": 166, "query": "Whats the best order to read HP Lovecrafts novels?", "score": 96, "views": 237381, "answer_pids": [110198, 110212, 110359, 124596, 149555, 149557], "question_author": "PearsonArtPhoto"} {"qid": 167, "query": "Why did Lucas begin the episode numbering at IV?", "score": 96, "views": 178775, "answer_pids": [109605, 109624, 113960, 138485], "question_author": "Richard Gadsden"} {"qid": 168, "query": "Why Was It Essential That Voldemort Kill Harry Potter?", "score": 96, "views": 141977, "answer_pids": [117538, 117547, 128773, 132416, 141328, 143288, 144883, 154908, 156941, 164872, 181062], "question_author": ""} {"qid": 169, "query": "Where is the rest of Yodas species?", "score": 96, "views": 44585, "answer_pids": [112163], "question_author": "eipipuz"} {"qid": 170, "query": "Why didnt Frodo take a map with him?", "score": 96, "views": 23659, "answer_pids": [173256, 173260, 173261, 173294, 173305, 173348, 173363], "question_author": ""} {"qid": 171, "query": "How come the Federation did not routinely use projectile weapons against the Borg?", "score": 96, "views": 22585, "answer_pids": [109861, 109862, 109864, 109873, 109875, 112469, 113998, 116684, 119581, 119739, 122592, 126752, 127491, 178266], "question_author": ""} {"qid": 172, "query": "Story about humanity realizing they are in a simulation", "score": 96, "views": 14512, "answer_pids": [179350, 179359, 179368, 188463], "question_author": "mmccoo"} {"qid": 173, "query": "Why did Harry Potter intentionally lose the Resurrection Stone in the Forbidden Forest?", "score": 95, "views": 567393, "answer_pids": [111340, 111341, 111350, 111351, 111538, 112221, 112471, 113720, 118798, 131463, 134902, 205472], "question_author": "rjstreet"} {"qid": 174, "query": "Why didnt Saruman take Narya from Gandalf?", "score": 95, "views": 29249, "answer_pids": [115863, 115880, 121527, 122966, 124302, 125968, 126358, 127918, 129499, 129501, 135521, 151048], "question_author": "David G"} {"qid": 175, "query": "How does Gandalf (or anyone) know how to destroy the Ring?", "score": 95, "views": 17353, "answer_pids": [179180, 179186, 179187, 179195, 179216], "question_author": "ahockley"} {"qid": 176, "query": "Why does Millennium Falcon bank when turning in vacuum?", "score": 95, "views": 15617, "answer_pids": [112859, 112860, 112861, 112866, 112869, 112872, 112876, 112931, 142623, 184530], "question_author": "Agent_9191"} {"qid": 177, "query": "Since when has Darth Vader had a sense of humor?", "score": 94, "views": 24036, "answer_pids": [171375, 171377, 171647], "question_author": "PearsonArtPhoto"} {"qid": 178, "query": "Does Aragorn wear pants?", "score": 94, "views": 20871, "answer_pids": [164911, 164926, 164927, 164950, 164951], "question_author": "John Rudy"} {"qid": 179, "query": "Why were the Death Eaters so lenient during the fight at the Department of Mysteries?", "score": 94, "views": 19241, "answer_pids": [120374, 120376, 120379, 120399, 133656, 134535, 135989, 142418], "question_author": "C. Ross"} {"qid": 180, "query": "How do the Ninja Turtles get the money to pay for their pizza?", "score": 94, "views": 16718, "answer_pids": [175812], "question_author": "lavonardo"} {"qid": 181, "query": "Does Lwaxana Troi ever talk to herself (as the computer voice)?", "score": 94, "views": 8315, "answer_pids": [163979], "question_author": ""} {"qid": 182, "query": "Why didnt the Order of the Phoenix assist in hunting Horcruxes?", "score": 93, "views": 18034, "answer_pids": [191063, 191064, 191065], "question_author": "Don"} {"qid": 183, "query": "Why didn\u2019t EVE recognize the little cockroach as a living organism?", "score": 93, "views": 16944, "answer_pids": [195819, 195820, 195826], "question_author": ""} {"qid": 184, "query": "What kind of creature is Pac-Man?", "score": 93, "views": 16121, "answer_pids": [179462, 179476], "question_author": "Mathias"} {"qid": 185, "query": "What is the logical reason for bullying Spock?", "score": 93, "views": 15780, "answer_pids": [138191, 138193, 138197, 138212, 138224, 138261, 138301, 138368], "question_author": ""} {"qid": 186, "query": "How is Gandalf the White a significantly more powerful figure than Gandalf the Gray?", "score": 92, "views": 163556, "answer_pids": [116148, 129598, 139256], "question_author": ""} {"qid": 187, "query": "Why couldnt the Jedi detect Senator Palpatine as having powers?", "score": 92, "views": 91095, "answer_pids": [116423, 117992, 150691], "question_author": ""} {"qid": 188, "query": "How was the Chosen One meant to bring balance to the Force?", "score": 92, "views": 77864, "answer_pids": [121491, 121493, 121495, 121497, 125546, 133687, 134380, 134382, 146631, 147822, 158190, 160413, 163661, 167829, 168693], "question_author": "Morgan May"} {"qid": 189, "query": "Are there any programmers in Star Trek?", "score": 92, "views": 16710, "answer_pids": [133152, 133153, 133154, 133155, 133158, 133162, 133174, 133203, 133281, 146451, 151267, 153485, 187400], "question_author": "lavonardo"} {"qid": 190, "query": "Where is Hill Valley?", "score": 92, "views": 13747, "answer_pids": [153373, 153378, 153445], "question_author": ""} {"qid": 191, "query": "Why would Shelob own Sauron?", "score": 92, "views": 13320, "answer_pids": [149356, 149360, 149367], "question_author": "Josh Goldshlag"} {"qid": 192, "query": "Why does Prof. Farnsworth run Planet Express?", "score": 92, "views": 12320, "answer_pids": [170567, 170584], "question_author": ""} {"qid": 193, "query": "Why did Anakin lose against Obi-Wan in Episode 3 - Revenge of the Sith?", "score": 91, "views": 107243, "answer_pids": [111933, 120913, 121006, 133737, 136821, 139049, 140513, 143461, 144496, 146868, 147356, 172705], "question_author": ""} {"qid": 194, "query": "Why isnt Captain America worthy of Mjolnir?", "score": 91, "views": 77402, "answer_pids": [146337, 146339, 146356, 146426, 146464, 147191, 196766, 199482, 199604], "question_author": "kchau"} {"qid": 195, "query": "Why didnt the Dursleys gladly ship Harry off to Hogwarts?", "score": 91, "views": 65651, "answer_pids": [137673, 137674, 137675, 137676, 137678, 137684, 137685, 137744, 137827, 146130], "question_author": ""} {"qid": 196, "query": "Why was Newt Scamanders name on the Marauders Map when Fred and George gave it to Harry?", "score": 91, "views": 40441, "answer_pids": [173424, 173428, 173438, 173477], "question_author": ""} {"qid": 197, "query": "Why Did Gringotts Pay Out for Harrys Firebolt?", "score": 91, "views": 39833, "answer_pids": [122268, 122390, 124647, 124648, 144159, 148221, 166390, 169430, 170161], "question_author": "Brisbe42"} {"qid": 198, "query": "Where do teachers sleep at Hogwarts?", "score": 91, "views": 37852, "answer_pids": [119606, 119610, 125213, 155459, 164320], "question_author": ""} {"qid": 199, "query": "In the Avengers movie, why is Hulks behaviour inconsistent?", "score": 91, "views": 36849, "answer_pids": [117445, 117645, 117651, 117923, 118199, 118371, 120783, 123015, 142902], "question_author": ""} {"qid": 200, "query": "Why was Hermione always late when she used the Time Turner?", "score": 91, "views": 31028, "answer_pids": [180307, 180309, 180312, 180321], "question_author": "pupeno"} {"qid": 201, "query": "Why arent there any sieges in Lord of The Rings?", "score": 91, "views": 16651, "answer_pids": [152622, 152623, 152624, 152626, 152641, 152643, 152644, 152650, 152688, 152791], "question_author": ""} {"qid": 202, "query": "Were there science fiction stories written during the Middle Ages?", "score": 91, "views": 12952, "answer_pids": [168859, 168870, 168872, 168873], "question_author": ""} {"qid": 203, "query": "Is this Beverly Crusher?", "score": 91, "views": 10024, "answer_pids": [152440], "question_author": "ICodeForCoffee"} {"qid": 204, "query": "In which episodes did the Enterprise boldly go where no one had gone before?", "score": 91, "views": 8150, "answer_pids": [174501, 174584], "question_author": ""} {"qid": 205, "query": "In what order should Tolkiens writings on Middle-earth be read?", "score": 90, "views": 340620, "answer_pids": [117323, 117324, 117325, 144351, 150895], "question_author": ""} {"qid": 206, "query": "Why was the Black Pearl so special?", "score": 90, "views": 40063, "answer_pids": [175115, 175122], "question_author": "keithjgrant"} {"qid": 207, "query": "Why is Cersei commonly referred to as Cersei Lannister instead of Cersei Baratheon?", "score": 90, "views": 31356, "answer_pids": [119425, 119432, 164322], "question_author": ""} {"qid": 208, "query": "Is there any proof that Jaime Lannister is actually a good swordsman?", "score": 90, "views": 29384, "answer_pids": [146376, 146379, 179095], "question_author": "ahockley"} {"qid": 209, "query": "Was Voyager really significantly slower than the Enterprise-D?", "score": 90, "views": 19203, "answer_pids": [142465, 142467, 142468, 142484, 142493], "question_author": ""} {"qid": 210, "query": "Why did the Fellowship not choose to pass the Misty Mountains further north or south?", "score": 90, "views": 19072, "answer_pids": [126105, 126106, 126107, 126111, 126142, 141992, 180879], "question_author": ""} {"qid": 211, "query": "What is this green alien supposed to be on the American covers of the Hitchhikers Guide to the Galaxy?", "score": 90, "views": 18010, "answer_pids": [199043, 199050, 199065, 199066], "question_author": "Michiel de Mare"} {"qid": 212, "query": "Who is the narrator of Star Wars?", "score": 90, "views": 17062, "answer_pids": [201787, 201809, 201811, 201816], "question_author": "Commodore Jaeger"} {"qid": 213, "query": "From which Star Trek TNG episode/movie has this famous facepalm image been taken?", "score": 90, "views": 15054, "answer_pids": [183291], "question_author": null} {"qid": 214, "query": "Why didnt Harry Potter play as the king?", "score": 90, "views": 12717, "answer_pids": [179560, 179571, 179601], "question_author": ""} {"qid": 215, "query": "What order should the Babylon 5 movies and series be watched in?", "score": 89, "views": 470962, "answer_pids": [114315, 120188, 123555, 124181, 124246, 128598, 155311, 208087], "question_author": "Maniero"} {"qid": 216, "query": "What are Gandalfs powers?", "score": 89, "views": 123802, "answer_pids": [123566, 123570, 123573, 123579, 124906, 131350, 131597, 134722, 144021], "question_author": null} {"qid": 217, "query": "Why do the lightsaber moves of Luke Skywalker look so uncoordinated and crude compared to the prequels?", "score": 89, "views": 46429, "answer_pids": [129629, 129630, 129631, 129636, 129645, 129651, 129652, 129655, 129665, 129670, 134478, 139052, 142631, 148446, 148486, 154922], "question_author": ""} {"qid": 218, "query": "Why Did the Machines Even Bother With a Matrix?", "score": 89, "views": 45435, "answer_pids": [115534, 115540, 115545, 115601, 115605, 115704, 131746], "question_author": ""} {"qid": 219, "query": "If the Galactic Empire had over 25,000 Star Destroyers, why were only 27 at the Battle of Endor?", "score": 89, "views": 41343, "answer_pids": [177533, 177538, 177539, 177542, 177543, 177545, 177550, 177575, 177656, 177719, 186463, 206723], "question_author": ""} {"qid": 220, "query": "Why didnt Dumbledore realise that Moody had been replaced?", "score": 89, "views": 36870, "answer_pids": [127926, 127927, 130170, 193236], "question_author": ""} {"qid": 221, "query": "Does Deadpool know he is breaking the 4th wall?", "score": 89, "views": 33880, "answer_pids": [160805, 160806, 160816, 160818, 160853, 160880], "question_author": "Maniero"} {"qid": 222, "query": "Was there ever any actual Spaceballs merchandise?", "score": 89, "views": 6342, "answer_pids": [208256, 208290], "question_author": "Maniero"} {"qid": 223, "query": "Is there a hidden significance behind the fact Galadriel gave 3 hairs to Gimli instead of one?", "score": 88, "views": 211009, "answer_pids": [122595, 122608, 130040, 130272, 146496, 180410], "question_author": ""} {"qid": 224, "query": "Why was the identity of the Half-Blood Prince important to the story?", "score": 88, "views": 64467, "answer_pids": [169292, 169316], "question_author": ""} {"qid": 225, "query": "Why did Eddard Stark keep Catelyn in the dark?", "score": 88, "views": 27895, "answer_pids": [165432, 165517, 165518, 165562], "question_author": ""} {"qid": 226, "query": "Why does an X-Wing have this name when there is no X in the Aurebesh alphabet?", "score": 88, "views": 23865, "answer_pids": [120431, 120432, 139606, 139627], "question_author": ""} {"qid": 227, "query": "Harry Potter: Why 7?", "score": 88, "views": 23452, "answer_pids": [116932, 116940, 154058, 154059, 154161, 189017], "question_author": ""} {"qid": 228, "query": "Why couldnt Harry or anybody else see Peter Pettigrew aka Scabbers on the Marauders Map when Lupin could?", "score": 87, "views": 97261, "answer_pids": [115070, 115071, 115072, 115074, 115084, 115266, 127539, 146824, 160485, 166930, 180562, 186865], "question_author": ""} {"qid": 229, "query": "Are orcs and goblins really the same thing?", "score": 87, "views": 73766, "answer_pids": [123453, 123454, 123455, 123460, 123839, 124501, 129280, 129870, 143242, 144688], "question_author": "Imageree"} {"qid": 230, "query": "Did J.K Rowling have the Tom Marvolo Riddle anagram set up from the start?", "score": 87, "views": 58461, "answer_pids": [140474, 140479, 140481], "question_author": "Simon P Stevens"} {"qid": 231, "query": "Why does Darth Vader stop Boba Fett from shooting Chewie in The Empire Strikes Back?", "score": 87, "views": 50011, "answer_pids": [111942, 116908, 118662, 119745, 125520, 128106, 128114, 138506, 141510, 144445, 168744, 168745], "question_author": "Mark Rogers"} {"qid": 232, "query": "How does Cypher exit/enter the Matrix without an operator?", "score": 87, "views": 44860, "answer_pids": [115550, 115551, 115564, 115565, 115581, 115593, 128949, 143400], "question_author": "Stefano Borini"} {"qid": 233, "query": "Did Voldemort actually curse the job of Defense Against Dark Arts professor after being denied the position?", "score": 87, "views": 27925, "answer_pids": [114564, 136898], "question_author": ""} {"qid": 234, "query": "Since Voldemort knew that Dumbledore did not kill Grindelwald, why did he think that killing Snape was necessary for him to wield the Elder Wand?", "score": 87, "views": 18175, "answer_pids": [155881, 155883, 155884, 155890, 155895, 155897, 155899, 155904, 155915, 155933, 165962, 193486], "question_author": "frumious"} {"qid": 235, "query": "The use of the word gun in The Hobbit", "score": 87, "views": 14662, "answer_pids": [141985, 141990, 141991, 141993, 142054, 142148], "question_author": "WireGuy"} {"qid": 236, "query": "What is Hermione Grangers ethnicity?", "score": 86, "views": 111714, "answer_pids": [153542, 153544, 153545, 153547, 153551, 153552, 153555, 153559, 153571, 153626, 153641, 156481, 156484, 157430, 157487, 158089, 207816], "question_author": "JustJeff"} {"qid": 237, "query": "What is the reason that lightsabers have different colors?", "score": 86, "views": 91225, "answer_pids": [109950, 109957, 109972, 124281, 169230], "question_author": "Noel M"} {"qid": 238, "query": "Why doesnt Quark wear a headdress?", "score": 86, "views": 52094, "answer_pids": [124058, 126184, 127351, 143800, 180861, 186978, 186998], "question_author": "frumious"} {"qid": 239, "query": "Why hasnt Anakin Skywalkers Force Ghost advised Kylo Ren?", "score": 86, "views": 48870, "answer_pids": [157357, 157361, 157426, 157436, 158812, 161218, 172165, 173087], "question_author": ""} {"qid": 240, "query": "Werent there originally going to be nine Star Wars films?", "score": 86, "views": 32076, "answer_pids": [109708, 109724, 111870, 117143, 118517, 123143, 157728], "question_author": ""} {"qid": 241, "query": "Why specifically was Obi-Wan Kenobi the only hope of Princess Leia?", "score": 86, "views": 26443, "answer_pids": [143449, 143457, 143487, 164728, 186170], "question_author": ""} {"qid": 242, "query": "In Star Trek why is warp speed the ultimate litmus test on galactic acceptance of a civilisation?", "score": 86, "views": 18851, "answer_pids": [175231, 175232, 175233, 175234, 175242, 175245, 175249, 175257, 175259, 175272, 175287, 175291, 175305], "question_author": "Bryant"} {"qid": 243, "query": "Was Tolkien good at his day job?", "score": 86, "views": 17957, "answer_pids": [187372, 187375, 187422], "question_author": ""} {"qid": 244, "query": "What was Saurons plan for a Middle-earth conquered and ruled by him?", "score": 86, "views": 16191, "answer_pids": [189818, 189825, 189838, 189854, 189867], "question_author": ""} {"qid": 245, "query": "What does Gandalf intend to chat with Tom Bombadil about?", "score": 86, "views": 15491, "answer_pids": [152570, 152621], "question_author": "frumious"} {"qid": 246, "query": "What is the first reference to an internet of computers in science fiction?", "score": 86, "views": 10805, "answer_pids": [148898, 148902, 148906, 192026], "question_author": ""} {"qid": 247, "query": "Are the crew of the Enterprise-D famous?", "score": 86, "views": 7578, "answer_pids": [150249, 150251, 150252, 150269, 150304, 150319], "question_author": "Maniero"} {"qid": 248, "query": "Did Philip K. Dick watch Blade Runner before his death?", "score": 86, "views": 6343, "answer_pids": [109503, 181449], "question_author": ""} {"qid": 249, "query": "Why didnt Gimli know Moria had fallen?", "score": 85, "views": 314641, "answer_pids": [123054, 123055, 123058, 123061, 123359, 131005], "question_author": "Extrakun"} {"qid": 250, "query": "Why do some Jedi disappear when they die while others dont?", "score": 85, "views": 225753, "answer_pids": [111792, 111794, 113208, 144147], "question_author": "Tobiasopdenbrouw"} {"qid": 251, "query": "Exactly how secret was the reveal in The Empire Strikes Back?", "score": 85, "views": 40239, "answer_pids": [138469, 138472, 138477, 191798, 211638], "question_author": "C. Ross"} {"qid": 252, "query": "How can Ron know that Voldemort was in Slytherin if nobody knows his real identity?", "score": 85, "views": 23903, "answer_pids": [187894, 187898, 187906, 187913, 187920, 187930, 187945, 191249], "question_author": "yhw42"} {"qid": 253, "query": "Why did Gollum ever take off the ring in the first place?", "score": 85, "views": 22871, "answer_pids": [114144, 114147, 114163, 114621, 115831, 115842, 199409], "question_author": "C. Edwards"} {"qid": 254, "query": "Short story where human explorers accidentally blind an alien race", "score": 85, "views": 5844, "answer_pids": [178009, 179002], "question_author": "txwikinger"} {"qid": 255, "query": "Which are the Two Towers in The Lord of the Rings?", "score": 84, "views": 77394, "answer_pids": [136138, 136141, 136151, 166177, 177594, 190547], "question_author": "Samuel Herzog"} {"qid": 256, "query": "Were there any canon examples of Harry being an innately powerful wizard?", "score": 84, "views": 67889, "answer_pids": [118439, 118445, 125334, 134101, 141062, 147841, 176998, 194091, 211776], "question_author": ""} {"qid": 257, "query": "Why didnt Sauron guard Mount Doom?", "score": 84, "views": 26782, "answer_pids": [136005, 136006, 136007, 136046, 137647], "question_author": ""} {"qid": 258, "query": "The Matrix: When Neo and Trinity shoot security/police are they actually killing real people?", "score": 84, "views": 23072, "answer_pids": [122870, 122872, 122887, 144032], "question_author": "kristof"} {"qid": 259, "query": "Why was Han Solo acting like he\u2019s never seen Chewbacca\u2019s bowcaster before?", "score": 84, "views": 21340, "answer_pids": [156598, 156628, 157848, 158362, 160055], "question_author": "Stefano Borini"} {"qid": 260, "query": "Why do zombies eat our brains?", "score": 84, "views": 17296, "answer_pids": [160845, 160855, 160856], "question_author": ""} {"qid": 261, "query": "Does it Matter if Han Shot First?", "score": 84, "views": 15151, "answer_pids": [112494, 141322, 173585], "question_author": ""} {"qid": 262, "query": "Will Ezras lightsaber revolutionize dueling?", "score": 84, "views": 11318, "answer_pids": [161417, 161431, 161444], "question_author": "Bevan"} {"qid": 263, "query": "Who actually sent the assassin to kill Bran?", "score": 83, "views": 125048, "answer_pids": [111342, 117071, 117091, 185590, 196497], "question_author": ""} {"qid": 264, "query": "Why didnt James love and sacrifice for Lily protect her?", "score": 83, "views": 40135, "answer_pids": [122970, 124124, 127788, 172119], "question_author": ""} {"qid": 265, "query": "What happened to warp drives destroying the universe?", "score": 83, "views": 35024, "answer_pids": [110289, 110290, 156372], "question_author": "Don Kirkby"} {"qid": 266, "query": "Whats the official explanation for Harrys (and everyone elses) absence from muggle school?", "score": 83, "views": 19879, "answer_pids": [141038, 141039, 141040, 141041, 141043, 141044, 141047, 141050, 141055, 141064, 141095], "question_author": "tttppp"} {"qid": 267, "query": "Why Does Gandalf Think Rohan Should Stand and Fight?", "score": 83, "views": 18821, "answer_pids": [138925, 138927, 138957, 141648], "question_author": "tttppp"} {"qid": 268, "query": "What keeps a lightsaber from going on infinitely?", "score": 83, "views": 18174, "answer_pids": [111470, 111471, 111473, 111481], "question_author": "Massimo"} {"qid": 269, "query": "Why did Luke use his left hand to shoot?", "score": 83, "views": 17931, "answer_pids": [195264, 195268, 195365], "question_author": "Pr\u00e4riewolf"} {"qid": 270, "query": "Why do people risk death by joining Starfleet if not for money or preservation of their homes?", "score": 83, "views": 17577, "answer_pids": [144222, 144224, 144226, 144240, 144261, 144263, 144343, 145487, 181300, 195458], "question_author": "Stefano Borini"} {"qid": 271, "query": "Why did the Voyager Doctor have to scrub in?", "score": 83, "views": 8297, "answer_pids": [181106], "question_author": "Ioannis Karadimas"} {"qid": 272, "query": "Time Travel story where an imprisoned psychopath kills people from his past?", "score": 83, "views": 4381, "answer_pids": [174857], "question_author": ""} {"qid": 273, "query": "Movie with magical book showing events of any day in history, except missing days of the Gregorian calendar change", "score": 83, "views": 3697, "answer_pids": [193709, 193714], "question_author": ""} {"qid": 274, "query": "Why was Thors quinjet password Point Break?", "score": 82, "views": 751505, "answer_pids": [182284, 182285, 182363], "question_author": "RaYell"} {"qid": 275, "query": "Why did all the Ringbearers leave Middle-earth in the end?", "score": 82, "views": 509994, "answer_pids": [116238, 116278, 135793, 158546], "question_author": ""} {"qid": 276, "query": "Why does C-3PO have a silver lower leg?", "score": 82, "views": 241746, "answer_pids": [109636, 150043], "question_author": "Michiel de Mare"} {"qid": 277, "query": "Why, exactly, does kryptonite hurt Superman?", "score": 82, "views": 116194, "answer_pids": [143494, 143495, 143513, 180583], "question_author": "ahockley"} {"qid": 278, "query": "Is Big Brother an actual person in the novel 1984?", "score": 82, "views": 26132, "answer_pids": [138119, 138123, 138171], "question_author": ""} {"qid": 279, "query": "How did Sauron feed the armies of Mordor?", "score": 82, "views": 25768, "answer_pids": [173711, 173718, 173723], "question_author": ""} {"qid": 280, "query": "Why was Thor worthy to lift Mjolnir?", "score": 82, "views": 23027, "answer_pids": [177169, 177170, 177178], "question_author": ""} {"qid": 281, "query": "Does Data need to cut his hair?", "score": 82, "views": 13408, "answer_pids": [161964, 161965, 161966, 161967, 161970, 161998], "question_author": ""} {"qid": 282, "query": "Why did Gilderoy try to mend Harrys arm?", "score": 82, "views": 12890, "answer_pids": [178367, 178368, 178370, 178371, 178403, 178409, 178450], "question_author": "Toby Allen"} {"qid": 283, "query": "What makes the world so oblivious to Clark Kents secret?", "score": 82, "views": 12649, "answer_pids": [110131, 110132, 110133, 110134, 110455, 126004, 126017, 143591, 189348, 189361], "question_author": ""} {"qid": 284, "query": "Did Hermione ever fix her parents memories?", "score": 81, "views": 240484, "answer_pids": [111399, 160788], "question_author": "adrianbanks"} {"qid": 285, "query": "Who was The Phantom Menace?", "score": 81, "views": 81493, "answer_pids": [114551, 114562, 114566, 114574, 141221, 154221, 155479, 179423, 179850], "question_author": ""} {"qid": 286, "query": "Can anyone explain the Star Trek astrography (i.e. quadrants)?", "score": 81, "views": 67239, "answer_pids": [109854, 109855], "question_author": "Noel M"} {"qid": 287, "query": "Was the Eye of Sauron Saurons actual physical form?", "score": 81, "views": 57379, "answer_pids": [122795, 122796, 138073, 161528], "question_author": ""} {"qid": 288, "query": "What is the Jokers real name?", "score": 81, "views": 54088, "answer_pids": [112167, 130514, 136673, 151053], "question_author": "Don"} {"qid": 289, "query": "What is written on Darth Vaders Chest control panel?", "score": 81, "views": 49016, "answer_pids": [120871, 120880, 134294], "question_author": ""} {"qid": 290, "query": "In Stargate, how can you have a point of origin with only one coordinate?", "score": 81, "views": 26653, "answer_pids": [109581, 109582, 109583, 109584, 109588, 110172, 110482, 110580, 138980], "question_author": "adrianbanks"} {"qid": 291, "query": "Whats the purpose of the earpiece Agent Smith is wearing?", "score": 81, "views": 16838, "answer_pids": [172596, 172603, 172661], "question_author": ""} {"qid": 292, "query": "What lies in Mordor besides Mount Doom and Barad-dur?", "score": 81, "views": 15651, "answer_pids": [188382, 188383, 188386], "question_author": "Imageree"} {"qid": 293, "query": "Safety railings in the Star Wars universe", "score": 81, "views": 15272, "answer_pids": [121940, 121941, 168642, 170172], "question_author": ""} {"qid": 294, "query": "What was the tallest structure built in Middle-earth?", "score": 81, "views": 13851, "answer_pids": [173458, 173471], "question_author": ""} {"qid": 295, "query": "Sol \u2162 = Earth: What is the origin of this planetary naming scheme?", "score": 81, "views": 12147, "answer_pids": [200061, 200062, 200072, 200171], "question_author": "Noel M"} {"qid": 296, "query": "Why was Dawlish not fired as an Auror, given his incompetence?", "score": 81, "views": 11332, "answer_pids": [178373, 178374, 178376, 178380], "question_author": ""} {"qid": 297, "query": "Why four lights?", "score": 80, "views": 103963, "answer_pids": [115434, 115435, 115436, 115437, 115438, 116721, 180456], "question_author": "Albort"} {"qid": 298, "query": "Why did Batman do so much better in his second fight with Bane?", "score": 80, "views": 37840, "answer_pids": [166197, 166219, 166224, 166356, 208088], "question_author": "Albort"} {"qid": 299, "query": "Is there an episode of Game of Thrones where nobody dies?", "score": 80, "views": 28413, "answer_pids": [135489, 135517], "question_author": ""} {"qid": 300, "query": "Why is there an adult Dianoga living on a brand new Death Star?", "score": 80, "views": 21558, "answer_pids": [138636, 138650], "question_author": ""} {"qid": 301, "query": "Why didnt Harry want to be placed in Slytherin?", "score": 80, "views": 20989, "answer_pids": [149250, 149251, 149283, 158271, 164793], "question_author": null} {"qid": 302, "query": "When Did George Lucas Make Up the Stuff About the Sith and Midichlorians?", "score": 80, "views": 20192, "answer_pids": [112930, 113021, 113153, 148127], "question_author": ""} {"qid": 303, "query": "They ARE Klingons, and its a long story... Whats the story?", "score": 80, "views": 19587, "answer_pids": [173900, 173901, 173959], "question_author": "Joel in G\u00f6"} {"qid": 304, "query": "Are Tolkiens orcs immortal?", "score": 80, "views": 19144, "answer_pids": [136436, 136438, 152453, 167068, 167084, 209492], "question_author": ""} {"qid": 305, "query": "What evidence is there supporting an alternative parentage for Jon Snow?", "score": 80, "views": 18931, "answer_pids": [112601, 112629, 112746, 122335, 123068, 130156, 138207, 165824], "question_author": "Rismo"} {"qid": 306, "query": "Did Luke play any significant role in \u201cReturn of the Jedi?\u201d", "score": 80, "views": 16071, "answer_pids": [172405, 172407, 172409, 172505], "question_author": "Piotr Zurek"} {"qid": 307, "query": "What language(s) were Yodas speech patterns based on?", "score": 80, "views": 15464, "answer_pids": [175870, 175879, 175887], "question_author": ""} {"qid": 308, "query": "Star Wars Imperial Ships: Oversized?", "score": 80, "views": 14559, "answer_pids": [113036, 113037, 113038, 113085, 152171], "question_author": ""} {"qid": 309, "query": "In Episode VI, what did the good guys really accomplish since the bad guys are back stronger in Episode VII?", "score": 80, "views": 13124, "answer_pids": [158764, 158819, 158894], "question_author": ""} {"qid": 310, "query": "Why cant Transporter technology be used as a weapon?", "score": 80, "views": 12917, "answer_pids": [116251, 116252, 116254, 116256, 116275, 128676, 135698, 142749, 145277], "question_author": "Hendrik Brummermann"} {"qid": 311, "query": "Why not encase the One Ring in solid metal?", "score": 80, "views": 12893, "answer_pids": [157975, 157987], "question_author": "DavRob60"} {"qid": 312, "query": "Why are there so few robots in Star Trek?", "score": 80, "views": 11788, "answer_pids": [113099, 113100, 113102, 113103, 113105, 113123, 115331, 142904, 142905], "question_author": "John Rudy"} {"qid": 313, "query": "Why was Hitchhikers Guide censored?", "score": 80, "views": 9845, "answer_pids": [163581], "question_author": ""} {"qid": 314, "query": "Why is there no night shift in Monsters, Inc.?", "score": 80, "views": 9769, "answer_pids": [210023, 210029, 210050], "question_author": ""} {"qid": 315, "query": "Is there an identifiable origin for the association of minotaurs with battle axes?", "score": 80, "views": 8663, "answer_pids": [188706, 188729, 188730], "question_author": ""} {"qid": 316, "query": "What went down at the July 1974 Comic Art Con?", "score": 80, "views": 8324, "answer_pids": [177440, 196983], "question_author": "Brenton Taylor"} {"qid": 317, "query": "Why couldnt Harry see Thestrals at the end of Goblet of Fire?", "score": 79, "views": 107340, "answer_pids": [113287, 129217, 132709, 132721, 172450, 186758], "question_author": ""} {"qid": 318, "query": "Why didnt Elrond or C\u00edrdan take the ring from Isildur by force and destroy it?", "score": 79, "views": 55958, "answer_pids": [127479, 127486, 127487, 128940, 131188, 131205, 168900, 176895, 183534], "question_author": "Scott Ferguson"} {"qid": 319, "query": "What was Shepherd Books past in Firefly?", "score": 79, "views": 31686, "answer_pids": [109542, 111311, 121470, 128391, 149696], "question_author": "John Rudy"} {"qid": 320, "query": "How do humans dominate the Federation in Star Trek?", "score": 79, "views": 25794, "answer_pids": [110017, 110018, 110025, 110071, 110621, 111818, 112847, 112878, 113932, 126714], "question_author": ""} {"qid": 321, "query": "Why did Voldemort assume that no-one knew about Room of Hidden Things?", "score": 79, "views": 23354, "answer_pids": [113609, 113613, 113614, 113615, 113619, 113661, 113735, 120205, 124125, 165366, 165375, 169372], "question_author": "Hamish Downer"} {"qid": 322, "query": "Why does the Star Wars universe use such a primitive technology as data-tapes?", "score": 79, "views": 19796, "answer_pids": [182719, 182721, 182723, 182764, 182786, 182807, 182845], "question_author": "C. Ross"} {"qid": 323, "query": "Why did Sauron think that no one in possession of the One Ring would want to destroy it?", "score": 79, "views": 17773, "answer_pids": [183492, 183494, 183502, 183503, 183509, 183526], "question_author": ""} {"qid": 324, "query": "Where does Gandalf Live?", "score": 79, "views": 17259, "answer_pids": [120684, 124178, 128897, 131082, 148475], "question_author": "Craig Walker"} {"qid": 325, "query": "How do the Turtles get their pizza?", "score": 79, "views": 16705, "answer_pids": [139156, 139159, 139161, 139185], "question_author": "chris"} {"qid": 326, "query": "Why did the Terminator need sunglasses?", "score": 79, "views": 16075, "answer_pids": [149163, 149164, 149177, 149308], "question_author": ""} {"qid": 327, "query": "In Tolkiens opinion, does Tom Bombadil (and the concept of pacifism) actually hold any power?", "score": 79, "views": 14082, "answer_pids": [168246, 168248, 168249, 168262, 168277, 168328], "question_author": "chills42"} {"qid": 328, "query": "Does anyone know what this fantasy script is from?", "score": 79, "views": 11928, "answer_pids": [177062, 177069], "question_author": "DampeS8N"} {"qid": 329, "query": "What did Godric Gryffindor have a sword for, anyway?", "score": 79, "views": 8936, "answer_pids": [146851], "question_author": ""} {"qid": 330, "query": "Where did Rockets repair tool come from?", "score": 79, "views": 8339, "answer_pids": [184946], "question_author": ""} {"qid": 331, "query": "What is the viewing order of the re-imagined Battlestar Galactica?", "score": 78, "views": 264890, "answer_pids": [110871, 110873, 129854], "question_author": ""} {"qid": 332, "query": "What is the true origin of the orcs?", "score": 78, "views": 175280, "answer_pids": [122354, 124254, 124255, 125743, 129974, 137338, 140825], "question_author": ""} {"qid": 333, "query": "Where did the Red Brick Road lead to?", "score": 78, "views": 139228, "answer_pids": [118857, 118893, 125304, 153800], "question_author": ""} {"qid": 334, "query": "Did the Ring Bearers and Gimli die in Valinor?", "score": 78, "views": 118212, "answer_pids": [128196, 128197, 128198, 130407, 144792, 151103], "question_author": "chills42"} {"qid": 335, "query": "Why did Obi-Wan leave Vader on Mustafar?", "score": 78, "views": 87728, "answer_pids": [114968, 114969, 120103, 121007, 130856, 130869], "question_author": "Rismo"} {"qid": 336, "query": "Why are there humans in the Star Wars Universe?", "score": 78, "views": 67955, "answer_pids": [111047, 111048, 111050, 111090, 115403, 121422, 129874, 138351, 138369, 149233], "question_author": "Rusty"} {"qid": 337, "query": "Why didnt Sauron fight in the final battle?", "score": 78, "views": 60882, "answer_pids": [112042, 112043, 112046, 112051, 115502, 146080, 162758, 167162, 171992], "question_author": ""} {"qid": 338, "query": "Why does Smaug have 4 legs in the 1st movie but only 2 legs in the 2nd?", "score": 78, "views": 28196, "answer_pids": [201220, 201236, 201237, 201259], "question_author": ""} {"qid": 339, "query": "What are the rules and limits of Accio spell?", "score": 78, "views": 27866, "answer_pids": [121861, 124879, 160037, 188308], "question_author": "Craig Walker"} {"qid": 340, "query": "Why arent there any fighter carriers in the Star Trek universe?", "score": 78, "views": 23549, "answer_pids": [112047, 112050, 112054, 112057, 112060, 112376, 112672, 113557, 118241, 127453, 129216, 134100, 168031], "question_author": ""} {"qid": 341, "query": "Explanation of seasons in A Song of Ice and Fire", "score": 78, "views": 18742, "answer_pids": [110559, 110562, 110563, 110585, 110854, 111649, 183621], "question_author": "John Rudy"} {"qid": 342, "query": "What do the Death Stars do, other than destroying planets?", "score": 78, "views": 17934, "answer_pids": [165939, 165940, 165942, 165985, 165987], "question_author": "TML"} {"qid": 343, "query": "Why did Moody suggest to Harry that he should consider becoming an Auror?", "score": 78, "views": 16008, "answer_pids": [180076, 180077, 180079, 180089, 180381], "question_author": "Brian Campbell"} {"qid": 344, "query": "Why did the One Ring betray Isildur?", "score": 78, "views": 15516, "answer_pids": [168891, 168894, 168895, 168896, 168897, 168899], "question_author": ""} {"qid": 345, "query": "Why isnt the Memory Charm (Obliviate) an Unforgivable Curse?", "score": 78, "views": 14819, "answer_pids": [169090, 169100, 169121, 181253], "question_author": ""} {"qid": 346, "query": "Why did Wood have Muggle golf balls?", "score": 78, "views": 11350, "answer_pids": [180387, 180388, 180394, 180402, 180434, 180484], "question_author": "RaYell"} {"qid": 347, "query": "How did Weird Al Yankovic predict The Phantom Menace?", "score": 78, "views": 7337, "answer_pids": [115215], "question_author": ""} {"qid": 348, "query": "How many episodes of Star Trek didnt happen?", "score": 78, "views": 7121, "answer_pids": [168030, 168038], "question_author": "morganpdx"} {"qid": 349, "query": "Why would Dumbledore say After all this time? to Snape?", "score": 77, "views": 249845, "answer_pids": [173329, 173330, 173370], "question_author": "Brenton Taylor"} {"qid": 350, "query": "If Obi-Wan and Yoda were trying to keep Luke safe from Vader, why would they let him keep the Skywalker name?", "score": 77, "views": 32674, "answer_pids": [113725, 113727, 113748, 113819, 155830, 155838, 175418], "question_author": "Nathan W"} {"qid": 351, "query": "What happened to Yoda?", "score": 77, "views": 20939, "answer_pids": [111168, 111169, 111427], "question_author": "chills42"} {"qid": 352, "query": "Why are Starfleet bridges positioned so vulnerably?", "score": 77, "views": 19900, "answer_pids": [157900, 157901, 157913, 157918, 157920, 157974, 157996, 158064, 158079, 166731, 184386, 204098, 204506], "question_author": "Rismo"} {"qid": 353, "query": "If Dumbledore is the most powerful wizard (allegedly), why would he work at a glorified boarding school? Is there a role with more responsibility?", "score": 77, "views": 18783, "answer_pids": [169244, 169251, 169269, 169327], "question_author": ""} {"qid": 354, "query": "In The Matrix, why is Neo confusing left and right?", "score": 77, "views": 18636, "answer_pids": [179817, 179823, 179842, 179883, 179909], "question_author": ""} {"qid": 355, "query": "Do the books ever say oliphaunts aren\u2019t elephants?", "score": 77, "views": 17703, "answer_pids": [199272, 199275, 199279, 199290], "question_author": "Pat Ludwig"} {"qid": 356, "query": "Does Sauron have any dialogue in any of the books?", "score": 77, "views": 17347, "answer_pids": [203238, 203705, 210514, 210548], "question_author": "Grant Palin"} {"qid": 357, "query": "Who has been killed by a Terminator, a Predator, and a Xenomorph?", "score": 77, "views": 15834, "answer_pids": [174226], "question_author": ""} {"qid": 358, "query": "How was construction of the Death Star(s) kept secret?", "score": 77, "views": 13496, "answer_pids": [154765, 154766, 154767, 154772, 154778], "question_author": ""} {"qid": 359, "query": "When were female captains banned from Starfleet?", "score": 77, "views": 12153, "answer_pids": [195902, 195910, 195916, 195936], "question_author": ""} {"qid": 360, "query": "Why was a young actor in age makeup cast in the Battlestar Galactica miniseries, rather than an older actor?", "score": 77, "views": 11293, "answer_pids": [176299], "question_author": ""} {"qid": 361, "query": "Why didnt Nero visit his homeworld to warn of its destruction?", "score": 77, "views": 9029, "answer_pids": [109408, 109410, 109783, 109786, 109892, 111916, 111975, 126573, 128562, 148163], "question_author": ""} {"qid": 362, "query": "Invisibility should cause blindness: how does hard sf cope?", "score": 77, "views": 4260, "answer_pids": [109828, 109829, 109837, 112192, 133016], "question_author": ""} {"qid": 363, "query": "Why did Tony Stark keep the electromagnet in his chest?", "score": 76, "views": 364161, "answer_pids": [121472, 123341, 124499, 126023, 126030], "question_author": ""} {"qid": 364, "query": "How Did \u00c9owyn Slay the Witch King of Angmar if He Wasnt Alive?", "score": 76, "views": 148768, "answer_pids": [114882, 114883, 114895, 118178, 123206, 123410, 126130, 128017], "question_author": "Numenetics"} {"qid": 365, "query": "Would Smaug have actually followed and allied with Sauron had he lived on?", "score": 76, "views": 50134, "answer_pids": [142325, 142517], "question_author": "MatthewMartin"} {"qid": 366, "query": "Why was the Holdo manoeuvre not used earlier?", "score": 76, "views": 33471, "answer_pids": [183570, 183618, 183640, 183710, 183753, 183793, 183794, 184233, 184266, 188142, 191006, 191025, 194847, 202090], "question_author": ""} {"qid": 367, "query": "Why was Spider-Man the only person to feel any physical sensation from this event in Infinity War?", "score": 76, "views": 31521, "answer_pids": [188108, 188109, 188111, 188114, 188117, 188118, 188120, 188130, 188132, 188640], "question_author": ""} {"qid": 368, "query": "Why is killing someone using the Avada Kedavra spell considered worse or different than killing him with any other means?", "score": 76, "views": 30115, "answer_pids": [131764, 131766, 131778, 131784, 131800, 131808, 149161], "question_author": ""} {"qid": 369, "query": "When Jedi lose parts of their body, does their midi-chlorian count and strength in the Force diminish?", "score": 76, "views": 29758, "answer_pids": [111063, 111066, 111072, 111076, 111089, 111411, 111417, 112698, 113147, 127002, 138377, 147830], "question_author": "Jack C Buel"} {"qid": 370, "query": "Was the rescue in Episode 6 Beyond the Wall achieved in a plausible time?", "score": 76, "views": 26713, "answer_pids": [179661, 179662, 179663, 179684, 179686, 179723, 179729, 179730], "question_author": "CodeToGlory"} {"qid": 371, "query": "Why didnt they make the dinosaurs all male?", "score": 76, "views": 24105, "answer_pids": [153602, 153603, 153605, 153608, 153614, 153615, 153616, 160780], "question_author": ""} {"qid": 372, "query": "In Star Trek, why do Federation / Starfleet ships always seem to delay returning fire until its almost too late?", "score": 76, "views": 22165, "answer_pids": [174146, 174148, 174162, 174185, 174214, 174282], "question_author": null} {"qid": 373, "query": "Why exactly do the Death Eaters serve Lord Voldemort?", "score": 76, "views": 20926, "answer_pids": [110332, 110333, 110354, 129997, 138189, 166152, 171865], "question_author": "JuanZe"} {"qid": 374, "query": "What was Starbuck\u2019s role?", "score": 76, "views": 16626, "answer_pids": [109639, 109640, 109641, 109642, 110192, 110193, 113108, 132300], "question_author": "morganpdx"} {"qid": 375, "query": "Is anyone else not buying the Buggers story?", "score": 76, "views": 16281, "answer_pids": [180524, 180529, 180536, 180538, 180545, 180561], "question_author": ""} {"qid": 376, "query": "Why does Gollum calling the ring his own not alert Sauron?", "score": 76, "views": 15969, "answer_pids": [136357, 136366, 136380, 136383, 136537, 172478, 175559], "question_author": "Piotr Zurek"} {"qid": 377, "query": "Was there ever a situation where a proposal from Worf was accepted?", "score": 76, "views": 12421, "answer_pids": [182897, 182931], "question_author": "Piotr Zurek"} {"qid": 378, "query": "Why would Vulcans be vegetarian in an era with replicators?", "score": 76, "views": 10293, "answer_pids": [112409, 112410, 112411, 112412, 112430, 113093, 113094, 126448], "question_author": ""} {"qid": 379, "query": "How does gravity work on Serenity?", "score": 76, "views": 8156, "answer_pids": [116304, 116305, 116479, 120026, 137072, 151025, 183328, 183867], "question_author": "heavilyinvolved"} {"qid": 380, "query": "Short story about Neurosurgeon uses nanobots to cure patients, reprograms ex-wife to fall back in love with him", "score": 76, "views": 3182, "answer_pids": [138907], "question_author": "CodeToGlory"} {"qid": 381, "query": "How did the Death Star move?", "score": 75, "views": 81184, "answer_pids": [118251, 118252, 172061], "question_author": "Rasmus"} {"qid": 382, "query": "How did Harry become a Horcrux?", "score": 75, "views": 37513, "answer_pids": [115114, 115117, 115118, 115120, 115121, 115124, 135147], "question_author": ""} {"qid": 383, "query": "What happened to Fawkes?", "score": 75, "views": 35422, "answer_pids": [161987, 161989, 161990, 161992], "question_author": "CodeToGlory"} {"qid": 384, "query": "Who created the first Transformer?", "score": 75, "views": 30406, "answer_pids": [116671, 116672, 116673, 116674], "question_author": "CodeToGlory"} {"qid": 385, "query": "Why did Palpatine care about Anakin Skywalker in the Prequels given Palpatines pretty good Force Vision?", "score": 75, "views": 27281, "answer_pids": [137843, 137844, 137852, 137863, 137869, 137878, 137931, 137933, 145622, 146923], "question_author": ""} {"qid": 386, "query": "Why did Peter Quill wait 26 years to open his mothers gift?", "score": 75, "views": 26910, "answer_pids": [144035, 144044], "question_author": ""} {"qid": 387, "query": "Was the mythology in Moana based on an established myth?", "score": 75, "views": 20555, "answer_pids": [183488, 183490, 183542], "question_author": ""} {"qid": 388, "query": "What are those sticks on Jyn Ersos back?", "score": 75, "views": 19890, "answer_pids": [171540, 171542, 171579], "question_author": "Piotr Zurek"} {"qid": 389, "query": "Did the robot that saved Spooners life in I, Robot break the 2nd law of robotics?", "score": 75, "views": 19191, "answer_pids": [178319, 178320, 178323, 178339, 178348, 178392, 178402], "question_author": ""} {"qid": 390, "query": "Why was Dolores Umbridge not arrested after the end of Harrys fifth year?", "score": 75, "views": 18015, "answer_pids": [185463, 185471, 185479], "question_author": ""} {"qid": 391, "query": "Why is The Wall so tall in Game of Thrones?", "score": 75, "views": 16660, "answer_pids": [121970, 121971, 121973, 121978, 130827, 140408, 148655], "question_author": ""} {"qid": 392, "query": "Why do Hogwarts teachers have such strange names?", "score": 75, "views": 14101, "answer_pids": [153089, 153093, 153102, 153103, 153121, 166879], "question_author": "Bill the Lizard"} {"qid": 393, "query": "Why does Battle School train kids in physical combat when its irrelevant to the ultimate goal?", "score": 75, "views": 13003, "answer_pids": [175664, 175665, 175668, 175691, 175699, 175701, 175708], "question_author": ""} {"qid": 394, "query": "Why dont Wizards use wrist straps to protect against disarming charms?", "score": 75, "views": 12733, "answer_pids": [200130, 200133, 200146, 200154, 200162, 200163, 200170], "question_author": ""} {"qid": 395, "query": "Are the mountains surrounding Mordor natural?", "score": 75, "views": 11615, "answer_pids": [171481, 171483], "question_author": ""} {"qid": 396, "query": "How did J.R.R. Tolkien learn about the events of his books? (In-Universe)", "score": 75, "views": 10286, "answer_pids": [152209, 152210, 152234], "question_author": ""} {"qid": 397, "query": "Why didnt Obi-Wan remember R2-D2 and C-3PO in A New Hope?", "score": 74, "views": 331677, "answer_pids": [124332, 126514, 129520, 135000, 136664, 142712, 145021, 147259, 158662], "question_author": ""} {"qid": 398, "query": "Is there a story behind the misquote, Luke, I am your father?", "score": 74, "views": 110159, "answer_pids": [162782, 162840], "question_author": ""} {"qid": 399, "query": "Why did Sm\u00e9agol turn into a creature when addicted to the ring, but not Bilbo Baggins?", "score": 74, "views": 72385, "answer_pids": [130829, 130831, 130833, 130835, 130837, 130840, 130859, 130860, 130895, 130906], "question_author": "WireGuy"} {"qid": 400, "query": "What order should I be reading the Discworld books in?", "score": 74, "views": 45055, "answer_pids": [109791, 109793, 109902, 152302, 164937], "question_author": ""} {"qid": 401, "query": "What episodes of Doctor Who should I watch first?", "score": 74, "views": 42389, "answer_pids": [110318, 110320, 110324, 110325, 110848, 111973, 111993, 112997, 113236, 113971], "question_author": "WireGuy"} {"qid": 402, "query": "Does Hogwarts teach non magical classes?", "score": 74, "views": 40706, "answer_pids": [112880, 112881, 135277, 141965, 187654], "question_author": "heavilyinvolved"} {"qid": 403, "query": "What did Darth Vader mean by \u201cThere\u2019ll be no one to stop us this time.\u201d Who had \u201cstopped\u201d Vader\u2014or the Empire\u2014previously?", "score": 74, "views": 29923, "answer_pids": [172271, 172272, 172314, 172362], "question_author": "Steve Homer"} {"qid": 404, "query": "Why did Peter Parker work as a poor photographer given that he was a top science student who could earn much more in Silicon Valley?", "score": 74, "views": 26484, "answer_pids": [180156, 180162, 180166, 180181, 180188, 180193, 180196, 180217, 180226], "question_author": ""} {"qid": 405, "query": "Why does the Batmobile have a NO STEP sign?", "score": 74, "views": 16578, "answer_pids": [146612, 146614, 146676, 146680, 146681, 146684, 146687, 148369], "question_author": ""} {"qid": 406, "query": "Out of Universe, was Vader always planned to be Lukes father?", "score": 74, "views": 16517, "answer_pids": [158937, 158939, 159007, 189685], "question_author": "Daniel O"} {"qid": 407, "query": "Why was the Triwizard cup portkey in Goblet of Fire two-way?", "score": 74, "views": 16018, "answer_pids": [118599, 118603, 118607, 118608, 164079, 195330], "question_author": ""} {"qid": 408, "query": "What color is smurf blood?", "score": 74, "views": 14688, "answer_pids": [122512], "question_author": ""} {"qid": 409, "query": "Would Gollum qualify, as a Ringbearer, to go to the Undying Lands?", "score": 74, "views": 14015, "answer_pids": [146353, 146354, 146371, 146415, 146510], "question_author": ""} {"qid": 410, "query": "Whats the big deal about the Nazg\u00fbl losing their horses?", "score": 74, "views": 13673, "answer_pids": [198713, 198755], "question_author": "Niall C."} {"qid": 411, "query": "Why do less-bright students end up in Slytherin?", "score": 74, "views": 13587, "answer_pids": [175484, 175489, 175491, 175493, 175494, 175503, 175525, 178260], "question_author": "Sebastian Dietz"} {"qid": 412, "query": "Why was the ...philosophers stone retitled to ... the sorcerers stone for the US Market?", "score": 74, "views": 12766, "answer_pids": [153807, 153813, 153852], "question_author": ""} {"qid": 413, "query": "Was Palpatine an effective ruler?", "score": 74, "views": 12457, "answer_pids": [153621, 153623, 153625, 153683], "question_author": "Alex Schr\u00f6der"} {"qid": 414, "query": "What is the largest (engineered) physical object dreamed of in science fiction?", "score": 74, "views": 11769, "answer_pids": [109484, 109485, 109486, 109487, 109488, 109490, 109494, 109495, 109497, 109530, 109576, 109997, 110001, 110009, 110011, 110021, 110022, 110029, 110046, 110049, 110996, 111000, 111034, 111115, 115685, 115686, 120534, 121989, 121993, 124794], "question_author": "Daniel O"} {"qid": 415, "query": "Why have most Superhero movies ditched the Secret Identity?", "score": 74, "views": 11743, "answer_pids": [163266, 163268, 163274, 163280, 163295, 163338], "question_author": null} {"qid": 416, "query": "Are Pokemon farmed and eaten?", "score": 74, "views": 11325, "answer_pids": [175818, 175920], "question_author": ""} {"qid": 417, "query": "Why did the Nazgul start on horseback?", "score": 74, "views": 9983, "answer_pids": [113345, 113346, 113354, 114908, 115295, 119697, 124955, 136766, 188548], "question_author": "Andrew \u0410\u043d\u0434\u0440\u0435\u0439 \u041b\u0438\u0441\u0442\u043e\u0447\u043a\u0438\u043d"} {"qid": 418, "query": "Can you tell my robot to kill itself? (Three Laws)", "score": 74, "views": 9706, "answer_pids": [128558, 128559, 128564, 128566, 128567, 128569, 128572, 128576, 131706, 131709, 142784], "question_author": "MatthewMartin"} {"qid": 419, "query": "Who or what inspired the character Geordi La Forge?", "score": 74, "views": 9122, "answer_pids": [200725], "question_author": "Andrew M"} {"qid": 420, "query": "Man serves a sentence for the murder he will be allowed to commit in the future", "score": 74, "views": 7146, "answer_pids": [141359], "question_author": ""} {"qid": 421, "query": "If the Borg assimilate a planet of idiots, does the Collective become dumber?", "score": 74, "views": 5726, "answer_pids": [116096, 116097, 116098, 116099, 116127], "question_author": "Mark Rogers"} {"qid": 422, "query": "Why did they name the movie Blade Runner?", "score": 73, "views": 72123, "answer_pids": [119974], "question_author": ""} {"qid": 423, "query": "Is Tolkiens Middle-earth in our Universe?", "score": 73, "views": 49801, "answer_pids": [120529, 120976, 147710], "question_author": "Dr Rob Lang"} {"qid": 424, "query": "What is the benefit of an AT-AT walker over a conventional tank?", "score": 73, "views": 46138, "answer_pids": [141855, 141857, 141861, 141868, 141889, 141914, 141918, 141966, 147812, 154603], "question_author": ""} {"qid": 425, "query": "Why is Sonic so fast?", "score": 73, "views": 43622, "answer_pids": [180198, 180201], "question_author": ""} {"qid": 426, "query": "Did the trio (and others) go back to finish at Hogwarts?", "score": 73, "views": 42709, "answer_pids": [120472], "question_author": "Reinstate Monica - Goodbye SE"} {"qid": 427, "query": "Why did Red Squadron give their call signs in random order?", "score": 73, "views": 35507, "answer_pids": [144589, 144596, 144597, 144599, 144645, 167331], "question_author": ""} {"qid": 428, "query": "At what time can you start feeding Mogwai?", "score": 73, "views": 27679, "answer_pids": [113837, 126731], "question_author": "dieki"} {"qid": 429, "query": "Why didnt Molly Weasley remember the platform number?", "score": 73, "views": 17679, "answer_pids": [139023, 139025, 139027, 139040, 147558, 150187], "question_author": ""} {"qid": 430, "query": "Why did Mark Hamill want everyone to \u201cflip the bird\u201d in tribute to Carrie Fisher?", "score": 73, "views": 15822, "answer_pids": [181331], "question_author": "Wazery"} {"qid": 431, "query": "What prevents Asimovs robots from locking all humans in padded cells for the humans protection?", "score": 73, "views": 14841, "answer_pids": [192751, 192752, 192756, 192758, 192767, 192770, 192800, 192808], "question_author": "C. Ross"} {"qid": 432, "query": "What was the Watcher in the water?", "score": 73, "views": 13567, "answer_pids": [122667, 153264, 203195, 208804, 209057], "question_author": "Marcus Downing"} {"qid": 433, "query": "Why does Superman wear his underwear outside his pants?", "score": 73, "views": 13030, "answer_pids": [116111, 116112, 170078], "question_author": ""} {"qid": 434, "query": "Why didnt Gandalf and Elrond equip the Fellowship (especially hobbits) better?", "score": 73, "views": 12841, "answer_pids": [155455, 155457, 155534, 155788, 165555], "question_author": ""} {"qid": 435, "query": "How was Spider-Man persuaded to join the side he did?", "score": 73, "views": 12625, "answer_pids": [163728, 163734, 163756], "question_author": ""} {"qid": 436, "query": "Is Vader sincere with Luke in ESB?", "score": 73, "views": 10154, "answer_pids": [119848, 119849, 119850, 119851, 119855, 156909], "question_author": "Andrew \u0410\u043d\u0434\u0440\u0435\u0439 \u041b\u0438\u0441\u0442\u043e\u0447\u043a\u0438\u043d"} {"qid": 437, "query": "Were McCoys medical instruments Swedish salt shakers?", "score": 73, "views": 7033, "answer_pids": [165643], "question_author": ""} {"qid": 438, "query": "Has there ever been a good Sith?", "score": 72, "views": 80030, "answer_pids": [117405, 117421, 129252, 130852, 141550, 156107], "question_author": "Mnementh"} {"qid": 439, "query": "In the Aliens movie, what do the Xenomorphs eat?", "score": 72, "views": 69451, "answer_pids": [112210, 113473, 116789, 118451, 118455, 121177, 123931, 126838, 127584, 132448, 150572, 170957, 173634], "question_author": "Josh Goldshlag"} {"qid": 440, "query": "Why didnt Vader let Luke kill the Emperor, or even kill him himself?", "score": 72, "views": 62091, "answer_pids": [119854, 119856, 119857, 119870, 123979, 141928, 142303, 168723], "question_author": ""} {"qid": 441, "query": "How could Fred and George have figured the exact password to the Marauders map?", "score": 72, "views": 59077, "answer_pids": [121686], "question_author": ""} {"qid": 442, "query": "Why was Ron Weasley permitted to bring a rat to Hogwarts?", "score": 72, "views": 35111, "answer_pids": [117659, 163987, 172922], "question_author": "ICodeForCoffee"} {"qid": 443, "query": "So what was the answer to Varys riddle?", "score": 72, "views": 33242, "answer_pids": [177562, 177565, 177566, 177577, 177657], "question_author": ""} {"qid": 444, "query": "What is Deanna Trois accent?", "score": 72, "views": 31080, "answer_pids": [167189, 167193, 167239], "question_author": "Andrew \u0410\u043d\u0434\u0440\u0435\u0439 \u041b\u0438\u0441\u0442\u043e\u0447\u043a\u0438\u043d"} {"qid": 445, "query": "Was the Sorcerers (Philosophers) Stone really that well protected?", "score": 72, "views": 23648, "answer_pids": [136768, 136771, 136796, 136810, 136823, 136854, 189333, 193579], "question_author": "reuscam"} {"qid": 446, "query": "Why didnt Hermione get an Outstanding in her Defence Against the Dark Arts O.W.L.?", "score": 72, "views": 22323, "answer_pids": [176855, 176869, 176885, 176886, 210119], "question_author": ""} {"qid": 447, "query": "Do the villains know Batman has no superpowers?", "score": 72, "views": 18142, "answer_pids": [200269, 200270, 200303, 200322], "question_author": ""} {"qid": 448, "query": "Who is now the Lord of Casterly Rock?", "score": 72, "views": 16675, "answer_pids": [163702, 163703], "question_author": ""} {"qid": 449, "query": "Is any reason given for the low sea level?", "score": 72, "views": 15430, "answer_pids": [120163], "question_author": ""} {"qid": 450, "query": "What was Gandalfs plan for getting the Ring to Mt. Doom?", "score": 72, "views": 14602, "answer_pids": [112162, 112172, 112173, 171201], "question_author": "heavilyinvolved"} {"qid": 451, "query": "How did they have time to build Balin a tomb?", "score": 72, "views": 13976, "answer_pids": [185768, 185769], "question_author": "Michiel de Mare"} {"qid": 452, "query": "Whats the worst thing anyone could do in a fast-food restaurant?", "score": 72, "views": 13381, "answer_pids": [175619, 175632, 175641, 175657, 175690], "question_author": ""} {"qid": 453, "query": "Does Pinocchios nose grow only when he knows he is lying?", "score": 72, "views": 13077, "answer_pids": [191428], "question_author": ""} {"qid": 454, "query": "Were wizards initially intended to be much more common?", "score": 72, "views": 10586, "answer_pids": [187976, 187978, 187995, 188005], "question_author": ""} {"qid": 455, "query": "Did Dumbledore lie to Harry about how long he had James Potters invisibility cloak when he was examining it? If so, why?", "score": 72, "views": 7004, "answer_pids": [196059], "question_author": "LittleBobbyTables - Au Revoir"} {"qid": 456, "query": "Exactly when did Captain Picard perform his trick on Moriarty in Ship in a Bottle?", "score": 72, "views": 6675, "answer_pids": [191357, 191361], "question_author": null} {"qid": 457, "query": "What is this painting on the Enterprise showing?", "score": 72, "views": 6164, "answer_pids": [154883], "question_author": ""} {"qid": 458, "query": "If a Jedi knight were frozen in carbonite, would they be able to use their Force powers?", "score": 72, "views": 5598, "answer_pids": [112432, 112433, 112435, 119729, 119735], "question_author": "Stefano Borini"} {"qid": 459, "query": "How could Count Dooku lose against Anakin Skywalker in a lightsaber battle?", "score": 71, "views": 117837, "answer_pids": [114127, 122953, 123426, 128575, 129625, 136580, 138511, 141171], "question_author": "BigEndian"} {"qid": 460, "query": "What happened to the Hobbits and the Dwarves after the Elves left?", "score": 71, "views": 52533, "answer_pids": [110797, 110798, 128688], "question_author": ""} {"qid": 461, "query": "Why Did Sidious Tell Vader Who Luke Was?", "score": 71, "views": 49858, "answer_pids": [112396, 112404, 112408, 115784, 122036, 131514, 141964], "question_author": ""} {"qid": 462, "query": "Did Darth Sidious and Vader ever discover how to cheat death?", "score": 71, "views": 37675, "answer_pids": [114627, 114635, 114658, 171430, 202093], "question_author": ""} {"qid": 463, "query": "Why was the riddle on the way to Moria written in Elvish?", "score": 71, "views": 28175, "answer_pids": [118033, 118034, 118041, 143310], "question_author": ""} {"qid": 464, "query": "Why werent Fred and George Weasley Placed in Ravenclaw?", "score": 71, "views": 17196, "answer_pids": [154704, 154710, 171945, 173552, 173573], "question_author": "Jonathan Campbell"} {"qid": 465, "query": "Why do Tolkiens wizards look Human, and not Elven?", "score": 71, "views": 16654, "answer_pids": [136431, 136434, 136437, 136445, 136447, 136471], "question_author": ""} {"qid": 466, "query": "During the war of the ring did Sauron or Saruman actually win any major battles?", "score": 71, "views": 16519, "answer_pids": [144923, 144925, 144926, 144928, 144934, 144938, 144954, 145027, 145077, 177118], "question_author": ""} {"qid": 467, "query": "Why are energy weapons seen as more acceptable in childrens shows than guns that fire bullets?", "score": 71, "views": 16405, "answer_pids": [195101, 195109, 195124, 195151], "question_author": ""} {"qid": 468, "query": "Why didnt anyone stop Luke from leaving the second Death Star?", "score": 71, "views": 16074, "answer_pids": [145257, 145258, 145259, 145268, 145271, 145273], "question_author": "Mark Rogers"} {"qid": 469, "query": "How does Phineas put his shirt on?", "score": 71, "views": 15497, "answer_pids": [189669], "question_author": ""} {"qid": 470, "query": "Is Jango Fett inept?", "score": 71, "views": 13995, "answer_pids": [175239, 175247, 175260, 175263, 175288], "question_author": "Tim Post"} {"qid": 471, "query": "How did Molly Weasley make sauce out of nothing if Gamps Law states that this is impossible?", "score": 71, "views": 13348, "answer_pids": [148810, 148826, 148851, 148869], "question_author": "Jack"} {"qid": 472, "query": "Nude time travel in Terminator Universe", "score": 71, "views": 13055, "answer_pids": [111426, 111428, 111433, 111440, 111462], "question_author": ""} {"qid": 473, "query": "What is the architecture of the Floo Network?", "score": 71, "views": 11342, "answer_pids": [167222, 167226, 167227, 167228, 167232, 167264], "question_author": "Albort"} {"qid": 474, "query": "Book in which the mountain in the distance was a hole in the flat world", "score": 71, "views": 10460, "answer_pids": [199133], "question_author": ""} {"qid": 475, "query": "Does the death penalty exist in comics?", "score": 71, "views": 9710, "answer_pids": [126828, 126831, 126837, 126847, 126859, 158293], "question_author": "Scott Mitchell"} {"qid": 476, "query": "Why arent alien planets as varied in climate as the Earth?", "score": 71, "views": 9263, "answer_pids": [118918, 118919, 118920, 118921, 118923, 118926, 118927, 118928, 118929, 118934, 118940, 138877, 176442], "question_author": "Tim Post"} {"qid": 477, "query": "What research was being performed on Pinky and the Brain?", "score": 71, "views": 4009, "answer_pids": [136683], "question_author": "StuperUser"} {"qid": 478, "query": "What really happened to Voldemorts nose?", "score": 70, "views": 380148, "answer_pids": [118232, 126598, 130267, 164023, 166471], "question_author": "StuperUser"} {"qid": 479, "query": "How does Captain Americas shield work?", "score": 70, "views": 70056, "answer_pids": [112714, 112722], "question_author": "Narayana"} {"qid": 480, "query": "When will copyright restrictions expire on The Lord of the Rings?", "score": 70, "views": 58809, "answer_pids": [146513, 146551], "question_author": "Tim Post"} {"qid": 481, "query": "Why didnt Voldemort suspect Snape as a double agent during Philosophers Stone?", "score": 70, "views": 52821, "answer_pids": [118942, 147504, 149599, 176894], "question_author": "spinodal"} {"qid": 482, "query": "Was Vger responsible for the creation of the Borg?", "score": 70, "views": 52477, "answer_pids": [109610, 161985, 166190], "question_author": ""} {"qid": 483, "query": "Why Does Vader say \u201cNo disintegrations\u2026\u201d to Boba Fett in particular?", "score": 70, "views": 44358, "answer_pids": [116476, 116477, 116491, 124272], "question_author": "gabr"} {"qid": 484, "query": "How does Superman shave?", "score": 70, "views": 39562, "answer_pids": [111708, 111710, 111711, 111722, 112403, 112406, 125683, 126137, 163852], "question_author": "gabr"} {"qid": 485, "query": "Why didnt Gandalf just fight Smaug directly?", "score": 70, "views": 28383, "answer_pids": [175955, 175963, 175977, 175978, 176002], "question_author": "RaYell"} {"qid": 486, "query": "Why do the ellipses in the Star Wars opening crawl have four dots?", "score": 70, "views": 21620, "answer_pids": [154696, 154697, 154698, 154700, 154702, 154711], "question_author": ""} {"qid": 487, "query": "Why do the Teenage Mutant Ninja Turtles wear masks?", "score": 70, "views": 19661, "answer_pids": [137850, 137851, 137857, 137903, 137949, 138012], "question_author": "CoderHawk"} {"qid": 488, "query": "How is the Starkiller Base supposed to work more than one time, given how it draws its power?", "score": 70, "views": 19225, "answer_pids": [156592, 156632, 188301], "question_author": "akf"} {"qid": 489, "query": "Why did Stormtroopers leave their stations to watch lightsaber duel between Vader and Kenobi?", "score": 70, "views": 18525, "answer_pids": [194122, 194124, 194127, 194164], "question_author": ""} {"qid": 490, "query": "Why was this character made Grand Maester?", "score": 70, "views": 16907, "answer_pids": [197646, 197650, 197674, 197681, 197691, 197733], "question_author": "tttppp"} {"qid": 491, "query": "Was Luke in \u201cReturn of the Jedi\u201d really \u201cbetter\u201d than Obi Wan in \u201cRevenge of the Sith?\u201d", "score": 70, "views": 16805, "answer_pids": [172380, 172389, 172401], "question_author": "Brenton Taylor"} {"qid": 492, "query": "Is Joss Whedons Firefly based on the Traveller RPG he played?", "score": 70, "views": 16377, "answer_pids": [116346, 116687, 119534, 124865, 133667, 189521], "question_author": ""} {"qid": 493, "query": "Why isnt there ever any mention of the Americas in Harry Potter?", "score": 70, "views": 16316, "answer_pids": [112974, 114989, 129940, 131912, 158290], "question_author": "gabr"} {"qid": 494, "query": "Why does C-3PO talk to R2-D2 in English?", "score": 70, "views": 14754, "answer_pids": [135242, 135246, 135253, 135255, 135256, 135260, 135280, 135286, 135343], "question_author": ""} {"qid": 495, "query": "What is on Steve Rogers list of things to watch?", "score": 70, "views": 13321, "answer_pids": [133700], "question_author": "balexandre"} {"qid": 496, "query": "Did Tolkien create any characters based on people he knew?", "score": 70, "views": 10489, "answer_pids": [155273, 155274, 155276, 155281, 155282], "question_author": "Mikael Sundberg"} {"qid": 497, "query": "Elvish tattoo - can it be translated into English?", "score": 70, "views": 9892, "answer_pids": [150588, 150595], "question_author": ""} {"qid": 498, "query": "Why does Watney\u2019s notebook in The Martian have four holes?", "score": 70, "views": 9830, "answer_pids": [166867, 166932], "question_author": ""} {"qid": 499, "query": "Who was the fifth army in The Hobbit: The Battle of the Five Armies movie?", "score": 69, "views": 245468, "answer_pids": [141738, 141744, 141746, 141811, 142177, 144882], "question_author": ""} {"qid": 500, "query": "What is the recommended viewing order for young first-timers to watch Star Trek TV episodes and movies?", "score": 69, "views": 131341, "answer_pids": [112460, 112478, 112487, 112598, 114901, 120343, 122240, 125707, 128221, 129961, 132872], "question_author": ""} {"qid": 501, "query": "What is this man doing with a Starfleet mini-skirt?", "score": 69, "views": 92106, "answer_pids": [115018, 115020, 121647, 123126], "question_author": ""} {"qid": 502, "query": "What do wizard children do before attending Hogwarts?", "score": 69, "views": 74534, "answer_pids": [115251, 115258, 115265, 149868], "question_author": ""} {"qid": 503, "query": "Did Darth Vader ever talk to R2-D2 again?", "score": 69, "views": 47767, "answer_pids": [143098], "question_author": "DavRob60"} {"qid": 504, "query": "Are there other governments in The Hunger Games world?", "score": 69, "views": 43108, "answer_pids": [115150, 120109, 127409, 206743], "question_author": "GWLlosa"} {"qid": 505, "query": "Why was the Triwizard Cup used as the portkey?", "score": 69, "views": 31748, "answer_pids": [113163, 113164, 113166, 113172, 113192, 113231, 122061, 129814, 129815, 140158, 146386], "question_author": ""} {"qid": 506, "query": "Why is Belgium the rudest word in Hitchikers Guide to the Galaxy?", "score": 69, "views": 25365, "answer_pids": [144759, 144763, 144777], "question_author": ""} {"qid": 507, "query": "Why were the Monsters, Inc. monsters told that touching kids would kill them?", "score": 69, "views": 24006, "answer_pids": [139593, 139595, 139597, 139666, 139673], "question_author": ""} {"qid": 508, "query": "Why didnt a French actor play Captain Picard? What was the reaction to this on the part of the French?", "score": 69, "views": 22326, "answer_pids": [113938, 113949, 113952, 113962, 125318, 127975, 140415, 202889, 202892], "question_author": "C. Ross"} {"qid": 509, "query": "Why does a lightsaber turn off when a Jedi dies or drops it?", "score": 69, "views": 20961, "answer_pids": [112106], "question_author": ""} {"qid": 510, "query": "What term is being referred to with reflected-sound-of-underground-spirits?", "score": 69, "views": 16085, "answer_pids": [196719, 196720], "question_author": ""} {"qid": 511, "query": "Have the writers and actors of Game Of Thrones responded to its poor reception?", "score": 69, "views": 15580, "answer_pids": [197551], "question_author": "beggs"} {"qid": 512, "query": "Why do blaster bolts move slower than the speed of light?", "score": 69, "views": 15440, "answer_pids": [116347, 149386, 152339, 164837], "question_author": ""} {"qid": 513, "query": "Has there ever been an instance in Star Trek where someone beamed into a solid object?", "score": 69, "views": 15258, "answer_pids": [167963, 167973, 167989, 167990, 167997, 167998, 168008, 168014, 168022, 168027, 168029, 197891], "question_author": ""} {"qid": 514, "query": "Why did the Minister of Magic execute Dumbledores will?", "score": 69, "views": 14622, "answer_pids": [186179, 186189, 186195], "question_author": ""} {"qid": 515, "query": "Could Harry ruin the Regeneration potion by willingly giving blood?", "score": 69, "views": 11317, "answer_pids": [144680, 144682, 144687, 144715, 144717, 144758, 144776], "question_author": "RMorrisey"} {"qid": 516, "query": "Identify a movie in which aliens believed a sci-fi TV show to be true documentary", "score": 69, "views": 11086, "answer_pids": [173586], "question_author": ""} {"qid": 517, "query": "In which Deadpool comic does he kill over liking Star Wars prequels?", "score": 69, "views": 9270, "answer_pids": [175342], "question_author": "Stefano Borini"} {"qid": 518, "query": "Why was a hobbit the most unlikely creature imaginable to pick up The One Ring?", "score": 69, "views": 9015, "answer_pids": [114077, 114080, 114093, 114104, 114105, 116399, 122458, 138661], "question_author": "Powerlord"} {"qid": 519, "query": "Does each season of Game of Thrones roughly equal a book?", "score": 68, "views": 267584, "answer_pids": [110842, 129612, 136351, 147382], "question_author": ""} {"qid": 520, "query": "Who was The Chosen One: Luke or Anakin?", "score": 68, "views": 137533, "answer_pids": [111414, 111415, 123880, 124948, 152363, 167223, 174721], "question_author": null} {"qid": 521, "query": "After Star Trek: The Next Generation, can I skip to Voyager without watching Deep Space Nine and not get spoiled?", "score": 68, "views": 62819, "answer_pids": [109871, 109872, 120161, 137871], "question_author": "AJ Finch"} {"qid": 522, "query": "Were there ever any good Orcs?", "score": 68, "views": 61290, "answer_pids": [115314, 115316, 122138, 125390, 130698, 132117, 134352, 178022], "question_author": "matt burns"} {"qid": 523, "query": "How do the Incredible Hulks pants stay on but not his shirt?", "score": 68, "views": 39269, "answer_pids": [111965, 170406, 195247], "question_author": ""} {"qid": 524, "query": "Where did King Arthur get his sword, Excalibur?", "score": 68, "views": 25986, "answer_pids": [140962, 140964], "question_author": null} {"qid": 525, "query": "Is Agent Smith a bug in the Matrix or a virus?", "score": 68, "views": 25099, "answer_pids": [173101, 173104, 173114, 173128, 173140, 173145, 173146, 173150, 173152], "question_author": ""} {"qid": 526, "query": "What was the origin of the meme where Batman slaps Robin?", "score": 68, "views": 24058, "answer_pids": [152806], "question_author": "Stefano Borini"} {"qid": 527, "query": "Why did Harry Potter get a bedroom?", "score": 68, "views": 23477, "answer_pids": [198841, 198842], "question_author": ""} {"qid": 528, "query": "Was Gandalf meant to confront the Balrog?", "score": 68, "views": 20818, "answer_pids": [168612, 168613, 168617, 168625, 168626, 168628, 168672, 168673], "question_author": ""} {"qid": 529, "query": "Why is the clone army so...small?", "score": 68, "views": 19617, "answer_pids": [158873, 158874, 158878, 158888, 158953], "question_author": "Brian Campbell"} {"qid": 530, "query": "What does the blue pill in The Matrix actually do?", "score": 68, "views": 18452, "answer_pids": [128187, 128193, 128218, 133269, 141595, 148589], "question_author": ""} {"qid": 531, "query": "Do any Harry Potter characters turn from good to evil?", "score": 68, "views": 17136, "answer_pids": [161742, 161743, 161747, 161756, 161758, 161803, 161859, 179847], "question_author": ""} {"qid": 532, "query": "Are there any spells that a wizard/witch can cast without a wand in the Harry Potter universe?", "score": 68, "views": 16166, "answer_pids": [110341, 110343, 110344, 110345, 111574, 111746, 155475, 159704, 160768, 190779], "question_author": "Karel"} {"qid": 533, "query": "Why didnt Yoda or Obi-Wan tell Luke the truth about his father?", "score": 68, "views": 15884, "answer_pids": [152803, 152838, 152861, 152937], "question_author": "matt burns"} {"qid": 534, "query": "What would have happened if the Balrog of Moria had taken possession of The One Ring?", "score": 68, "views": 14291, "answer_pids": [121207], "question_author": ""} {"qid": 535, "query": "Which Sci-fi universe uses the most forms of FTL (Faster Than Light)?", "score": 68, "views": 14100, "answer_pids": [150299, 150305, 150306, 150308, 150310, 150311, 150315, 150323, 150324, 150333, 150349, 150383, 150410, 150425, 150498, 150504], "question_author": ""} {"qid": 536, "query": "In Orwells 1984, why does the book even exist?", "score": 68, "views": 12855, "answer_pids": [176146, 181784, 181785], "question_author": "moodygrrl"} {"qid": 537, "query": "If there are universal translators, why are some words/phrases not translated?", "score": 68, "views": 12456, "answer_pids": [112247, 119750, 124212, 147763, 147969], "question_author": "Stewbob"} {"qid": 538, "query": "Has the autodestruct sequence ever not been aborted on a Starfleet vessel?", "score": 68, "views": 12204, "answer_pids": [153864, 153865, 153872, 153968, 153975, 153978], "question_author": ""} {"qid": 539, "query": "Why was only one person used as the template for the Clone Army?", "score": 68, "views": 11983, "answer_pids": [188078, 188081, 188143], "question_author": ""} {"qid": 540, "query": "Do the Borg not wonder why everybody hates them?", "score": 68, "views": 11749, "answer_pids": [173170, 173171, 173175, 173177, 173217, 173250], "question_author": "Agent_9191"} {"qid": 541, "query": "Is this a laptop in the Weasleys house?", "score": 68, "views": 9467, "answer_pids": [173814, 173832], "question_author": "Brian Campbell"} {"qid": 542, "query": "Why didnt the IF want Peter Wiggin?", "score": 68, "views": 8078, "answer_pids": [121301, 121307, 121309, 121311, 121427, 180526], "question_author": ""} {"qid": 543, "query": "Are the Ferengi racially discriminated against by members of the Federation?", "score": 68, "views": 7660, "answer_pids": [172645, 172646, 172649, 172680, 209365], "question_author": "Scott Mitchell"} {"qid": 544, "query": "Was the casting of Alan Rickman as Snape based on the full knowledge of the future of that character?", "score": 68, "views": 7038, "answer_pids": [113337, 158238], "question_author": ""} {"qid": 545, "query": "How does the Doctor know how old he is?", "score": 68, "views": 6671, "answer_pids": [110490, 110491, 110494, 110608, 112013], "question_author": ""} {"qid": 546, "query": "Who was the old man at the beginning of The Force Awakens?", "score": 67, "views": 573141, "answer_pids": [156124, 156389], "question_author": "kristof"} {"qid": 547, "query": "Why did Luke Skywalker call Obi-Wan Ben?", "score": 67, "views": 71073, "answer_pids": [111807, 138444, 151161, 188279], "question_author": "Rapida"} {"qid": 548, "query": "Why didn\u2019t Voyager fly to the end of the Bajoran wormhole in the Gamma Quadrant?", "score": 67, "views": 64309, "answer_pids": [110032, 110073, 110077, 112467, 131575, 152135, 164949], "question_author": ""} {"qid": 549, "query": "Is there evidence to suggest that Supreme Leader Snoke is Darth Plagueis?", "score": 67, "views": 63197, "answer_pids": [156257, 156272, 156311, 169575, 202368], "question_author": "Brenton Taylor"} {"qid": 550, "query": "How to kill Wolverine?", "score": 67, "views": 55998, "answer_pids": [116691, 116692, 116698, 116700, 116705, 116711, 118008, 122280, 122281, 122283, 122886, 126503, 135583, 144753, 174218], "question_author": "Stefano Borini"} {"qid": 551, "query": "Why is Leia a princess?", "score": 67, "views": 43358, "answer_pids": [111773, 154204, 169344], "question_author": ""} {"qid": 552, "query": "How old should you be when you begin Jedi training?", "score": 67, "views": 26909, "answer_pids": [109893, 110067, 110110, 121016, 148162], "question_author": "LittleBobbyTables - Au Revoir"} {"qid": 553, "query": "How did Voldemort get his wand back?", "score": 67, "views": 26658, "answer_pids": [111283, 111285, 111297, 112555, 113014, 182180, 189264], "question_author": "paercebal"} {"qid": 554, "query": "Why is Wednesday Addams named ... Wednesday?", "score": 67, "views": 25404, "answer_pids": [140442, 140443, 140446, 190097], "question_author": "jedihawk"} {"qid": 555, "query": "When did Obi-Wan Kenobi learn that Anakin was dead?", "score": 67, "views": 20119, "answer_pids": [109424, 109425, 109468, 111808, 121005], "question_author": "dieki"} {"qid": 556, "query": "Why would Arya reveal her fighting ability?", "score": 67, "views": 19705, "answer_pids": [179485, 179486, 179489, 179492, 179498, 179506, 179532, 179557, 179598, 179637], "question_author": "chills42"} {"qid": 557, "query": "Can the Doctor be female?", "score": 67, "views": 19090, "answer_pids": [113842, 121663, 121666, 135283, 138299, 140391, 142453, 158629, 178454, 178475], "question_author": ""} {"qid": 558, "query": "Why hasnt Captain America been promoted?", "score": 67, "views": 18779, "answer_pids": [135348, 135352, 135358], "question_author": null} {"qid": 559, "query": "What does Sauron need with mithril?", "score": 67, "views": 17871, "answer_pids": [168629, 168645, 168649, 168652, 168726], "question_author": ""} {"qid": 560, "query": "Is there a passage in Lord of the Rings told from the perspective of an orc?", "score": 67, "views": 16343, "answer_pids": [189491, 189494, 189500, 189513], "question_author": ""} {"qid": 561, "query": "Are Stormtroopers paid or rewarded in any way for their service?", "score": 67, "views": 15204, "answer_pids": [160578, 160594, 182184], "question_author": ""} {"qid": 562, "query": "Why doesnt anyone run a DNA test on Batmans blood?", "score": 67, "views": 14590, "answer_pids": [146619, 146626, 146632, 146656], "question_author": ""} {"qid": 563, "query": "Has Snape ever given points to Gryffindor?", "score": 67, "views": 14322, "answer_pids": [181815], "question_author": "Pat Ludwig"} {"qid": 564, "query": "Why dont the students of Hogwarts write with ballpoint pens?", "score": 67, "views": 13908, "answer_pids": [177759, 177760, 184595], "question_author": ""} {"qid": 565, "query": "What was the purpose of having Zion?", "score": 67, "views": 13858, "answer_pids": [135017, 135018, 135019, 135141], "question_author": ""} {"qid": 566, "query": "How could Harry and Ron bypass Hogwarts security in a flying Ford Anglia?", "score": 67, "views": 11307, "answer_pids": [171213, 171214], "question_author": "Kjartan \u00de\u00f3r Kjartansson"} {"qid": 567, "query": "How did Faramir know that Aragorn was the King in the Houses of Healing?", "score": 67, "views": 11293, "answer_pids": [176612, 176613, 176614], "question_author": "ire_and_curses"} {"qid": 568, "query": "Does any man ever cross his arms?", "score": 67, "views": 11185, "answer_pids": [194137], "question_author": "Brenton Taylor"} {"qid": 569, "query": "Is the Star Trek (2009) movie considered canon?", "score": 67, "views": 10281, "answer_pids": [109414, 109415, 109426, 125028, 128315], "question_author": ""} {"qid": 570, "query": "Did Tolkien popularize the generic description of a wizard?", "score": 67, "views": 9932, "answer_pids": [143936, 143938, 143947, 143948, 143949, 166374], "question_author": "matt burns"} {"qid": 571, "query": "Are werewolves in the Forbidden Forest consistent with later explanations on werewolves?", "score": 67, "views": 7751, "answer_pids": [173779, 173782], "question_author": ""} {"qid": 572, "query": "Where did the concept of a (fantasy-style) dungeon originate?", "score": 67, "views": 7274, "answer_pids": [207587, 207591, 207616], "question_author": ""} {"qid": 573, "query": "Why was Crusher replaced with Pulaski who was replaced by Crusher?", "score": 66, "views": 317809, "answer_pids": [121034, 121039, 122979, 150374], "question_author": "Brian Campbell"} {"qid": 574, "query": "What was the intended ending of Monty Python and the Holy Grail?", "score": 66, "views": 79347, "answer_pids": [111586, 111592, 119819, 152890, 162146], "question_author": ""} {"qid": 575, "query": "How do people understand astromech droids?", "score": 66, "views": 73463, "answer_pids": [112187, 112188, 112200, 124620, 154005], "question_author": ""} {"qid": 576, "query": "Why did the Trade Federation blockade Naboo?", "score": 66, "views": 67231, "answer_pids": [111830, 111832, 111839, 132254, 161850, 163626], "question_author": "RaYell"} {"qid": 577, "query": "When Frodo offers her the ring, is Galadriel really tempted, or just making a point?", "score": 66, "views": 38430, "answer_pids": [146213, 146218, 146226, 146238, 175780], "question_author": "AJ Finch"} {"qid": 578, "query": "Why did it seem like only Voldemort and Snape could fly?", "score": 66, "views": 26539, "answer_pids": [110665, 110699, 112227, 112737, 127716, 166624, 166651], "question_author": ""} {"qid": 579, "query": "Why did the Jedi serve a corrupt Republic?", "score": 66, "views": 25224, "answer_pids": [164295, 164296, 164307, 165786, 165788, 165804, 165926, 169991, 172255, 177226], "question_author": "naugtur"} {"qid": 580, "query": "Why didnt the Marauders Map show Mad Eye Moody hidden in the trunk?", "score": 66, "views": 22465, "answer_pids": [173489, 173497, 173498, 180563, 193382], "question_author": ""} {"qid": 581, "query": "How did first years obtain wands during Harrys 7th year?", "score": 66, "views": 22204, "answer_pids": [181612, 181613, 181614, 181616, 181622, 181677], "question_author": "reuscam"} {"qid": 582, "query": "Why do starships fall when they blow up?", "score": 66, "views": 19476, "answer_pids": [183572, 183577, 183590, 183591, 183603, 183667], "question_author": ""} {"qid": 583, "query": "Is the humanoid appearance of most alien species in the Star Trek universe ever canonically addressed?", "score": 66, "views": 18705, "answer_pids": [112434, 112439, 112455, 112457, 112815, 120093], "question_author": "naugtur"} {"qid": 584, "query": "Why do the Rohirrim apparently find the H-word offensive?", "score": 66, "views": 18665, "answer_pids": [176315, 176316, 182316], "question_author": "Maximillian"} {"qid": 585, "query": "Why did other houses not demand this?", "score": 66, "views": 18607, "answer_pids": [197639, 197645, 197647, 197648, 197663, 197700], "question_author": "Maximillian"} {"qid": 586, "query": "Has Galactus ever been depicted as anything other than a humanoid?", "score": 66, "views": 18182, "answer_pids": [188502, 188503, 188507, 188508, 188561], "question_author": "Hamish Downer"} {"qid": 587, "query": "Was Agent Smith created by the Oracle and if so, with what purpose?", "score": 66, "views": 18173, "answer_pids": [182336], "question_author": "Goran Jovic"} {"qid": 588, "query": "Are there any known Sith from Yodas species?", "score": 66, "views": 17657, "answer_pids": [139884], "question_author": "Powerlord"} {"qid": 589, "query": "Why did Obi-Wan use his lightsaber in the Mos Eisley Cantina?", "score": 66, "views": 16641, "answer_pids": [145834, 145836, 145838, 145866, 145886, 145904, 200967], "question_author": ""} {"qid": 590, "query": "Why does Captain America need to exercise?", "score": 66, "views": 16631, "answer_pids": [146291, 146293, 146294, 146307, 146347], "question_author": "Hamish Downer"} {"qid": 591, "query": "Does the Rule of Two Serve any Actual Purpose?", "score": 66, "views": 16213, "answer_pids": [112994, 112996, 137910, 151819, 162517, 193318, 204252], "question_author": "wdypdx22"} {"qid": 592, "query": "Why didnt the Rebel Alliance (or the Empire) build YT-1300 light freighters with the same specs as the Millennium Falcon for space combat?", "score": 66, "views": 14669, "answer_pids": [190885, 190891, 190896, 190898, 190899, 190920, 190922, 191032], "question_author": ""} {"qid": 593, "query": "If being unseated by a bludger is a common occurrence, why did everyone freak out about Harry falling off of his broom?", "score": 66, "views": 13662, "answer_pids": [192325, 192328, 192329, 192335, 192344], "question_author": "matt burns"} {"qid": 594, "query": "Who is this character on the Star Wars: A New Hope poster?", "score": 66, "views": 13250, "answer_pids": [173500, 173501], "question_author": "matt burns"} {"qid": 595, "query": "Were the Moon landings faked in Interstellar?", "score": 66, "views": 12183, "answer_pids": [152728, 152729], "question_author": "Numenetics"} {"qid": 596, "query": "Why did the Dwarves build Erebor with wide hallways big enough for dragons to fit in?", "score": 66, "views": 11868, "answer_pids": [130095, 130102, 130112, 130119], "question_author": ""} {"qid": 597, "query": "Why is the stardate explicitly mentioned when creating a new Captains log?", "score": 66, "views": 11751, "answer_pids": [141532, 141533, 141534, 141544, 141589, 141632], "question_author": "chills42"} {"qid": 598, "query": "Is the Iron Throne in the show faithful to the books?", "score": 66, "views": 11733, "answer_pids": [183311, 183312], "question_author": "MatthewMartin"} {"qid": 599, "query": "Why do Klingons use cloaking devices?", "score": 66, "views": 11624, "answer_pids": [198708, 198709, 198718, 198723, 198724, 198726], "question_author": ""} {"qid": 600, "query": "Do all Far Side cartoons take place in the same universe?", "score": 66, "views": 10362, "answer_pids": [187536, 187537], "question_author": "morganpdx"} {"qid": 601, "query": "Why did OBrien start out as a transporter chief in TNG?", "score": 66, "views": 9088, "answer_pids": [136430, 136432, 136433, 136738], "question_author": ""} {"qid": 602, "query": "Were there any jokes in the Lord of the Rings?", "score": 66, "views": 7743, "answer_pids": [121435, 121471, 131383, 132965, 156576, 165611, 172531, 172543, 172572, 173364, 175965], "question_author": "morganpdx"} {"qid": 603, "query": "Firefly scenes with Han Solo in the background", "score": 66, "views": 5855, "answer_pids": [109631], "question_author": "ICodeForCoffee"} {"qid": 604, "query": "Is Daenerys Targaryen immune to fire?", "score": 65, "views": 108726, "answer_pids": [113944, 113954, 125069, 127027, 127028, 131990, 154376], "question_author": null} {"qid": 605, "query": "Do Non-Jedi (and Non-Sith) Use Lightsabers?", "score": 65, "views": 89794, "answer_pids": [114234, 114235, 114236, 114237, 126109, 138376, 146918, 155925, 156273, 158145], "question_author": "morganpdx"} {"qid": 606, "query": "How many different Enterprises were there?", "score": 65, "views": 48007, "answer_pids": [109413, 109886, 111872, 113326, 150652, 186819], "question_author": "Hamish Downer"} {"qid": 607, "query": "Why didnt the machine kill Neo after he was ejected?", "score": 65, "views": 35733, "answer_pids": [143322, 143323, 143324, 143326], "question_author": "Stu Pegg"} {"qid": 608, "query": "Why would the Terminator ask for a weapon that surely only exists in the distant future?", "score": 65, "views": 27102, "answer_pids": [111219, 111220, 118523], "question_author": ""} {"qid": 609, "query": "Why didnt Voldemort kill Snape with Avada Kedavra?", "score": 65, "views": 26871, "answer_pids": [154893, 154894], "question_author": "Saiboogu"} {"qid": 610, "query": "Why didnt Snape just kill Voldemort to buy more time while the latter was recuperating as a Horcrux?", "score": 65, "views": 25498, "answer_pids": [173996, 173997, 173998, 174003, 174016, 174046, 174083], "question_author": "kristof"} {"qid": 611, "query": "Why wasnt James Potter his own Secret Keeper?", "score": 65, "views": 24619, "answer_pids": [123074, 128703, 134531, 134886, 170777, 210585], "question_author": "Todd"} {"qid": 612, "query": "Why does Arthur Dent need to bring a towel with him in the film version of Hitchhikers Guide to the Galaxy?", "score": 65, "views": 21545, "answer_pids": [115232, 115233, 115254], "question_author": "Todd"} {"qid": 613, "query": "How did Darth Vader know the rebel base was on Hoth?", "score": 65, "views": 16798, "answer_pids": [164065, 164066, 164068, 164086, 211252], "question_author": "chills42"} {"qid": 614, "query": "What are some examples of Lovecrafts racism in his published short stories?", "score": 65, "views": 15054, "answer_pids": [202730, 202732, 202733, 202734, 202739, 202740, 202752], "question_author": ""} {"qid": 615, "query": "Why does Binary exist?", "score": 65, "views": 12834, "answer_pids": [157854, 157865, 157875, 157894, 157922, 157962, 158015, 158058, 193616], "question_author": "C. Ross"} {"qid": 616, "query": "Why did he call himself The Mule?", "score": 65, "views": 10900, "answer_pids": [178287, 178290], "question_author": "hawkeye"} {"qid": 617, "query": "Can the Delorean meet its speed requirement without moving?", "score": 65, "views": 10722, "answer_pids": [132320, 132321, 132324, 132330, 132331, 132352, 132367, 132467, 169012], "question_author": "Kevin Montrose"} {"qid": 618, "query": "Did Ducktales actually ever rewrite history?", "score": 65, "views": 6270, "answer_pids": [184034, 184113], "question_author": ""} {"qid": 619, "query": "Is Stan Lee playing the Watcher in all his cameos?", "score": 65, "views": 6005, "answer_pids": [141945, 141962, 142074, 144608, 175943], "question_author": "John"} {"qid": 620, "query": "What happened to Winston Smith at the end of George Orwell\u2019s \u201c1984?\u201d", "score": 64, "views": 299631, "answer_pids": [115465, 115466, 115468, 115490, 115500, 118116, 118121, 121424, 125275, 128122, 135361, 139262, 140495, 149048, 149691, 159588, 162069, 170130, 174345, 186224], "question_author": "John"} {"qid": 621, "query": "How well does Shadow of Mordor fit into Middle-earth canon?", "score": 64, "views": 176231, "answer_pids": [139872, 139997, 141181, 143125, 143472, 144821, 151091], "question_author": "ahockley"} {"qid": 622, "query": "Could Tom Bombadil have defeated Sauron?", "score": 64, "views": 48124, "answer_pids": [119520, 123920], "question_author": ""} {"qid": 623, "query": "How was Dumbledore able to defeat Grindelwald, since Grindelwald had the Elder Wand?", "score": 64, "views": 46330, "answer_pids": [115731, 115732, 129063, 137648, 150907, 170210, 190855], "question_author": ""} {"qid": 624, "query": "Do Lord of the Rings or The Silmarillion pass the Bechdel test?", "score": 64, "views": 27691, "answer_pids": [131346, 131348, 131355], "question_author": "TML"} {"qid": 625, "query": "Is Tony Stark an atheist?", "score": 64, "views": 26490, "answer_pids": [146566, 146596, 146598, 146665, 146772], "question_author": ""} {"qid": 626, "query": "Why are the names Sauron and Saruman so similar?", "score": 64, "views": 24229, "answer_pids": [145476, 145479], "question_author": ""} {"qid": 627, "query": "Are there any bad elves in the Lord of the Rings universe?", "score": 64, "views": 23487, "answer_pids": [139166, 139168, 139169, 139173, 139175], "question_author": ""} {"qid": 628, "query": "Why do the Jedi say May the Force be with you?", "score": 64, "views": 23299, "answer_pids": [117607, 117608, 117629, 117632, 117772], "question_author": ""} {"qid": 629, "query": "Why is Robin Hood French in Shrek?", "score": 64, "views": 21839, "answer_pids": [200826, 200836], "question_author": ""} {"qid": 630, "query": "Why did Voldemort kill Lily Potter instead of stunning her?", "score": 64, "views": 21695, "answer_pids": [174369, 174371, 174407, 174426, 174439], "question_author": ""} {"qid": 631, "query": "Why dont elves have more babies?", "score": 64, "views": 20517, "answer_pids": [148022, 148023, 189293, 211477], "question_author": "Sean Moubry"} {"qid": 632, "query": "Asimovs story where a mans speech contains no information", "score": 64, "views": 18155, "answer_pids": [200834], "question_author": "jfklein13"} {"qid": 633, "query": "Who gets Muggle-born students onto Platform 9 3/4?", "score": 64, "views": 17958, "answer_pids": [114446, 114447, 114448, 118374, 123792, 123809, 125477, 127250], "question_author": "reuscam"} {"qid": 634, "query": "Do lightsaber blades/blaster bolts cast shadows?", "score": 64, "views": 15995, "answer_pids": [188154, 188158, 188159, 188180], "question_author": ""} {"qid": 635, "query": "What effect would the One Ring have on other people?", "score": 64, "views": 15819, "answer_pids": [110934, 110935, 111981, 111983], "question_author": ""} {"qid": 636, "query": "Why does Sauron not permit his followers to use his name?", "score": 64, "views": 15326, "answer_pids": [199826, 199832, 199836, 199838, 199878], "question_author": ""} {"qid": 637, "query": "If the galaxy far far away is really so big, how come it seems so small in the movies?", "score": 64, "views": 14992, "answer_pids": [181125, 181126, 181133, 181137, 181138, 181161, 181168], "question_author": "reuscam"} {"qid": 638, "query": "Which Asimov story has malevolent 3-law AI?", "score": 64, "views": 14638, "answer_pids": [168344, 168347, 168351, 168353, 168359, 168362, 168368, 168373, 168374, 168375], "question_author": "Sean Moubry"} {"qid": 639, "query": "Do Morpheus and his crew kill potential Ones?", "score": 64, "views": 14500, "answer_pids": [170012, 170016], "question_author": ""} {"qid": 640, "query": "How large was Khazad-d\u00fbm?", "score": 64, "views": 14357, "answer_pids": [176530, 176549], "question_author": ""} {"qid": 641, "query": "Is there any relevance to Thor getting his hair cut other than comedic value?", "score": 64, "views": 14202, "answer_pids": [195469, 195477], "question_author": "Sean Moubry"} {"qid": 642, "query": "Why do the Agents bother trying to get the Zion mainframe codes?", "score": 64, "views": 14026, "answer_pids": [143292, 143293, 143325, 143369], "question_author": "Bossykena"} {"qid": 643, "query": "How does one ride a broom in Harry Potter films?", "score": 64, "views": 11496, "answer_pids": [188544, 188582], "question_author": "Jared Updike"} {"qid": 644, "query": "Were there any democracies in Middle-earth?", "score": 64, "views": 9838, "answer_pids": [141759, 141760, 141767, 141875], "question_author": ""} {"qid": 645, "query": "Why was Mal so quick to drop Bester in favour of Kaylee?", "score": 64, "views": 9788, "answer_pids": [198524, 198536], "question_author": ""} {"qid": 646, "query": "Why was there a pit in the Emperors chamber on Death Star?", "score": 64, "views": 9738, "answer_pids": [134114, 134118, 134125, 134156, 161515], "question_author": ""} {"qid": 647, "query": "Why was the Balrog beneath Moria", "score": 64, "views": 9723, "answer_pids": [208649], "question_author": ""} {"qid": 648, "query": "Short story read ~25 year ago where changing persons name prevents a war", "score": 64, "views": 8011, "answer_pids": [193975], "question_author": "Michael"} {"qid": 649, "query": "Trying to identify I will measure the dust story", "score": 64, "views": 3681, "answer_pids": [110495, 116324, 120451], "question_author": ""} {"qid": 650, "query": "A man takes riskier and riskier jobs (involving a transportation ring) for more money", "score": 64, "views": 2931, "answer_pids": [176262], "question_author": "thelsdj"} {"qid": 651, "query": "Why does Tywin correct Arya on usage of mLord and my Lord?", "score": 63, "views": 96150, "answer_pids": [117930, 118049, 121563, 122005, 125588], "question_author": "MatthewMartin"} {"qid": 652, "query": "Why is Lando wearing Hans clothes?", "score": 63, "views": 67618, "answer_pids": [139624, 173842, 186163], "question_author": ""} {"qid": 653, "query": "Asimov - Start with Foundation or Robots", "score": 63, "views": 58136, "answer_pids": [109597, 109598, 109600, 109604, 112171, 116811, 119179, 173821, 205892], "question_author": ""} {"qid": 654, "query": "If Galadriel had claimed the One Ring, would Sauron have become her servant, or would have Galadriel become his servant?", "score": 63, "views": 55232, "answer_pids": [115504, 115507, 115510, 118023, 120903, 124634, 140494, 140995], "question_author": "Goodbye Stack Exchange"} {"qid": 655, "query": "Why did the Borg name Locutus?", "score": 63, "views": 49964, "answer_pids": [111846, 122913, 124261], "question_author": "Bossykena"} {"qid": 656, "query": "How can every Cubone be wearing its dead mothers skull?", "score": 63, "views": 42427, "answer_pids": [176477, 176495, 176529], "question_author": "Todd"} {"qid": 657, "query": "Were all titans as powerful as Thanos?", "score": 63, "views": 38635, "answer_pids": [189774, 189780, 193037], "question_author": ""} {"qid": 658, "query": "Why did Tyrion visit the Wall?", "score": 63, "views": 37757, "answer_pids": [110846, 110849, 110852, 111443, 148834, 153652], "question_author": "Todd"} {"qid": 659, "query": "Why are the colors reversed between some blasters and lightsabers of the Republic/Rebels and Empire?", "score": 63, "views": 37094, "answer_pids": [113641, 113642], "question_author": ""} {"qid": 660, "query": "Which button activates Skywalkers lightsaber, and why is it inconsistent?", "score": 63, "views": 35865, "answer_pids": [157387, 157398, 159863, 173161, 178814], "question_author": ""} {"qid": 661, "query": "Is Quidditch an inherently flawed sport?", "score": 63, "views": 33301, "answer_pids": [136912, 136915, 136976, 148130, 169748, 170899, 170909, 170923], "question_author": "F'x"} {"qid": 662, "query": "Why did Dumbledore hire Lockhart?", "score": 63, "views": 32687, "answer_pids": [128582, 128595, 136791], "question_author": "Owen"} {"qid": 663, "query": "Was the Millennium Falcon a one-off or was it mass produced?", "score": 63, "views": 28826, "answer_pids": [159153, 159154, 159160, 159202, 159227, 159257, 159268, 181207], "question_author": ""} {"qid": 664, "query": "The use of Darth as Vaders name, not title, in Star Wars", "score": 63, "views": 26221, "answer_pids": [126494, 126500, 132633, 144609, 158657, 171525, 183608, 193877], "question_author": "Karel"} {"qid": 665, "query": "Kessel Run in 12 parsecs: screenplay error, or part of the movie?", "score": 63, "views": 26028, "answer_pids": [119662, 119731, 126955, 134937, 135919, 143958, 149430, 149433, 152110, 158664, 159614, 171234, 188790], "question_author": ""} {"qid": 666, "query": "When did Darth Vader get his red lightsaber?", "score": 63, "views": 16571, "answer_pids": [172924, 172929, 172937, 172998, 173005, 207760], "question_author": null} {"qid": 667, "query": "What flaw in the Matrix was Neo supposed to correct?", "score": 63, "views": 16505, "answer_pids": [148924, 148925, 148926, 149004], "question_author": "Karel"} {"qid": 668, "query": "How did Hagrid and Dumbledore know the Potters had been killed?", "score": 63, "views": 16362, "answer_pids": [115001, 115002, 115012, 115115, 115158, 136490, 166662, 169054], "question_author": "Numenetics"} {"qid": 669, "query": "Why is the Star Wars universe not a post-scarcity economy?", "score": 63, "views": 15893, "answer_pids": [162586, 162589, 162598, 162601, 162603, 162605, 162610, 162624, 162634, 162637, 162638, 162644, 162766], "question_author": "LittleBobbyTables - Au Revoir"} {"qid": 670, "query": "Is Palpatines identity a spoiler?", "score": 63, "views": 14926, "answer_pids": [188955, 188956, 188957, 188965, 188971, 188997, 189021], "question_author": "Karel"} {"qid": 671, "query": "Why was Marty present in his jump to 2015 in Back to the Future 2?", "score": 63, "views": 14846, "answer_pids": [113313, 113314, 113315, 114005, 114019, 115795, 133240, 166815, 168257, 168258, 194523], "question_author": "GWLlosa"} {"qid": 672, "query": "Did Saruman know about the Ents?", "score": 63, "views": 14004, "answer_pids": [158883, 158884], "question_author": "RMorrisey"} {"qid": 673, "query": "Soviet statistician deduces the existence of parasites feeding on the human race", "score": 63, "views": 13534, "answer_pids": [201082], "question_author": "Shaihi"} {"qid": 674, "query": "Why wouldnt Luke use his lightsaber to cut the feet off the Imperial Walker?", "score": 63, "views": 12995, "answer_pids": [139361, 139371, 139393], "question_author": ""} {"qid": 675, "query": "What characters in Game of Thrones have been recast?", "score": 63, "views": 11669, "answer_pids": [186942, 186943, 186957], "question_author": "Martha F."} {"qid": 676, "query": "Has any Star Trek character appeared in all three division uniform colors?", "score": 63, "views": 11456, "answer_pids": [151274, 151275, 151279, 151362, 151421, 175271], "question_author": ""} {"qid": 677, "query": "Why Was Obi Wan So Eager to Train Luke as a Jedi?", "score": 63, "views": 11051, "answer_pids": [112307, 112308, 112309, 123771, 137020, 142288], "question_author": "chills42"} {"qid": 678, "query": "How could rebels trust R2-D2?", "score": 63, "views": 10992, "answer_pids": [176620, 176621, 176628, 176629, 176637, 176689], "question_author": ""} {"qid": 679, "query": "Does Ron Weasley successfully cast a spell on camera in books 2 through 6?", "score": 63, "views": 10933, "answer_pids": [179812, 179843], "question_author": ""} {"qid": 680, "query": "Are there any more races in Middle-earth?", "score": 63, "views": 10322, "answer_pids": [110542, 110546, 111501, 111514, 122140, 143971, 147853], "question_author": ""} {"qid": 681, "query": "Was anyone other than Edwards actually being considered for admission to the Men in Black?", "score": 63, "views": 9750, "answer_pids": [183137, 183139, 183140, 183141], "question_author": "Joe Zack"} {"qid": 682, "query": "Did Luke accomplish anything by going to Cloud City?", "score": 63, "views": 9730, "answer_pids": [111914, 111915, 111917, 111919, 111925, 129693, 133220, 133227, 203759], "question_author": ""} {"qid": 683, "query": "Has the phrase Star Trek ever been used in Star Trek?", "score": 63, "views": 9590, "answer_pids": [151966, 151974], "question_author": "Matt Bishop"} {"qid": 684, "query": "How do we know that the First Doctor was indeed the first?", "score": 63, "views": 8934, "answer_pids": [138106, 138109, 138154, 138295, 141523], "question_author": ""} {"qid": 685, "query": "Toilets in the Star Wars galaxy", "score": 63, "views": 7530, "answer_pids": [157139, 157294, 175524, 208559], "question_author": ""} {"qid": 686, "query": "How does one leave platform nine and three-quarters?", "score": 63, "views": 6771, "answer_pids": [155963, 155964, 155965], "question_author": "paercebal"} {"qid": 687, "query": "When did pure-historical Doctor Who go out of fashion?", "score": 63, "views": 5056, "answer_pids": [158040], "question_author": "Mike Bohlmann"} {"qid": 688, "query": "What story does C.S. Lewis acknowledge in the preface to The Great Divorce?", "score": 63, "views": 4543, "answer_pids": [191408, 191420], "question_author": ""} {"qid": 689, "query": "Was Gimli the last of his race?", "score": 62, "views": 161584, "answer_pids": [112423, 112424, 113264, 127202, 127611, 128687], "question_author": ""} {"qid": 690, "query": "Why Couldnt a Time Turner Have been Used to Stop Voldemort?", "score": 62, "views": 100416, "answer_pids": [115370, 115372, 116727, 116882, 116915, 126156, 127736, 128968, 129414, 129934, 129947, 130613, 146481, 149836, 158582], "question_author": "Tobiasopdenbrouw"} {"qid": 691, "query": "Why didnt Sauron find Bilbo when he put on the ring like Frodo?", "score": 62, "views": 59774, "answer_pids": [131718, 131730, 156776, 171986, 206456], "question_author": "Tobiasopdenbrouw"} {"qid": 692, "query": "Why did Voldemort need to drink Unicorn blood?", "score": 62, "views": 53822, "answer_pids": [138621, 140780, 140782, 180493], "question_author": "Tobiasopdenbrouw"} {"qid": 693, "query": "Is James Doohan\u2019s missing finger ever noticeable in Star Trek?", "score": 62, "views": 46890, "answer_pids": [117764, 117775, 136504, 141081, 172128, 206802], "question_author": "Tobiasopdenbrouw"} {"qid": 694, "query": "Why didnt Luke become disfigured when he was electrocuted by the Emperor?", "score": 62, "views": 43557, "answer_pids": [112489, 112490, 112491, 112495, 112518, 112587, 119391, 125660], "question_author": "matt burns"} {"qid": 695, "query": "Why does Hermione say she hadnt used a memory charm before?", "score": 62, "views": 39365, "answer_pids": [111405, 111409, 117981], "question_author": ""} {"qid": 696, "query": "Why doesnt either side in the Wizarding War use Felix Felicis?", "score": 62, "views": 32303, "answer_pids": [121196, 128126, 128873, 163456, 189077, 189080], "question_author": ""} {"qid": 697, "query": "Why does Tony Stark not like being handed things?", "score": 62, "views": 26402, "answer_pids": [206320, 206331], "question_author": ""} {"qid": 698, "query": "Why didnt Chewbacca get a medal after the battle of Yavin?", "score": 62, "views": 26018, "answer_pids": [112354, 112358, 112364, 112382, 157465, 166570, 174483], "question_author": "lavonardo"} {"qid": 699, "query": "Why did Neo even have to fight?", "score": 62, "views": 24716, "answer_pids": [179066, 179073, 179074, 179103], "question_author": ""} {"qid": 700, "query": "Was the thermal exhaust port on the Death Star really a design flaw?", "score": 62, "views": 24031, "answer_pids": [110643, 110644, 110645, 110646, 110650, 110652, 111869, 112196, 113126, 118379, 171349], "question_author": "WireGuy"} {"qid": 701, "query": "Who is the eponymous Jedi in Return of the Jedi?", "score": 62, "views": 24024, "answer_pids": [137105, 137118, 137126, 137177, 140048, 157698, 180161], "question_author": "John Virgolino"} {"qid": 702, "query": "Were parts of Harry Potter inspired by James and the Giant Peach?", "score": 62, "views": 23257, "answer_pids": [145524, 145526, 189560, 201381], "question_author": ""} {"qid": 703, "query": "Can Superman get drunk?", "score": 62, "views": 22422, "answer_pids": [175106, 175128, 175166], "question_author": "Tobiasopdenbrouw"} {"qid": 704, "query": "Why didnt Hermione use the Time-Turner to sleep more?", "score": 62, "views": 21297, "answer_pids": [137563, 137564, 137565, 137567, 137568, 137591, 137643], "question_author": ""} {"qid": 705, "query": "How does Malfoy know about helicopters?", "score": 62, "views": 20173, "answer_pids": [180891, 180893, 180900], "question_author": ""} {"qid": 706, "query": "Why did R2-D2 not fly in Return of the Jedi?", "score": 62, "views": 19988, "answer_pids": [110177, 110178, 110182, 110183, 120947, 123729, 135678, 162134], "question_author": "CodeToGlory"} {"qid": 707, "query": "Why are there no Imperial KX-series Security Droids in the original trilogy?", "score": 62, "views": 18224, "answer_pids": [172013, 172014, 172018, 172019, 172049, 172062, 172147], "question_author": "Stu Pegg"} {"qid": 708, "query": "In Star Trek TNG, why does everyone have plants at the head of their beds?", "score": 62, "views": 17710, "answer_pids": [140584, 140587, 140601, 157352], "question_author": ""} {"qid": 709, "query": "Why did the villain in the first Men in Black movie care about Earths Cockroaches?", "score": 62, "views": 17260, "answer_pids": [195311, 195312, 195325, 195328, 195329], "question_author": ""} {"qid": 710, "query": "Why doesnt Harry live with the Weasleys?", "score": 62, "views": 17186, "answer_pids": [153083, 153190], "question_author": "CodeToGlory"} {"qid": 711, "query": "Why did the best potions master have greasy hair?", "score": 62, "views": 16794, "answer_pids": [169874, 169880, 169884, 169885, 169887, 169892, 169899], "question_author": "Albort"} {"qid": 712, "query": "What exactly is a Hadouken?", "score": 62, "views": 16409, "answer_pids": [198994], "question_author": "Numenetics"} {"qid": 713, "query": "Why isnt everyone flabbergasted about Brans gift?", "score": 62, "views": 16380, "answer_pids": [196653, 196660, 196679, 196710], "question_author": "Karel"} {"qid": 714, "query": "Why do X-wings change wing configuration?", "score": 62, "views": 16346, "answer_pids": [112055, 112056, 112061, 114910, 117440, 156651], "question_author": "Stefano Borini"} {"qid": 715, "query": "What was the deal Lando made with Darth Vader?", "score": 62, "views": 15072, "answer_pids": [162242, 162277, 162280], "question_author": ""} {"qid": 716, "query": "How are Stormtroopers uniquely identified in the field?", "score": 62, "views": 14877, "answer_pids": [157670, 157675, 157713], "question_author": ""} {"qid": 717, "query": "Why does Hermione love Arithmancy so much?", "score": 62, "views": 13770, "answer_pids": [174608, 174610, 174611, 174621, 174639], "question_author": ""} {"qid": 718, "query": "Whats the pyramid joke?", "score": 62, "views": 12645, "answer_pids": [190556, 190557, 190558, 190604], "question_author": ""} {"qid": 719, "query": "After Gandalf kills the Balrog of Moria, do any more Balrogs remain in Middle-earth?", "score": 62, "views": 11640, "answer_pids": [120780, 120782, 205069], "question_author": "gabr"} {"qid": 720, "query": "Is Sam Gamgee an orphan?", "score": 62, "views": 11564, "answer_pids": [184445], "question_author": ""} {"qid": 721, "query": "How was Lukes prosthetic hand in Episode V filmed?", "score": 62, "views": 9500, "answer_pids": [199081, 199085], "question_author": "JSB\u0571\u0578\u0563\u0579"} {"qid": 722, "query": "Where are Draxs nipples?", "score": 62, "views": 9431, "answer_pids": [189090, 189101], "question_author": "C. Ross"} {"qid": 723, "query": "What is the Positronic brain coined by Asimov?", "score": 62, "views": 9317, "answer_pids": [109520, 109521, 109526, 110337, 110456, 114170, 132521], "question_author": "Espo"} {"qid": 724, "query": "What do they do with all the plates and cups in Star Trek?", "score": 62, "views": 8952, "answer_pids": [129118, 129119, 133997], "question_author": "Swapnonil Mukherjee"} {"qid": 725, "query": "What made R2-D2 reactivate in \u201cThe Force Awakens?\u201d", "score": 62, "views": 8572, "answer_pids": [156204, 156216, 156255, 156261, 156449], "question_author": "deadprogrammer"} {"qid": 726, "query": "Is C-3PO a slave?", "score": 62, "views": 5761, "answer_pids": [111006, 111007, 111009], "question_author": ""} {"qid": 727, "query": "Does Red Skull die at the end of Captain America?", "score": 61, "views": 157013, "answer_pids": [112232, 122141, 188036, 197494], "question_author": ""} {"qid": 728, "query": "Why isnt Obi-Wans ghost young?", "score": 61, "views": 86447, "answer_pids": [112528, 112529, 112532, 112536, 112538, 112856, 112961, 113469], "question_author": ""} {"qid": 729, "query": "Why did Deanna Troi keep wearing a Starfleet uniform after Chain of Command?", "score": 61, "views": 74578, "answer_pids": [109975, 109979, 109981, 134057, 144635, 154619, 178456], "question_author": ""} {"qid": 730, "query": "When did the red shirt careers change to yellow shirts in Star Trek and why?", "score": 61, "views": 59992, "answer_pids": [111234, 111255, 144619], "question_author": "Powerlord"} {"qid": 731, "query": "Why didnt Aragorn keep going with the army of the dead?", "score": 61, "views": 50607, "answer_pids": [125503, 125505, 125519, 128985, 131314, 145853], "question_author": "eswald"} {"qid": 732, "query": "Who Erased Kamino from the Jedi-Archives?", "score": 61, "views": 41637, "answer_pids": [110573, 110680, 111897, 114771, 125204, 126900, 159535], "question_author": ""} {"qid": 733, "query": "Why did the Rebels have to fly along the Death Star trench?", "score": 61, "views": 30700, "answer_pids": [115476, 115477, 115478, 115479, 115480, 118163, 123427, 124842, 124849, 131993, 131995, 138294, 155860, 157170], "question_author": "CodeToGlory"} {"qid": 734, "query": "Do students wear muggle clothing while not in classes at Hogwarts (like they do in the films)?", "score": 61, "views": 25337, "answer_pids": [169894, 169896, 169915], "question_author": ""} {"qid": 735, "query": "In Dune, was the Golden Path really necessary?", "score": 61, "views": 17790, "answer_pids": [109714, 111702, 131833], "question_author": ""} {"qid": 736, "query": "Does the intro sequence to the Game of Thrones TV series have any meaning?", "score": 61, "views": 17301, "answer_pids": [110841, 110844, 110851, 110868, 110883, 111096, 115093, 165792], "question_author": "Maximillian"} {"qid": 737, "query": "Why didnt Gandalf know what the Palantir was?", "score": 61, "views": 16957, "answer_pids": [147230, 147231, 147238, 147360, 175626], "question_author": ""} {"qid": 738, "query": "Why is The Wall still manned?", "score": 61, "views": 16940, "answer_pids": [180248, 180249, 180262, 180376], "question_author": "Maximillian"} {"qid": 739, "query": "Why dont wizards rob muggle banks to become rich?", "score": 61, "views": 16919, "answer_pids": [181027, 181028, 181029, 181030, 181032, 181051], "question_author": "Bryant"} {"qid": 740, "query": "Why didnt Yoda have any children to take care of him during his last days given that he lived up to 900 years old?", "score": 61, "views": 16896, "answer_pids": [156196, 156197, 156199, 156209, 156222, 156260, 156266], "question_author": ""} {"qid": 741, "query": "Why did Harry tell Snapes secret to everyone?", "score": 61, "views": 16222, "answer_pids": [165126, 165177, 165225, 165240], "question_author": "Matt Dillard"} {"qid": 742, "query": "Why does Yoda mourn the Jedi after Order 66 is executed?", "score": 61, "views": 14899, "answer_pids": [140244, 140248, 140249, 140250, 140329], "question_author": "ahockley"} {"qid": 743, "query": "Whos older: Treebeard or Tom Bombadil?", "score": 61, "views": 14859, "answer_pids": [114828, 114831, 141751, 145342], "question_author": "Jorge Castro"} {"qid": 744, "query": "Why was the battle set up *outside* Winterfell?", "score": 61, "views": 14490, "answer_pids": [196942, 196946, 196949, 196961, 196997], "question_author": ""} {"qid": 745, "query": "Why does Quidditch allow faster brooms?", "score": 61, "views": 12897, "answer_pids": [145218, 145221, 145256, 145269], "question_author": "ahockley"} {"qid": 746, "query": "Was there a Biggs Darklighter scene on Tatooine in A New Hope?", "score": 61, "views": 12735, "answer_pids": [202874, 202884, 202886, 202905, 202907, 202909, 210883], "question_author": ""} {"qid": 747, "query": "What was the value of the Shire and everything in it?", "score": 61, "views": 12458, "answer_pids": [173743, 173749, 173750, 173751, 173758, 173831, 173849], "question_author": ""} {"qid": 748, "query": "Who are the Avengers avenging?", "score": 61, "views": 12311, "answer_pids": [116875, 117646, 195727], "question_author": "ahockley"} {"qid": 749, "query": "A Book where an Animal is advertising itself to be eaten", "score": 61, "views": 11853, "answer_pids": [170707, 170708, 170709], "question_author": "chills42"} {"qid": 750, "query": "Why does Seven of Nine pronounce Futile differently than Locutus?", "score": 61, "views": 11377, "answer_pids": [160139, 160161, 160190], "question_author": ""} {"qid": 751, "query": "Why would the elves help destroy the One Ring?", "score": 61, "views": 11020, "answer_pids": [143831, 143833, 143840, 143846, 145644, 150063], "question_author": ""} {"qid": 752, "query": "Why does Data have to use a computer in Star Trek?", "score": 61, "views": 10504, "answer_pids": [137838, 137847, 137896, 142559, 163678], "question_author": "RS Conley"} {"qid": 753, "query": "What is the point of evasive maneuvers in Star Trek?", "score": 61, "views": 9578, "answer_pids": [117938, 117939, 117940, 117946, 117959, 188276, 210148], "question_author": "tttppp"} {"qid": 754, "query": "What exactly about Aunt Petunia is More than meets the eye?", "score": 61, "views": 9150, "answer_pids": [159719, 159726], "question_author": ""} {"qid": 755, "query": "A cynical demon writing annotations into the book for puny human readers", "score": 61, "views": 8507, "answer_pids": [180940, 180956], "question_author": "Alan"} {"qid": 756, "query": "Looking for a really old novel where the people are geometric figures", "score": 61, "views": 5953, "answer_pids": [143759], "question_author": "Tobiasopdenbrouw"} {"qid": 757, "query": "In which order should I start watching Star Trek?", "score": 60, "views": 378804, "answer_pids": [127133, 127136, 127137, 127139, 127145, 127153, 127162, 128327], "question_author": ""} {"qid": 758, "query": "In what order should I watch the various Stargate TV Series?", "score": 60, "views": 159972, "answer_pids": [111484, 111650], "question_author": "Mag Roader"} {"qid": 759, "query": "Whats the origin of Picards signature phrase, make it so?", "score": 60, "views": 89363, "answer_pids": [114130, 121359, 123893, 128605, 140103, 140106, 159397, 160891, 178962, 198272], "question_author": "Jonathon Watney"} {"qid": 760, "query": "Are the Weeping Angels Timelords", "score": 60, "views": 86184, "answer_pids": [115933, 125768, 126703, 127417, 128064], "question_author": ""} {"qid": 761, "query": "What did Madam Hooch do all day?", "score": 60, "views": 47560, "answer_pids": [162775, 162777], "question_author": "Alan"} {"qid": 762, "query": "How did Voyager replace its photon torpedoes?", "score": 60, "views": 41517, "answer_pids": [121457, 121458], "question_author": "Jonathon Watney"} {"qid": 763, "query": "How do we know they were called Ewoks?", "score": 60, "views": 36654, "answer_pids": [111948, 111949, 132505, 132525], "question_author": ""} {"qid": 764, "query": "Can the Force be used to fly?", "score": 60, "views": 35565, "answer_pids": [114528, 155552, 160638, 161770, 161772, 171681, 173616, 184332, 184345], "question_author": ""} {"qid": 765, "query": "How did Dumbledore block the Avada Kedavra spell?", "score": 60, "views": 26188, "answer_pids": [159832, 159839, 159850], "question_author": "Mag Roader"} {"qid": 766, "query": "Is the Han/Leia vs Luke/Yoda timeline in The Empire Strikes Back broken?", "score": 60, "views": 19312, "answer_pids": [110168, 110169, 111976, 119163, 125202, 139684, 160817, 165747, 172076], "question_author": ""} {"qid": 767, "query": "Who was the student that previously brewed a potion of sufficient quality to claim the Felix Felicis?", "score": 60, "views": 18084, "answer_pids": [194101, 194111], "question_author": ""} {"qid": 768, "query": "How did Jabba become such a powerful crime lord?", "score": 60, "views": 17665, "answer_pids": [130126, 130132, 130135, 184363], "question_author": ""} {"qid": 769, "query": "How does Elsa resolve the practical problems of living in an ice palace?", "score": 60, "views": 16985, "answer_pids": [138186, 138201, 138217, 138223, 138241], "question_author": ""} {"qid": 770, "query": "Has anyone else in the Star Wars universe ever run out of fuel?", "score": 60, "views": 16517, "answer_pids": [185045, 185047, 201699], "question_author": ""} {"qid": 771, "query": "Why was Darth Vader given a downgraded version of General Grievouss mechanics?", "score": 60, "views": 14811, "answer_pids": [112697, 119477, 129585], "question_author": ""} {"qid": 772, "query": "Could the Sorting Hat Have Told Dumbledore That Harry Was a Horcrux?", "score": 60, "views": 14292, "answer_pids": [112950, 112951, 112952, 113567, 114809, 125133], "question_author": ""} {"qid": 773, "query": "Why do people say that Peter Jackson can not make LOTR/Hobbit sequels?", "score": 60, "views": 13931, "answer_pids": [130604, 130607, 148123], "question_author": "Grant Palin"} {"qid": 774, "query": "Why didnt Voldemort know what Grindelwald looked like?", "score": 60, "views": 12477, "answer_pids": [195845, 195847, 195848], "question_author": "Michiel de Mare"} {"qid": 775, "query": "If the Navi instantly recognize the Avatars as Sky People, what is the point of having them?", "score": 60, "views": 12445, "answer_pids": [176268, 176277, 176303], "question_author": "Michiel de Mare"} {"qid": 776, "query": "How much does Commander Data weigh?", "score": 60, "views": 12381, "answer_pids": [199767, 199790], "question_author": ""} {"qid": 777, "query": "The Moriarty Sentience Problem - A.K.A. Why isnt the Enterprise computer considered sentient?", "score": 60, "views": 11130, "answer_pids": [141045, 141051, 141052, 141053, 141058, 141060, 141068, 141079, 141096, 141124, 141189], "question_author": "Tony Meyer"} {"qid": 778, "query": "Whats in Naboos core, liquid water or plasma?", "score": 60, "views": 10870, "answer_pids": [169799, 169818], "question_author": "ahockley"} {"qid": 779, "query": "Was Jumanji intended to be a co-op game?", "score": 60, "views": 10762, "answer_pids": [201948, 201991, 201993, 202008], "question_author": "Stefano Borini"} {"qid": 780, "query": "What did Aragorn mean when he warned Gandalf about entering Moria?", "score": 60, "views": 10228, "answer_pids": [194183], "question_author": ""} {"qid": 781, "query": "Was Picard speaking French and being translated the entire time?", "score": 60, "views": 9189, "answer_pids": [125645, 125647, 125717], "question_author": ""} {"qid": 782, "query": "Did Data murder Lore?", "score": 60, "views": 8816, "answer_pids": [153162, 153163, 153165, 153185, 153194], "question_author": "Lazer"} {"qid": 783, "query": "How much would a ticket to Jurassic World cost?", "score": 60, "views": 8472, "answer_pids": [149711, 149712, 149794, 149917], "question_author": ""} {"qid": 784, "query": "Did the writers of Wonder Woman have some kind of fetish for bondage, in the early comics?", "score": 60, "views": 8422, "answer_pids": [174595], "question_author": "Drew Hoskins"} {"qid": 785, "query": "Did Sauron survive?", "score": 60, "views": 7932, "answer_pids": [115428, 144467, 211877], "question_author": ""} {"qid": 786, "query": "Why not continue to prevent murders?", "score": 60, "views": 7579, "answer_pids": [164044, 164077, 164271], "question_author": ""} {"qid": 787, "query": "Do you remember the memories that youve placed in a pensieve?", "score": 60, "views": 6847, "answer_pids": [161532, 161533, 178440, 208335], "question_author": ""} {"qid": 788, "query": "Is it ever explained where Elsa got her powers?", "score": 59, "views": 128587, "answer_pids": [135429, 135430, 135441, 140304, 201567], "question_author": "Lazer"} {"qid": 789, "query": "Why did Frodo start his adventure 17 years after he inherited the One Ring?", "score": 59, "views": 61096, "answer_pids": [137607, 137608, 137613, 137614, 137615], "question_author": "Tobiasopdenbrouw"} {"qid": 790, "query": "Is Magnetos power magnetism or can he manipulate all metals?", "score": 59, "views": 49123, "answer_pids": [111027, 111028, 111545, 111546, 121420, 135310], "question_author": "Lazer"} {"qid": 791, "query": "How did the black liquid work in Prometheus?", "score": 59, "views": 48162, "answer_pids": [118542, 118626, 118851, 118860, 119488, 121851, 126583, 143753], "question_author": "Tobiasopdenbrouw"} {"qid": 792, "query": "What did Smaug eat for 60 years?", "score": 59, "views": 33338, "answer_pids": [130130, 130136, 130144, 136941], "question_author": "Tobiasopdenbrouw"} {"qid": 793, "query": "Who is the most powerful user of the Force?", "score": 59, "views": 32958, "answer_pids": [110902, 112677, 115049, 119805, 129303, 150278, 151226], "question_author": "Chris Persichetti"} {"qid": 794, "query": "Do the Q fear the Borg?", "score": 59, "views": 32788, "answer_pids": [145989, 145995, 146018, 146023, 165590], "question_author": ""} {"qid": 795, "query": "Why does Hermione dislike Professor Trelawney from the start?", "score": 59, "views": 22637, "answer_pids": [171791, 171792, 171797, 171809], "question_author": "Nick Larsen"} {"qid": 796, "query": "Was Gollum always a hobbit?", "score": 59, "views": 22493, "answer_pids": [121572, 121574, 121586, 123108, 194908, 202911], "question_author": ""} {"qid": 797, "query": "Whats the name of the galaxy far, far away?", "score": 59, "views": 21844, "answer_pids": [126161], "question_author": "Nick Larsen"} {"qid": 798, "query": "Why does Yoda speak the way he does?", "score": 59, "views": 21673, "answer_pids": [114838, 121423], "question_author": "Tobiasopdenbrouw"} {"qid": 799, "query": "Why didn\u2019t they just take the ring to Valinor?", "score": 59, "views": 19886, "answer_pids": [137374, 137377, 137413, 137446, 141017], "question_author": "Tobiasopdenbrouw"} {"qid": 800, "query": "Treebeard: I always like going South, somehow it feels like going downhill. - Is it really?", "score": 59, "views": 19466, "answer_pids": [186887, 186888, 186902, 186925], "question_author": "BlueRaja - Danny Pflughoeft"} {"qid": 801, "query": "Why does Gandalf show such reverence for Galadriel and Elrond?", "score": 59, "views": 18129, "answer_pids": [152627, 152642, 152645, 152655], "question_author": "Tobiasopdenbrouw"} {"qid": 802, "query": "Why do consoles explode in Star Trek?", "score": 59, "views": 16921, "answer_pids": [112922, 112929, 122816, 149741, 195455], "question_author": ""} {"qid": 803, "query": "Why did Captain Needa apologize to Darth Vader?", "score": 59, "views": 16452, "answer_pids": [171181, 171182, 171189, 171212], "question_author": "Lazer"} {"qid": 804, "query": "Why does everyone assume that the Architect was telling the truth about there being previous Ones?", "score": 59, "views": 15335, "answer_pids": [170682, 170683, 170690, 170706], "question_author": "ahockley"} {"qid": 805, "query": "Was Sauron aware of what Gandalf was?", "score": 59, "views": 14005, "answer_pids": [150231, 150257, 150274, 150279], "question_author": ""} {"qid": 806, "query": "Are Hogwarts teachers allowed to hit students?", "score": 59, "views": 13419, "answer_pids": [174704, 174705, 174708, 204429], "question_author": "Jonny Blaze"} {"qid": 807, "query": "In Star Wars: A New Hope, why was one officer dressed in white?", "score": 59, "views": 13259, "answer_pids": [163546, 163547, 163596], "question_author": "WireGuy"} {"qid": 808, "query": "Why was Gandalf so concerned for Gollums well-being?", "score": 59, "views": 13064, "answer_pids": [112663, 113245, 113247, 116191, 119478, 119644, 136841], "question_author": "Buddy Lindsey"} {"qid": 809, "query": "Did Vader ever learn that Sidious controlled both sides in the Clone Wars?", "score": 59, "views": 12819, "answer_pids": [155999, 156000, 156001], "question_author": "Reinstate Monica - Goodbye SE"} {"qid": 810, "query": "How does Asimovs second law deal with contradictory orders from different people?", "score": 59, "views": 11674, "answer_pids": [199328, 199329, 199331, 199332, 199336, 199348, 199361], "question_author": ""} {"qid": 811, "query": "Why didnt Harry try harder to learn magic?", "score": 59, "views": 11529, "answer_pids": [115997, 122566, 131209, 135686, 189167], "question_author": ""} {"qid": 812, "query": "What are the revisions in the (Revised Edition) of The Hobbit?", "score": 59, "views": 11422, "answer_pids": [117301, 117302], "question_author": ""} {"qid": 813, "query": "Roughly how large is the population of Sandworms on Dune?", "score": 59, "views": 11380, "answer_pids": [203147, 203151], "question_author": ""} {"qid": 814, "query": "What was the first story to be set in the future?", "score": 59, "views": 9492, "answer_pids": [159553, 159554, 159563, 205867], "question_author": ""} {"qid": 815, "query": "Was the original Star Wars film originally called The Star Wars?", "score": 59, "views": 8781, "answer_pids": [172827, 172857, 172885], "question_author": ""} {"qid": 816, "query": "Why arent we all Borg?", "score": 59, "views": 8591, "answer_pids": [112014, 112015, 112969, 116330, 118988, 133002, 159966, 171913], "question_author": ""} {"qid": 817, "query": "Why did the stormtroopers care about hiding the fact that they killed the Jawas?", "score": 59, "views": 8458, "answer_pids": [151133, 152346, 166292], "question_author": "esm"} {"qid": 818, "query": "Was Tolkien the first to invent languages purely for fictional works?", "score": 59, "views": 7999, "answer_pids": [142396, 142409, 142411, 142450, 142457, 142504, 186633], "question_author": "Albort"} {"qid": 819, "query": "How many Ents were at the Entmoot?", "score": 59, "views": 7077, "answer_pids": [172619, 172620, 180367], "question_author": "Albort"} {"qid": 820, "query": "Does Thor know that Thursday is named after him?", "score": 59, "views": 5684, "answer_pids": [161659], "question_author": "Bobby Ketchum"} {"qid": 821, "query": "Where did the Wizarding World think Harry was?", "score": 59, "views": 4707, "answer_pids": [145425, 145429], "question_author": ""} {"qid": 822, "query": "Spaceship from a film or show that is semi circular in shape with a section jutting out of the middle", "score": 59, "views": 2010, "answer_pids": [204052], "question_author": ""} {"qid": 823, "query": "Significance of the origami Deckard found in Blade Runner", "score": 58, "views": 135602, "answer_pids": [109792, 109946, 111581, 122304, 123265, 129754], "question_author": "Albort"} {"qid": 824, "query": "Do any clones refuse Order 66?", "score": 58, "views": 105601, "answer_pids": [139045, 139085, 146253], "question_author": ""} {"qid": 825, "query": "Is it possible that Mace Windu survived his encounter with Sidious?", "score": 58, "views": 94411, "answer_pids": [115906, 115918, 116055, 116868, 140255], "question_author": ""} {"qid": 826, "query": "What is the symbolism of the colors assigned to Istari wizards?", "score": 58, "views": 57017, "answer_pids": [112217, 113253, 138232, 153528], "question_author": "heavilyinvolved"} {"qid": 827, "query": "Who was the little girl in the matrix and why was she so important?", "score": 58, "views": 46478, "answer_pids": [115804, 124892, 124958], "question_author": ""} {"qid": 828, "query": "How did Vader saving Luke turn him to the good side?", "score": 58, "views": 45254, "answer_pids": [150866, 150870, 153679, 157521, 174035, 193283], "question_author": "Maniero"} {"qid": 829, "query": "Why did Tony Stark build arc reactors into his extra Iron Man suits?", "score": 58, "views": 37532, "answer_pids": [117054, 120178, 122490, 122543, 124069, 125855, 125889, 126517, 126651, 128584, 128803], "question_author": "Maniero"} {"qid": 830, "query": "Wasnt Harrys father just like Draco?", "score": 58, "views": 36463, "answer_pids": [115994, 116000, 126691, 133018, 145850], "question_author": ""} {"qid": 831, "query": "How did the AT-ATs get to the surface of Hoth?", "score": 58, "views": 35396, "answer_pids": [113939, 123544, 144637, 168407], "question_author": "3rdparty"} {"qid": 832, "query": "Was Elrond, in Tolkiens opinion, more inherently powerful than Galadriel?", "score": 58, "views": 34693, "answer_pids": [123420, 123424, 123429, 123457, 134721, 155084, 155769, 160046], "question_author": "LudoMC"} {"qid": 833, "query": "Who or what is the oldest sentient being in Tolkiens lore?", "score": 58, "views": 34248, "answer_pids": [132018, 132020, 132023, 132052, 132070, 132074, 132084, 132118, 145343, 170328], "question_author": ""} {"qid": 834, "query": "Why didnt Tom Riddle kill Harry Potter in Chamber of Secrets?", "score": 58, "views": 33703, "answer_pids": [134313, 134317, 134337], "question_author": "Maniero"} {"qid": 835, "query": "Why arent there any main Federation battleships in the Star Trek universe?", "score": 58, "views": 32339, "answer_pids": [125014, 125015, 125022, 125023, 125029, 131438, 132830, 134191, 139402, 166644, 172798, 202012], "question_author": ""} {"qid": 836, "query": "Did Sauron give any care to Saruman or was he just a pawn?", "score": 58, "views": 30741, "answer_pids": [111245, 111246, 111249, 112052, 137499, 141129, 156947], "question_author": "jfoucher"} {"qid": 837, "query": "How did Paul Atreides Differ From the Intended Kwisatz Haderach?", "score": 58, "views": 25284, "answer_pids": [115205, 115220, 115223, 121445, 127631, 137403, 155267], "question_author": "GameDaddy"} {"qid": 838, "query": "How does Stark Enterprises maintain its cashflow?", "score": 58, "views": 23783, "answer_pids": [117116, 178256], "question_author": ""} {"qid": 839, "query": "What order should the Chronicles of Narnia books be read in?", "score": 58, "views": 18544, "answer_pids": [117486, 117487, 117488, 117489, 127033, 140351], "question_author": ""} {"qid": 840, "query": "Why did Gandalf not return to help Frodo and Sam?", "score": 58, "views": 13884, "answer_pids": [176333, 176335, 176336, 176343, 176346], "question_author": "jfoucher"} {"qid": 841, "query": "Which of Dumbledores abilities make him the greatest wizard of the age?", "score": 58, "views": 13236, "answer_pids": [154164, 154166, 154171, 154185, 154249, 155375, 155383], "question_author": "Extrakun"} {"qid": 842, "query": "What form would a boggart take for Snape?", "score": 58, "views": 13085, "answer_pids": [173253, 173254, 173255, 173322, 173388], "question_author": ""} {"qid": 843, "query": "Did Harry Potter ever get on a Famous Witches and Wizards card?", "score": 58, "views": 13021, "answer_pids": [153160, 153161, 160045], "question_author": "Andr\u00e9 Carregal"} {"qid": 844, "query": "Why does the Trade Federation become so alarmed upon learning the ambassadors are Jedi Knights?", "score": 58, "views": 12081, "answer_pids": [198960, 198961, 198985, 198987, 199025], "question_author": ""} {"qid": 845, "query": "Are any Harry Potter characters completely redeemed?", "score": 58, "views": 11552, "answer_pids": [158543, 158550, 158559, 158562, 174505, 179864, 179871], "question_author": ""} {"qid": 846, "query": "Was it Dumbledores charm or Lilys that protected Harry all those years?", "score": 58, "views": 11456, "answer_pids": [191101, 191104, 191105], "question_author": "Brian Ballsun-Stanton"} {"qid": 847, "query": "Did Balrogs have their own language?", "score": 58, "views": 11129, "answer_pids": [168466, 168468, 168470, 168493, 168522], "question_author": "RaYell"} {"qid": 848, "query": "How is it believable that Euron could so easily pull off this ambush?", "score": 58, "views": 11094, "answer_pids": [197200, 197202, 197206, 197213, 197219], "question_author": "RMorrisey"} {"qid": 849, "query": "How does Ant-Man pick up a drop of water?", "score": 58, "views": 10731, "answer_pids": [150255, 150283], "question_author": ""} {"qid": 850, "query": "Have there ever been 25 or more stars on Wonder Womans panties?", "score": 58, "views": 10713, "answer_pids": [177210, 177227, 177245], "question_author": ""} {"qid": 851, "query": "Why isnt the religion of Rhllor, The Lord of Light, dominant?", "score": 58, "views": 10505, "answer_pids": [172100, 172101, 172102, 172158], "question_author": ""} {"qid": 852, "query": "How do I get my PCs to not be a bunch of murderous cretins?", "score": 532, "views": 64808, "answer_pids": [5133, 5134, 5136, 5137, 5138, 5149, 5156, 5157, 5158, 5649, 5654, 6758, 6763, 6786, 6787, 7353, 8527, 10826, 11203, 11238, 14032, 14451, 18678, 31975, 32243, 33256, 42119, 46063, 87720], "question_author": "rjbs"} {"qid": 853, "query": "What is my guy syndrome and how do I handle it?", "score": 412, "views": 160634, "answer_pids": [20214, 20215, 20221, 35434, 47994], "question_author": "AJ Finch"} {"qid": 854, "query": "Managing a Medieval Low Orbit Ion Cannon", "score": 258, "views": 34055, "answer_pids": [35762, 35765, 35767, 35769, 35770, 35771, 35780, 35785, 35787, 35789, 35790, 35809, 35996, 36121, 43753], "question_author": ""} {"qid": 855, "query": "Without a magic item economy, what is gold for?", "score": 232, "views": 119906, "answer_pids": [23856, 23859, 23862, 23866, 23868, 23877, 23878, 23911, 23912, 23915, 39909, 40964, 54380, 66659], "question_author": "Brisbe42"} {"qid": 856, "query": "Dealing with difficult male players, when youre a new female player?", "score": 213, "views": 31234, "answer_pids": [25993, 25996, 25999, 26000, 26002, 26006, 26007, 26008, 26012, 26013, 26015, 26017, 26018, 26023, 26029, 26099, 26161], "question_author": "Rowland Shaw"} {"qid": 857, "query": "How does rolling two d20 and taking the higher affect the average outcome?", "score": 201, "views": 84696, "answer_pids": [9365, 9366, 9373, 35640, 38393, 51641], "question_author": "Stefano Borini"} {"qid": 858, "query": "Im at a loss with \u201cDungeons and Dragons.\u201d How does one play it, anyway?", "score": 185, "views": 77236, "answer_pids": [7030, 7031, 7032, 7033, 7036, 7938, 8623, 10331, 18359], "question_author": "EtienneT"} {"qid": 859, "query": "Player claims their wizard character knows everything (from books). Solutions?", "score": 184, "views": 75094, "answer_pids": [41820, 41821, 41822, 41823, 41824, 41825, 41827, 41837, 41847, 41863, 41889, 41975, 53336, 61530], "question_author": ""} {"qid": 860, "query": "How does a player correct a GM mistake without being a rules lawyer or pushover?", "score": 182, "views": 18137, "answer_pids": [2729, 2730, 2731, 2732, 2737, 2739, 2742, 5939, 7313, 16259], "question_author": ""} {"qid": 861, "query": "What are the primary differences between the rules of D&D 3.5 and Pathfinder?", "score": 176, "views": 159620, "answer_pids": [0, 1, 43, 1970, 65061], "question_author": "Matt Bishop"} {"qid": 862, "query": "How to let my players fail their rolls intentionally, but covertly?", "score": 174, "views": 13625, "answer_pids": [44321, 44337, 44344, 44346, 44363, 44383, 78350], "question_author": ""} {"qid": 863, "query": "Whats the greatest number of hands I can have to annoy my mother-in-law with?", "score": 172, "views": 12542, "answer_pids": [76627, 76628, 76633, 76635, 76677, 77199], "question_author": ""} {"qid": 864, "query": "How to deal with players who like to use the local authorities to solve their problems", "score": 158, "views": 20995, "answer_pids": [10958, 10959, 10960, 10961, 10962, 10963, 10966, 10967, 10969, 10970, 10974, 10976, 10978, 10981, 10985, 10991], "question_author": ""} {"qid": 865, "query": "How can DMs effectively telegraph specific dangers in D&D?", "score": 155, "views": 19499, "answer_pids": [2220, 2224, 2229, 2262, 8898, 17134, 21321, 35469, 35478, 35519, 52805], "question_author": ""} {"qid": 866, "query": "Players skipping side quests just to have a laugh at the DM", "score": 154, "views": 52608, "answer_pids": [55556, 55558, 55559, 55562, 55563, 55564, 55565, 55568, 55569, 55573, 55594, 55606, 55615, 55616, 55617, 55619, 55658, 55662, 55687, 55698, 55772, 55827, 61729], "question_author": "John"} {"qid": 867, "query": "What is the source of the spells do only what they say they do rules interpretation principle?", "score": 154, "views": 20148, "answer_pids": [51116, 51117, 51125, 51134, 53629, 64776, 88713], "question_author": "Matt Bishop"} {"qid": 868, "query": "How do you help players not focus on the rules?", "score": 147, "views": 18519, "answer_pids": [3952, 3954, 3955, 3962, 3967, 3971, 3974, 3975, 3979, 3980, 3983, 3993, 3997, 4003, 4029, 4033, 6484, 25151, 51690], "question_author": ""} {"qid": 869, "query": "Is there a maximum attainable beard thickness?", "score": 147, "views": 15172, "answer_pids": [60758, 60759, 60845, 71742], "question_author": "zacharythefirst"} {"qid": 870, "query": "Who created the idea of Experience Points?", "score": 146, "views": 16697, "answer_pids": [4777, 4780, 4784, 4787, 4792, 4797, 4803, 4820, 4830, 4831, 4854, 10845, 10910], "question_author": ""} {"qid": 871, "query": "Can I begin roleplaying if Im visually impaired?", "score": 144, "views": 11783, "answer_pids": [40283, 40284, 40292, 40295, 40300, 40301, 40325, 40355, 41927], "question_author": "DavRob60"} {"qid": 872, "query": "Can you drink a potion one-handed underwater?", "score": 143, "views": 36083, "answer_pids": [40218, 40219, 40220, 40221, 40223, 40229, 40323], "question_author": "MPelletier"} {"qid": 873, "query": "Betrayed by my GM; how can I respond?", "score": 143, "views": 29428, "answer_pids": [16506, 16507, 16508, 16509, 16511, 16514, 16515, 16516, 19209, 38418, 39381], "question_author": ""} {"qid": 874, "query": "How to stop players from making the game X-rated", "score": 138, "views": 23025, "answer_pids": [56146, 56147, 56201, 56212, 56219, 77387], "question_author": "Grant Palin"} {"qid": 875, "query": "What can I do when I accidentally gave out an overpowered item?", "score": 136, "views": 25762, "answer_pids": [31906, 31907, 31908, 31909, 31910, 31911, 31912, 31915, 31934, 31935, 31999, 33180], "question_author": ""} {"qid": 876, "query": "Where can I find other RPG players?", "score": 133, "views": 33792, "answer_pids": [747, 748, 751, 778, 2707, 3295, 9130, 9320, 9340, 16834, 18108, 18718, 31788, 36487, 42498, 77309, 79617, 87307], "question_author": ""} {"qid": 877, "query": "Are peoples competencies really as flat in D&D 5e as its math suggests?", "score": 131, "views": 45091, "answer_pids": [22300, 22303, 22307, 22309, 22317], "question_author": ""} {"qid": 878, "query": "New girl gamer in our D&D group is causing weird tension - what to do?", "score": 129, "views": 115431, "answer_pids": [43480, 43481, 43482, 43483, 43484, 43487, 44430, 48201], "question_author": ""} {"qid": 879, "query": "What are the big differences among the D&D editions?", "score": 128, "views": 179718, "answer_pids": [8423, 8427, 17519], "question_author": ""} {"qid": 880, "query": "Whats the point of long, empty hallways in dungeons?", "score": 126, "views": 25753, "answer_pids": [46801, 46809, 46810, 46817, 46832, 46864, 46889, 61730], "question_author": "Iain M Norman"} {"qid": 881, "query": "Evil Campaigns: How to explain the difference between being evil and being a jerk?", "score": 126, "views": 23150, "answer_pids": [57214, 57216, 57221, 57224, 57228, 57238, 57259, 57269, 57270, 57303, 57365], "question_author": ""} {"qid": 882, "query": "As a man, how can I roleplay a woman better?", "score": 124, "views": 40984, "answer_pids": [2651, 2654, 2661, 2673, 2689, 2701, 2717, 5113, 11239, 88985], "question_author": "John"} {"qid": 883, "query": "PC ate a necklace of fireballs bead. How dead should they be?", "score": 124, "views": 30153, "answer_pids": [51973, 51974, 51977, 51981, 51983, 51995, 52002, 52096], "question_author": "pian0"} {"qid": 884, "query": "Why would I ever cast True Strike?", "score": 122, "views": 109504, "answer_pids": [25749, 25902, 42019, 51321, 54059, 89062], "question_author": "Daniel Bingham"} {"qid": 885, "query": "What is a session 0?", "score": 120, "views": 65611, "answer_pids": [49947, 49948, 49949, 49950, 49960, 49969], "question_author": "mafu"} {"qid": 886, "query": "Given powerful low-level Clerical healing, how can sick, crippled or otherwise unhealthy people exist?", "score": 119, "views": 20441, "answer_pids": [39236, 39237, 39246, 39247, 39252, 39253, 39256, 39261, 39341, 39345], "question_author": "txwikinger"} {"qid": 887, "query": "As a player, I am approaching a situation where My Guy syndrome seems inevitable and almost appropriate. What can I do to soften any problems?", "score": 117, "views": 23173, "answer_pids": [47765, 47766, 47767, 47769, 47782, 47791, 47794, 47834, 47852], "question_author": "txwikinger"} {"qid": 888, "query": "How to prevent metagaming induced by trying to not metagame?", "score": 115, "views": 17055, "answer_pids": [57737, 57738, 57741, 57742, 57744, 57747, 57749, 57764, 57771, 57786, 57789, 57797, 57799, 57812, 57815, 57871], "question_author": ""} {"qid": 889, "query": "Why is the D&D gorgon a metal bull?", "score": 114, "views": 46615, "answer_pids": [52999], "question_author": "txwikinger"} {"qid": 890, "query": "How to deal with player MacGyver-ism?", "score": 114, "views": 26373, "answer_pids": [25035, 25037, 25038, 25039, 25040, 25043, 25044, 25047, 25048, 25049, 25050, 25056, 25063, 25067, 25072, 25095, 25183], "question_author": "matiash"} {"qid": 891, "query": "What tools are useful to organize a GMs campaign notes?", "score": 112, "views": 47745, "answer_pids": [133, 136, 138, 146, 150, 187, 189, 204, 207, 216, 219, 222, 233, 379, 396, 423, 462, 1136, 1175, 1643, 1940, 2313, 2316, 2319, 2383, 5021, 5511, 5724, 5725, 6218, 7114, 8514, 9162, 9632, 12599, 13707, 23965], "question_author": "mafu"} {"qid": 892, "query": "How can we handle firing our DM?", "score": 112, "views": 21711, "answer_pids": [47856, 47857, 47859, 47874, 47875, 47879], "question_author": ""} {"qid": 893, "query": "Can a snake be prone?", "score": 112, "views": 16075, "answer_pids": [54736, 54737, 54738, 54743, 54745, 54804], "question_author": ""} {"qid": 894, "query": "Is there a legal way to get D&D 5e core rulebook PDFs?", "score": 111, "views": 206038, "answer_pids": [24291, 25653, 25672, 27469, 28056, 40236, 45569], "question_author": "Drew Hoskins"} {"qid": 895, "query": "How should a GM deal with standing orders?", "score": 111, "views": 9784, "answer_pids": [14127, 14128, 14129, 14130, 14131, 14135, 14136, 14142, 14143, 14149, 14317, 15147, 25075], "question_author": "jrista"} {"qid": 896, "query": "I failed to open a lock. Now what?", "score": 110, "views": 34581, "answer_pids": [24703, 24704, 24705, 24709, 24726, 24727, 24746, 38737], "question_author": "R. Martinho Fernandes"} {"qid": 897, "query": "As a player with ADHD, how do I avoid disrupting the game?", "score": 109, "views": 18092, "answer_pids": [64927, 64928, 64930, 64931, 64940, 64948, 64976, 64983, 65042], "question_author": "Tim Post"} {"qid": 898, "query": "Dealing with (supposedly) in-character bullying", "score": 109, "views": 15624, "answer_pids": [59831, 59832, 59833, 59837, 59841, 59848, 59849, 59871, 60377, 74557], "question_author": "beggs"} {"qid": 899, "query": "How can I test whether a die is fair?", "score": 107, "views": 56404, "answer_pids": [33658, 33769], "question_author": ""} {"qid": 900, "query": "When should I fudge dice rolls?", "score": 107, "views": 19728, "answer_pids": [17070, 17071, 17072, 17073, 17074, 17075, 17076, 17077, 17078, 17093, 17094, 17109, 17140, 35586, 51715, 69042], "question_author": "Edd"} {"qid": 901, "query": "Is there a limit to Rule 0?", "score": 107, "views": 14944, "answer_pids": [13036, 13037, 13038, 13039, 13040, 13041, 13042, 13053, 13090, 16987, 17190, 65464], "question_author": ""} {"qid": 902, "query": "Why are psionics controversial?", "score": 106, "views": 21017, "answer_pids": [34469, 34471, 34477, 81018], "question_author": ""} {"qid": 903, "query": "How can I make my PCs flee?", "score": 106, "views": 17035, "answer_pids": [10559, 10560, 10566, 10571, 10576, 10577, 10581, 10588, 10589, 10735, 10742, 10746, 10975, 11291, 15365, 35433, 61051, 65465], "question_author": "liewl"} {"qid": 904, "query": "How many people does it take to steal a Star Destroyer?", "score": 106, "views": 15541, "answer_pids": [5106, 5107, 5108, 5110, 5111, 5117, 5124, 5264, 5265, 7232, 17532], "question_author": "mafu"} {"qid": 905, "query": "Why does everyone carry a 10 pole?", "score": 104, "views": 49432, "answer_pids": [5326, 5327, 5328, 5331, 5333, 5334, 5335, 5340, 5350, 9071, 9081, 10673, 13175, 18116, 19694], "question_author": ""} {"qid": 906, "query": "What is E6? Why would I use it?", "score": 104, "views": 24304, "answer_pids": [45676, 46288], "question_author": ""} {"qid": 907, "query": "What do you do when your players guess your plots?", "score": 104, "views": 14785, "answer_pids": [46148, 46156, 46157, 46158, 46162, 46174, 46179, 46180, 46216, 46222, 46278, 46301, 46313, 54666], "question_author": ""} {"qid": 908, "query": "What are the most and least-resisted damage types?", "score": 103, "views": 255788, "answer_pids": [48775, 48803], "question_author": "che"} {"qid": 909, "query": "What do the terms lines and veils mean?", "score": 103, "views": 54477, "answer_pids": [17211, 17214], "question_author": "Scott Mitchell"} {"qid": 910, "query": "How do I play a paladin without being a stick in the mud?", "score": 102, "views": 97374, "answer_pids": [11324, 11325, 11358, 11380, 13915, 13917, 13919, 13921, 15971, 15972, 15985, 16136, 19676, 22148, 22156, 22366], "question_author": "MPelletier"} {"qid": 911, "query": "Why does \u201cdungeon\u201d mean the places adventurers go to kill stuff?", "score": 102, "views": 16813, "answer_pids": [31974], "question_author": "Stefano Borini"} {"qid": 912, "query": "How do I not cheapen death, while also respecting my players time?", "score": 102, "views": 9732, "answer_pids": [11469, 11470, 11471, 11472, 11473, 11475, 11476, 11477, 11481, 11484, 11486, 11489, 11490, 11495, 11497, 11505, 16023], "question_author": "txwikinger"} {"qid": 913, "query": "3d6 vs a d20: What is the effect of a different probability curve?", "score": 99, "views": 29426, "answer_pids": [1675, 1676, 1677, 1683, 1734, 1746, 1871, 2264, 4286, 4291, 4292], "question_author": ""} {"qid": 914, "query": "What are the playable D&D races in 5e, and where can I find them?", "score": 98, "views": 481194, "answer_pids": [36392], "question_author": ""} {"qid": 915, "query": "How has D&Ds guidance to DMs on when to extrapolate from written rules and when to improvise changed over time?", "score": 98, "views": 28606, "answer_pids": [16326], "question_author": ""} {"qid": 916, "query": "Where do adventurers keep all their stuff?", "score": 95, "views": 45618, "answer_pids": [25098, 25099, 25100, 25101, 25102, 25103, 25108, 26852, 59965], "question_author": "txwikinger"} {"qid": 917, "query": "Can you multiclass the same class twice for different class features?", "score": 94, "views": 65449, "answer_pids": [23359, 23361, 23364, 23379], "question_author": ""} {"qid": 918, "query": "How do I handle my players killing things by catapulting a folding boat?", "score": 94, "views": 29029, "answer_pids": [37234, 37235, 37237, 37239, 37248, 37254, 37266, 37268, 37280, 37296, 37297, 37307, 37352, 53405], "question_author": ""} {"qid": 919, "query": "As a girl, how can I voice male characters effectively?", "score": 92, "views": 20632, "answer_pids": [74403, 74404, 74405, 74406, 74409, 74411, 74419, 74422], "question_author": "Matt Bishop"} {"qid": 920, "query": "Is there a real use for the Medicine skill?", "score": 91, "views": 221377, "answer_pids": [25495, 25507, 25531, 58015, 66173, 66178, 66181, 66235, 66240, 66243, 74955], "question_author": "txwikinger"} {"qid": 921, "query": "How do I roleplay a character more intelligent than I am?", "score": 91, "views": 26624, "answer_pids": [27, 29, 30, 32, 35, 106, 109, 1517, 1524, 1678, 4729, 8820, 8827, 12967, 16130, 24766, 32842], "question_author": ""} {"qid": 922, "query": "Did I overstep my power as DM?", "score": 90, "views": 23699, "answer_pids": [18443, 18444, 18445, 18446, 18450, 18451, 18471, 19871, 45272, 63402], "question_author": ""} {"qid": 923, "query": "How do I communicate to my players that a door is, for the time being, absolutely locked to them?", "score": 90, "views": 21931, "answer_pids": [61486, 61488, 61489, 61490, 61495, 61496, 61501, 61513, 61515, 61519, 61546, 61643, 61645], "question_author": ""} {"qid": 924, "query": "Total Party Kill - What do I do as a GM?", "score": 90, "views": 21843, "answer_pids": [545, 546, 547, 557, 564, 571, 773, 780, 828, 1352, 1618, 1741, 4802, 4880, 12976, 15970], "question_author": "Jean-Philippe Caruana"} {"qid": 925, "query": "How do I keep the game moving without undermining player agency?", "score": 90, "views": 12523, "answer_pids": [25953, 25954, 25957, 25958, 25959, 25964, 25968, 25969, 25991, 26022], "question_author": "warren"} {"qid": 926, "query": "Rolling a D10 (0-9) for hitpoints. What do I get with a zero?", "score": 89, "views": 19440, "answer_pids": [62166, 62167, 62168, 62170, 62171], "question_author": ""} {"qid": 927, "query": "Why do Crawfords tweets seem to be treated on par with the actual rules?", "score": 89, "views": 15500, "answer_pids": [45694, 65424], "question_author": "Jonathan Campbell"} {"qid": 928, "query": "How should a GM deal with sexuality in an RPG?", "score": 89, "views": 12421, "answer_pids": [100, 103, 166, 356, 662, 702, 766, 1051, 1595, 2105, 20036, 20053], "question_author": ""} {"qid": 929, "query": "Do Cantrips use your character level or class level?", "score": 88, "views": 149488, "answer_pids": [23346, 23745, 23938, 40479, 40703], "question_author": "Fixpoint"} {"qid": 930, "query": "Why is burying yourself not such a great plan?", "score": 88, "views": 24961, "answer_pids": [22801, 22802, 22803, 22808, 22812, 22833, 22835, 22839, 22845, 22847, 22849, 22854, 22878, 22922, 22932, 22933, 22938, 29479, 51025, 75289, 83455, 83481], "question_author": ""} {"qid": 931, "query": "Player may be manipulating die rolls, but Im uncertain. How do I approach the situation?", "score": 88, "views": 23189, "answer_pids": [46229, 46231, 46233, 46254, 46258, 46260, 46290, 46299, 46300, 46330, 46348], "question_author": "squelart"} {"qid": 932, "query": "How do I stop my PCs from acting like insomniac monkeys on crack?", "score": 88, "views": 10562, "answer_pids": [7498, 7499, 7500, 7501, 7504, 7511, 7512, 7520, 7653, 7654, 7657, 9080, 14480, 22519], "question_author": "Hamish Downer"} {"qid": 933, "query": "What exactly is a murder hobo?", "score": 87, "views": 48691, "answer_pids": [72069, 72070, 72072, 72073, 72092], "question_author": "Maximillian"} {"qid": 934, "query": "What happens when an Immovable Rod is activated while in a vehicle?", "score": 87, "views": 32122, "answer_pids": [33206, 33207, 33210, 33217, 33218, 33235, 33250, 50106], "question_author": "Hamish Downer"} {"qid": 935, "query": "What is Pathfinders relationship to D&D?", "score": 87, "views": 27140, "answer_pids": [63399, 63401, 63403, 63404], "question_author": ""} {"qid": 936, "query": "Should I warn my players when theyre about to do something stupid?", "score": 87, "views": 14751, "answer_pids": [29291, 29300, 29301, 29304, 29318, 29321, 29322, 29329], "question_author": "lilserf"} {"qid": 937, "query": "What is the Oberoni Fallacy?", "score": 87, "views": 10590, "answer_pids": [33341], "question_author": "Stefano Borini"} {"qid": 938, "query": "What can I do to paper to make it look aged?", "score": 86, "views": 49970, "answer_pids": [23986, 23988, 23989, 23990, 23991, 24000, 24014, 24017, 24024, 24043, 68615], "question_author": "Tony Meyer"} {"qid": 939, "query": "How to ask nicely in Dungeon World", "score": 86, "views": 26427, "answer_pids": [31655, 81926], "question_author": "Tony Meyer"} {"qid": 940, "query": "My group member is displaying disturbing behavior, what do I do?", "score": 86, "views": 20706, "answer_pids": [58486, 58488, 58530, 58532, 58575, 58619], "question_author": "Tony Meyer"} {"qid": 941, "query": "Does this character concept involving never taking a long rest and converting spell slots to sorcery points (aka coffeelock) violate RAW?", "score": 86, "views": 16438, "answer_pids": [37247, 37264, 37376, 50980, 56474, 84162], "question_author": "greatwolf"} {"qid": 942, "query": "How much is a Gold Piece really worth?", "score": 85, "views": 116881, "answer_pids": [29660, 29666, 29667, 29675, 29676, 29678, 29680, 29682, 29690, 29703], "question_author": "Tony Meyer"} {"qid": 943, "query": "Is the Moonbeam spell amazing, or are we doing it wrong?", "score": 85, "views": 94624, "answer_pids": [26203, 28252, 36618, 37269], "question_author": ""} {"qid": 944, "query": "How to stop a player constantly using Detect Magic/Evil?", "score": 84, "views": 47641, "answer_pids": [39765, 39768, 39773, 39774, 39775, 39779, 39783, 39787, 39792, 39798, 39808, 39830, 39838, 39889, 39976], "question_author": ""} {"qid": 945, "query": "A player always wants to recruit NPCs into the party. How do I handle this?", "score": 84, "views": 36099, "answer_pids": [46190, 46191, 46193, 46200, 46209, 46220, 46230, 46292, 46338, 46341, 46358, 46360, 46371, 46396], "question_author": ""} {"qid": 946, "query": "My PCs have a plan that will get them all killed; how and why should I save them?", "score": 84, "views": 21298, "answer_pids": [30544, 30546, 30547, 30549, 30550, 30555, 30557, 30558, 30559, 30560, 30632, 30653], "question_author": "chills42"} {"qid": 947, "query": "What does upper-case-A-Attack action vs. lower-case-a-attack mean?", "score": 84, "views": 11854, "answer_pids": [50143, 50146, 50190], "question_author": "Mark Rogers"} {"qid": 948, "query": "Can you choose to fail a saving throw?", "score": 83, "views": 41677, "answer_pids": [23792, 23794, 23795, 23805, 37288], "question_author": "Nick DeVore"} {"qid": 949, "query": "How should a DM resolve a smooth-talking player with a weak Charisma score PC?", "score": 83, "views": 18809, "answer_pids": [66600, 66602, 66604, 66614, 66621, 66630, 66645, 66648, 66651, 66664, 66677, 66679, 66716, 66721, 66727, 66730], "question_author": ""} {"qid": 950, "query": "Why does dandwiki have a poor reputation?", "score": 83, "views": 16403, "answer_pids": [52000, 52001, 52013, 52460], "question_author": ""} {"qid": 951, "query": "How can I stop myself from micromanaging other PCs actions?", "score": 83, "views": 10454, "answer_pids": [72700, 72701, 72702, 72710, 72725], "question_author": ""} {"qid": 952, "query": "Who can use magic scrolls?", "score": 82, "views": 173941, "answer_pids": [26127, 26132], "question_author": ""} {"qid": 953, "query": "Is saying Your PC wouldnt do that to a player denying their agency?", "score": 82, "views": 11950, "answer_pids": [38269, 38271, 38273, 38278, 38279, 38283, 38286, 38541, 71496], "question_author": "Quickhorn"} {"qid": 954, "query": "D&D 5e and Theatre of the Mind in combat", "score": 81, "views": 34367, "answer_pids": [20873, 20874, 20875, 20884, 24049, 24627, 67153], "question_author": "Toon Krijthe"} {"qid": 955, "query": "How can a GM quickly create interesting, engaging NPCs?", "score": 81, "views": 27107, "answer_pids": [1226, 1228, 1239, 1247, 1254, 1255, 1265, 1297, 1397, 1400, 1408, 5895, 5900, 6606, 6607, 7909, 12991, 13023, 13043, 13046, 13071, 13084, 16813], "question_author": ""} {"qid": 956, "query": "How can I avoid players spending too much time planning?", "score": 81, "views": 6348, "answer_pids": [8908, 8909, 8910, 8911, 8913, 8914, 8919, 8931, 8935, 8937, 8938, 8939, 8950, 8962, 9492, 9501, 9503, 9532, 9733, 10693, 11536, 30051], "question_author": ""} {"qid": 957, "query": "How can I suggest the DM stop trying to kill us?", "score": 80, "views": 34317, "answer_pids": [30177, 30178, 30179, 30185, 30209, 30280, 30303, 30343, 87983], "question_author": ""} {"qid": 958, "query": "Running a game of Microscope that isnt totally insane?", "score": 80, "views": 20688, "answer_pids": [13713, 13716, 13721, 18031], "question_author": ""} {"qid": 959, "query": "How can I get a player to accept that they should stop trying to pull stunts without thinking them through first?", "score": 80, "views": 12804, "answer_pids": [72191, 72195, 72208, 72209, 72234, 72304], "question_author": "GWLlosa"} {"qid": 960, "query": "OD&D said it could be played with 20-50 players and one referee. How was that expected to work and still be fun?", "score": 80, "views": 10210, "answer_pids": [43274, 43275, 43277, 43282], "question_author": "zacharythefirst"} {"qid": 961, "query": "Where do I find the official rules for D&D 5e?", "score": 80, "views": 8624, "answer_pids": [39832], "question_author": "Alan"} {"qid": 962, "query": "Do you have to have thieves tools in order to pick a lock?", "score": 79, "views": 79528, "answer_pids": [25545, 25553, 59158, 88558], "question_author": ""} {"qid": 963, "query": "Can a Black Dragon Hatchling be raised to be good? Or is it inherently evil?", "score": 79, "views": 35297, "answer_pids": [36148, 36149, 36150, 36151, 36157, 36159, 36173, 36182, 36202, 63560], "question_author": "Reinstate Monica - Goodbye SE"} {"qid": 964, "query": "How should I respond to a DM who unfairly plays the rules-lawyer card?", "score": 79, "views": 26131, "answer_pids": [10714, 10716, 10717, 10718, 10721, 10726, 10727, 10728, 21706], "question_author": ""} {"qid": 965, "query": "How can I explain to my player that Alchemists Fire is safe to carry?", "score": 79, "views": 23861, "answer_pids": [66322, 66324, 66325, 66327, 66336, 66343, 66345, 66347, 66348, 66585, 66640, 66654], "question_author": "Reinstate Monica - Goodbye SE"} {"qid": 966, "query": "What happens when my players get promoted and their puns stop working?", "score": 79, "views": 11411, "answer_pids": [47608, 47614, 47619, 47620], "question_author": "Simon Withers"} {"qid": 967, "query": "How can I help my PCs remember clues that they found?", "score": 79, "views": 6114, "answer_pids": [9634, 9635, 9636, 9637, 9640, 9645, 9646, 9647, 9653, 9711, 9758, 9826, 9856, 15039, 15418], "question_author": "ICodeForCoffee"} {"qid": 968, "query": "Why would I ever choose rolling hit points?", "score": 78, "views": 79175, "answer_pids": [24168, 24169, 24171, 24174, 24175, 24177, 24189, 24252, 25518, 25567, 25569, 25573, 30293, 32702, 32719, 34278, 61325, 76402], "question_author": "Aristos"} {"qid": 969, "query": "As a GM, is it bad form to ask for a moment to think when improvising?", "score": 78, "views": 8288, "answer_pids": [70142, 70143, 70144, 70146, 70147, 70148, 70170], "question_author": null} {"qid": 970, "query": "What is Player Agency and what is it good for?", "score": 77, "views": 43371, "answer_pids": [33883, 33888, 38369], "question_author": "Brisbe42"} {"qid": 971, "query": "What should I do about a religious player who refuses to accept the existence of multiple gods in D&D?", "score": 77, "views": 22095, "answer_pids": [70926, 70929, 70930, 70939, 70949, 70951, 70954, 70956, 70959, 70988, 70990, 70997, 71024, 72335], "question_author": "idan315"} {"qid": 972, "query": "Is it wrong to ask a player to justify their characters actions?", "score": 77, "views": 9439, "answer_pids": [16971, 16972, 16973, 16974, 16975, 16976, 16981, 16983, 16984, 16985, 17010, 28234], "question_author": "Morten Siebuhr"} {"qid": 973, "query": "Do I have to allow someone to play at my table in Adventurers League public events?", "score": 77, "views": 8432, "answer_pids": [44867, 45938, 46577], "question_author": ""} {"qid": 974, "query": "The StackOMounts", "score": 77, "views": 7839, "answer_pids": [22669], "question_author": ""} {"qid": 975, "query": "Does microwaving a die significantly alter its balance?", "score": 76, "views": 23342, "answer_pids": [53435, 53437], "question_author": "Dom"} {"qid": 976, "query": "Our DM keeps interrupting our long rests", "score": 76, "views": 16956, "answer_pids": [46609, 46610, 46611, 46614], "question_author": ""} {"qid": 977, "query": "How to fix a terrible lapse in judgement", "score": 76, "views": 9336, "answer_pids": [20417, 20418, 20419, 20420, 20422, 20436, 20447, 20453], "question_author": "Rodger Cooley"} {"qid": 978, "query": "What is role-playing, and where do I start?", "score": 76, "views": 8162, "answer_pids": [2832, 2833, 2835, 2837, 13815, 29522], "question_author": "chills42"} {"qid": 979, "query": "How can I prevent players from using Persuasion or Deception to weasel their way out of a murder?", "score": 75, "views": 18570, "answer_pids": [50793, 50794, 50795, 50796, 50808, 50811, 50812, 50816, 50834, 50875, 50891, 50895, 50962, 51002, 70586], "question_author": ""} {"qid": 980, "query": "How to deal with a player who refuses to learn the description of their spells and abilities?", "score": 75, "views": 16576, "answer_pids": [61058, 61059, 61064, 61066, 61083, 61137, 61206, 61213], "question_author": ""} {"qid": 981, "query": "Our GM wont let us affect the story; the more I play the less fun I have. What do I do?", "score": 75, "views": 11677, "answer_pids": [52911, 52912, 52936, 52938, 52960, 53050], "question_author": ""} {"qid": 982, "query": "What is the purpose of easy combat scenarios that dont need resource expenditure?", "score": 75, "views": 9464, "answer_pids": [67974, 67975, 67979, 67981, 67982, 67986, 67987, 67988, 67999, 68000, 68006, 68019], "question_author": "Dr Rob Lang"} {"qid": 983, "query": "What is the origin of the phrase \u2018orc and pie\u2019?", "score": 75, "views": 9186, "answer_pids": [65010, 65011, 65030], "question_author": ""} {"qid": 984, "query": "Whats the inspiration for the owlbear?", "score": 75, "views": 7564, "answer_pids": [18894], "question_author": ""} {"qid": 985, "query": "Whats a good mechanic for rewarding players for keeping their characters happy?", "score": 75, "views": 6277, "answer_pids": [12927, 12928, 12933, 12936, 12937, 12940, 12944, 12946, 12947, 12948, 12958], "question_author": ""} {"qid": 986, "query": "Im creating an original setting for D&D\u2014where do I start?", "score": 74, "views": 62334, "answer_pids": [683, 684, 685, 688, 717, 724, 741, 744, 754, 793, 826, 938, 1037, 1616, 8593, 9699, 11343, 17133], "question_author": ""} {"qid": 987, "query": "Are Trolls immune to all instant death effects?", "score": 74, "views": 14457, "answer_pids": [62919, 62920, 62921, 62944, 62952, 62957], "question_author": "Grant Palin"} {"qid": 988, "query": "How can I get people to remember my characters gender?", "score": 74, "views": 9694, "answer_pids": [70103, 70104, 70105, 70109], "question_author": ""} {"qid": 989, "query": "How do I narrate a players PCs actions without causing unintended consequences for the PC?", "score": 74, "views": 8587, "answer_pids": [43410, 43412, 43415, 43418, 43423, 43430, 43437, 43438, 43439, 43441, 43979, 68473], "question_author": ""} {"qid": 990, "query": "Does Warlock combat just equal Eldritch Blast spam?", "score": 73, "views": 190004, "answer_pids": [23326, 23404, 23483, 24756, 24785, 30715, 33652, 49580, 57273], "question_author": ""} {"qid": 991, "query": "How can I run an open-world game with vast power differences, without resulting in constant TPKs?", "score": 73, "views": 8839, "answer_pids": [76068, 76069, 76070, 76077, 76089, 76091, 76094, 76099, 76111, 76116, 76149], "question_author": ""} {"qid": 992, "query": "How do I get my players to form a PC party without just forcing them to?", "score": 72, "views": 73568, "answer_pids": [10611, 10613, 10710, 10715, 10724, 10725, 10749, 40884, 41056, 62494, 62764], "question_author": ""} {"qid": 993, "query": "Is starting a group with 5 newbies (DM included) possible?", "score": 72, "views": 10603, "answer_pids": [33470, 33471, 33475, 33482, 33485, 33519], "question_author": ""} {"qid": 994, "query": "Is 4 hours long enough for a long rest for Elves?", "score": 71, "views": 82725, "answer_pids": [21168, 24285, 29416, 37979, 78793, 88593], "question_author": ""} {"qid": 995, "query": "It seems like every skill check should always be made with advantage due to the Working Together rules. Is this accurate?", "score": 71, "views": 68334, "answer_pids": [27168, 27169, 27174, 27188, 27190, 27191, 31418, 31425, 31512], "question_author": ""} {"qid": 996, "query": "Whats the deal with alignment languages?", "score": 71, "views": 17248, "answer_pids": [6382, 6383, 6385, 6392, 6394, 22931], "question_author": ""} {"qid": 997, "query": "How to ease down the munchkin factor?", "score": 71, "views": 14863, "answer_pids": [25135, 25136, 25139, 25144, 25148, 25150, 25159, 25162, 25168], "question_author": null} {"qid": 998, "query": "What is a bag of rats?", "score": 71, "views": 13326, "answer_pids": [59727, 59731], "question_author": "chills42"} {"qid": 999, "query": "How to work with a player who cites ADHD as reason for disruptive behavior", "score": 71, "views": 12998, "answer_pids": [45908, 45909, 45910, 45914, 50729], "question_author": "Rodger Cooley"} {"qid": 1000, "query": "Problems with Party Formation: Youre the DM, you can figure it out.", "score": 71, "views": 12977, "answer_pids": [28326, 28327, 28329, 28330, 28337, 28338, 28342, 28349, 28363, 28371, 28387, 28393, 28395, 28472, 28483, 31232], "question_author": ""} {"qid": 1001, "query": "What dice mechanic gives a bell curve distribution that narrows and increases mean as skill increases?", "score": 70, "views": 19869, "answer_pids": [42599, 42605, 42611, 42614, 42629, 42641, 42661, 42668, 42688, 42690, 42705, 42728, 42755, 42872, 43950, 62263, 62489, 67357], "question_author": ""} {"qid": 1002, "query": "My DM insists on rolling a single save for groups affected by AoE save spells. How does this affect my odds of successfully affecting the enemy?", "score": 70, "views": 16108, "answer_pids": [45427, 45428, 45430, 45433, 45434, 45457, 45460, 45466, 45475], "question_author": "Rodger Cooley"} {"qid": 1003, "query": "How can metagaming retries on poor skill checks be prevented?", "score": 70, "views": 14260, "answer_pids": [56713, 56714, 56715, 56717, 56718, 56731, 56732, 56740, 56761, 56779, 56800], "question_author": "Rodger Cooley"} {"qid": 1004, "query": "Is it possible to fail an ability check on purpose?", "score": 70, "views": 12265, "answer_pids": [52141, 52142, 52149, 52150, 52151, 59334, 68561], "question_author": ""} {"qid": 1005, "query": "In a low-combat campaign, most PC abilities seem useless; what few fights there are barely use up resources. How do I deal with this as the DM?", "score": 70, "views": 6859, "answer_pids": [88513, 88516, 88521, 88525, 88528, 88531, 88547], "question_author": "Don Kirkby"} {"qid": 1006, "query": "When do I use active vs passive perception?", "score": 69, "views": 244468, "answer_pids": [24200, 24203, 24206, 24208, 24209, 25615, 45920, 51335], "question_author": ""} {"qid": 1007, "query": "Are there any penalties for wearing armor all the time?", "score": 69, "views": 78943, "answer_pids": [23592, 23595, 23596, 23599, 23604, 23620, 23633, 23646, 23812, 26790, 28248, 55221, 67184], "question_author": "Slick23"} {"qid": 1008, "query": "Are melee combatants limited to standing around saying I attack?", "score": 69, "views": 50148, "answer_pids": [22568, 22569, 22572, 22590], "question_author": "Alan"} {"qid": 1009, "query": "How do I manage a player who is handling traps by metagaming?", "score": 69, "views": 33660, "answer_pids": [39870, 39871, 39873, 39881, 39896, 39897, 39901, 39907, 39918, 39943, 39962, 39963, 39964, 39973, 45039], "question_author": ""} {"qid": 1010, "query": "What is the point of a spell component pouch?", "score": 69, "views": 32150, "answer_pids": [5604, 5607, 5608, 5623, 9216, 11294, 11873, 35431, 68044], "question_author": null} {"qid": 1011, "query": "I think my DM is consistently faking dice rolls for saves against a specific spell; how do I call my DM out?", "score": 69, "views": 31487, "answer_pids": [63817, 63821, 63823, 63825, 63830, 63832], "question_author": "Karel"} {"qid": 1012, "query": "What makes grease a good spell?", "score": 69, "views": 30377, "answer_pids": [10846, 10847, 10848, 10877, 11252, 35429], "question_author": ""} {"qid": 1013, "query": "I gave my players far too much wealth and they have far too much power. What do I do to get things back on track?", "score": 69, "views": 23731, "answer_pids": [63241, 63243, 63246, 63252, 63265, 63266, 63303, 63304], "question_author": ""} {"qid": 1014, "query": "How do I indicate to my players that an NPC is no longer useful and that they should move on?", "score": 69, "views": 16461, "answer_pids": [58969, 58970, 58977, 58980, 58992, 58998, 59019, 59063, 59066, 59096], "question_author": ""} {"qid": 1015, "query": "Dealing with PC that constantly lies to NPCs", "score": 69, "views": 15474, "answer_pids": [49189, 49190, 49191, 49192, 49194, 49195, 49196, 49198, 49261, 49323], "question_author": "Karel"} {"qid": 1016, "query": "How can I invoke actual fear in my players?", "score": 69, "views": 14472, "answer_pids": [12552, 12553, 12554, 12555, 12556, 12564, 12644, 17357, 17512, 37617], "question_author": ""} {"qid": 1017, "query": "How should the DM manage the discrepancy between the players memory and their PCs memory?", "score": 69, "views": 13621, "answer_pids": [36927, 36929, 36930, 36932, 36937, 36941], "question_author": "JoeGaggler"} {"qid": 1018, "query": "What is wrong with the D&D 3.5 FAQ?", "score": 69, "views": 9198, "answer_pids": [29234, 29235, 29244, 29267, 29317], "question_author": "Xavier V."} {"qid": 1019, "query": "What is the highest possible AC?", "score": 68, "views": 380775, "answer_pids": [31859, 31871, 36456, 41136, 41147, 57370, 82335, 84442], "question_author": ""} {"qid": 1020, "query": "Does Bards Jack of All Trades grant them greater initiative?", "score": 68, "views": 87864, "answer_pids": [23418, 23420], "question_author": ""} {"qid": 1021, "query": "What are tiers, and what tier is each class?", "score": 68, "views": 70959, "answer_pids": [20863], "question_author": ""} {"qid": 1022, "query": "Why is mayo in the alchemy jug?", "score": 68, "views": 51517, "answer_pids": [44155], "question_author": "Slick23"} {"qid": 1023, "query": "What happens when you run out of movement while jumping?", "score": 68, "views": 50696, "answer_pids": [27764, 27765, 27770, 35953, 36035, 48019, 56972, 76205, 81163], "question_author": "Brian Campbell"} {"qid": 1024, "query": "Is the old Linear Fighters Quadratic Wizards problem still around in 5e Basic?", "score": 68, "views": 48203, "answer_pids": [22316, 22318, 22466, 53291, 63379], "question_author": ""} {"qid": 1025, "query": "How does D&D 5e compare to AD&D 2e?", "score": 68, "views": 48022, "answer_pids": [23717, 33282], "question_author": "matt burns"} {"qid": 1026, "query": "How do you play D&D when we dont have dice to play with?", "score": 68, "views": 19044, "answer_pids": [20738, 20739, 20740, 20741, 20742, 20743, 20744, 20745, 20748, 20749, 20754, 20758, 20762, 20766, 20768, 20770, 20777, 20786, 20799, 20802, 20827, 29531, 74868, 88414, 88419, 88899], "question_author": "cfeduke"} {"qid": 1027, "query": "How do I challenge my PCs who fight every battle like Mongol horse archers?", "score": 68, "views": 18801, "answer_pids": [23813, 23815, 23816, 23817, 23818, 23842, 23843, 23851, 23867, 23869, 23890, 23895, 23899, 23906, 23913, 27090, 40319], "question_author": ""} {"qid": 1028, "query": "How to deal with a player who is a bad sport?", "score": 68, "views": 18471, "answer_pids": [53736, 53738, 53741, 53749, 53750, 53760, 53770, 53780, 53782, 53794, 65475], "question_author": "EtienneT"} {"qid": 1029, "query": "How can I impose consequences when PCs commit severe crimes?", "score": 68, "views": 17295, "answer_pids": [66352, 66353, 66356, 66357, 66369, 66375, 66386, 66398, 66405, 66444, 66482], "question_author": ""} {"qid": 1030, "query": "What is the probability of surviving my death saves?", "score": 68, "views": 15526, "answer_pids": [36243, 36255, 44232], "question_author": "splattered bits"} {"qid": 1031, "query": "My player wants to watch the world burn", "score": 68, "views": 13667, "answer_pids": [48009, 48010, 48011, 48012, 48013, 48014, 48016, 48017, 48025], "question_author": ""} {"qid": 1032, "query": "How can I effectively drop clues of corruption in my campaigns?", "score": 68, "views": 10869, "answer_pids": [18494, 18497, 18503, 18508, 18519, 18530, 18609], "question_author": "Anna"} {"qid": 1033, "query": "Can Ogre clerics use Purify Food and Drink on humanoid characters?", "score": 68, "views": 8850, "answer_pids": [71545, 71546, 71551, 71563, 71576, 71589], "question_author": "Karel"} {"qid": 1034, "query": "How to communicate to the players that an encounter can be solved also through diplomacy?", "score": 68, "views": 6076, "answer_pids": [1579, 1580, 1581, 1582, 1583, 1586, 1590, 1682, 1728, 2193, 13727], "question_author": ""} {"qid": 1035, "query": "Shoe-themed race or class features", "score": 68, "views": 3037, "answer_pids": [26217, 26218, 26232], "question_author": "Maciej Piechotka"} {"qid": 1036, "query": "Is it necessary to purchase all the D&D 5th edition books to have access to all player character options?", "score": 67, "views": 82029, "answer_pids": [30400, 49216, 71303], "question_author": "ahockley"} {"qid": 1037, "query": "Realistically, does Leomund\u2019s Tiny Hut allow for indefinite long rests in a dungeon?", "score": 67, "views": 67748, "answer_pids": [65533, 65536, 65545, 65547, 65548, 78829, 85341], "question_author": "Oscar Godson"} {"qid": 1038, "query": "What can I do about the guys being distracted by a new girl in D&D group?", "score": 67, "views": 25274, "answer_pids": [58468, 58474, 58490, 58525, 58534, 58547], "question_author": ""} {"qid": 1039, "query": "How do I encourage Drow players to not make Drizzt clones?", "score": 67, "views": 22736, "answer_pids": [60150, 60151, 60159, 60162, 60163, 60168, 60176, 60185, 60246, 60253], "question_author": "AJ Finch"} {"qid": 1040, "query": "How can I make Empty rooms interesting?", "score": 67, "views": 22275, "answer_pids": [28289, 28295, 28300, 28315, 28316, 28319, 28333, 28340, 28397, 28424], "question_author": ""} {"qid": 1041, "query": "Players playing weird characters breaking the immersion of my group", "score": 67, "views": 14596, "answer_pids": [21183, 21184, 21186, 21187, 21189, 21190, 21197, 21207, 21220, 21240, 21249, 21361], "question_author": "Numenetics"} {"qid": 1042, "query": "How can I politely handle a GM who doesnt appear to get how Dungeon World works?", "score": 67, "views": 13022, "answer_pids": [24202, 24204, 24213], "question_author": "Alan"} {"qid": 1043, "query": "As a GM, how can I stop killing my games?", "score": 67, "views": 13003, "answer_pids": [22305, 22306, 22308, 22310, 22320, 22329, 22330, 22331, 22332, 22361, 22371, 22402, 81840], "question_author": ""} {"qid": 1044, "query": "Is there a way to ask in game (i.e. in a non-meta way) what a characters class is?", "score": 67, "views": 11723, "answer_pids": [58784, 58785, 58786, 58806, 58807, 58816, 58823, 58834, 58847, 58872, 75796], "question_author": ""} {"qid": 1045, "query": "Abuse: Is it fair if its in-game?", "score": 67, "views": 9312, "answer_pids": [14891, 14892, 14893, 14894, 14909, 33493], "question_author": "Jack C Buel"} {"qid": 1046, "query": "What is the whiskey flask problem with Paradox?", "score": 67, "views": 9070, "answer_pids": [19642, 19644], "question_author": ""} {"qid": 1047, "query": "How could towns restrain a magic user?", "score": 67, "views": 8453, "answer_pids": [5163, 5165, 5167, 5169, 5171, 24665], "question_author": ""} {"qid": 1048, "query": "Origin of the term Splat Book", "score": 67, "views": 5953, "answer_pids": [24431], "question_author": "Aristos"} {"qid": 1049, "query": "Is Greatsword superior to Greataxe?", "score": 66, "views": 92958, "answer_pids": [57442, 57445, 57451, 57466, 72516], "question_author": ""} {"qid": 1050, "query": "What is the Stormwind Fallacy?", "score": 66, "views": 56986, "answer_pids": [12732], "question_author": "Aristos"} {"qid": 1051, "query": "How are Fighters Linear but Wizards Quadratic?", "score": 66, "views": 18600, "answer_pids": [34890, 34892, 34901, 34902, 34922, 34927], "question_author": ""} {"qid": 1052, "query": "The fastest way to remove bones from a man", "score": 66, "views": 18207, "answer_pids": [43822, 43823, 43826, 43830, 43835, 43841, 43851, 43869], "question_author": ""} {"qid": 1053, "query": "Could Vampires use touchscreen devices (iPhone, etc)?", "score": 66, "views": 10128, "answer_pids": [28148, 28149, 28150, 28159], "question_author": ""} {"qid": 1054, "query": "If I run a game of Paranoia, will knowing the rules lead to a worse experience if/when I participate as a player in the future?", "score": 66, "views": 10083, "answer_pids": [66461, 66464, 66466, 66467, 66596, 66643], "question_author": ""} {"qid": 1055, "query": "What is the background of Christian resistance to role-playing?", "score": 66, "views": 7873, "answer_pids": [20575, 20576, 20577, 20579, 20600], "question_author": ""} {"qid": 1056, "query": "Looking for solo (one person) RPGs - do they exist?", "score": 65, "views": 129191, "answer_pids": [456, 457, 472, 473, 475, 642, 644, 648, 673, 699, 710, 730, 830, 839, 1008, 1012, 1031, 1164, 1996, 2027, 3466, 3467, 4253, 8027, 9858, 12106, 13326, 13357, 14991, 15016, 18627], "question_author": "Jin"} {"qid": 1057, "query": "What defines a West Marches campaign?", "score": 65, "views": 96449, "answer_pids": [57530, 57531], "question_author": ""} {"qid": 1058, "query": "I dislike how players accurately place fireballs. Is there an alternative?", "score": 65, "views": 70668, "answer_pids": [37552, 37553, 37554, 37555, 37557, 37558, 37559, 37563, 37566, 37575, 37576, 37580, 37598, 84936], "question_author": "Jack C Buel"} {"qid": 1059, "query": "What are the rules for PC-PC seduction attempts?", "score": 65, "views": 17783, "answer_pids": [64178, 64179, 64183, 64185, 64198, 64199, 64231, 64274, 64312], "question_author": "Jake Pearson"} {"qid": 1060, "query": "Do Bugbears arms literally get longer when its their turn?", "score": 65, "views": 10549, "answer_pids": [68440, 68442], "question_author": "ahockley"} {"qid": 1061, "query": "As an unreliable player, how can I lessen the burden on my group?", "score": 65, "views": 2926, "answer_pids": [8355, 8356, 8362, 8367, 8396, 8429, 8637], "question_author": "Chuck Conway"} {"qid": 1062, "query": "Why do Warlocks only have spells up to 5th level? Whats the correct progression for their slots?", "score": 64, "views": 113054, "answer_pids": [23107], "question_author": ""} {"qid": 1063, "query": "Is the Saltwater Float represented in this question a good way to test for loaded dice?", "score": 64, "views": 39821, "answer_pids": [31363, 33657, 33660], "question_author": ""} {"qid": 1064, "query": "Players get angry if anything negative happens to them", "score": 64, "views": 22594, "answer_pids": [59460, 59463, 59467, 59470, 59471, 59586, 59590], "question_author": ""} {"qid": 1065, "query": "As a DM, how can I handle my Druid spying on everything with Wild shape as a spider?", "score": 64, "views": 16390, "answer_pids": [78968, 78971, 78972, 78990, 78991, 79001, 79003, 79017, 79018], "question_author": ""} {"qid": 1066, "query": "Is it OK for me to give my players characters punitive backstories when they dont provide their own?", "score": 64, "views": 15791, "answer_pids": [53136, 53137, 53141, 53149, 53157, 53160, 53164, 53225, 53237, 53252, 53253], "question_author": "davr"} {"qid": 1067, "query": "Overview of D&D retro-clones", "score": 64, "views": 14653, "answer_pids": [1095, 1096, 1101, 1110, 1115, 1336, 1637, 1961, 4543, 11914], "question_author": ""} {"qid": 1068, "query": "Death by leveling? The effects of 0 max HP and leveling with negative CON", "score": 64, "views": 14163, "answer_pids": [53154, 53162, 53170], "question_author": ""} {"qid": 1069, "query": "Has Christianity ever been given stats?", "score": 64, "views": 13686, "answer_pids": [34187, 34188, 34189, 34192, 34237, 35709, 71219], "question_author": "midas06"} {"qid": 1070, "query": "How can I engage players with melee monsters when players keep their distance?", "score": 64, "views": 12229, "answer_pids": [66166, 66167, 66169, 66170, 66172, 66174, 66175, 66180, 66184, 66186], "question_author": ""} {"qid": 1071, "query": "My players like to search everything. What do they find?", "score": 64, "views": 10658, "answer_pids": [71909, 71934, 71954, 71985, 71990, 71992], "question_author": ""} {"qid": 1072, "query": "Is it a violation of my player agency if another player is allowed to wish away memories from my character?", "score": 64, "views": 10462, "answer_pids": [66986, 66989, 66990, 66991, 66994, 66995, 66998], "question_author": ""} {"qid": 1073, "query": "How can I show-not-tell my players that they are The Bad Guys?", "score": 64, "views": 10257, "answer_pids": [60008, 60009, 60010, 60011, 60012, 60013, 60015, 60017, 60019, 60075, 60193, 63218], "question_author": ""} {"qid": 1074, "query": "How to play when you have little experience and speech difficulties?", "score": 64, "views": 5613, "answer_pids": [45731, 45758, 45769, 45864], "question_author": ""} {"qid": 1075, "query": "Can a party function without a Cleric or Paladin?", "score": 63, "views": 144480, "answer_pids": [23517, 23518, 23521, 23750, 29428, 29430, 33878, 33895, 40826, 52972, 52973, 64617], "question_author": ""} {"qid": 1076, "query": "What does minmax mean?", "score": 63, "views": 82846, "answer_pids": [31158, 31163, 31165, 31166, 31169, 31171, 31173, 31182, 31186, 31193, 31198, 31230], "question_author": ""} {"qid": 1077, "query": "Can the dungeon master have a player character?", "score": 63, "views": 33832, "answer_pids": [65630, 65631, 65632, 65634, 65635, 65636, 65640, 65712, 65725, 65731], "question_author": "Drake"} {"qid": 1078, "query": "Why do Sorcerers use Charisma?", "score": 63, "views": 33525, "answer_pids": [5254, 5255, 5256, 8510], "question_author": ""} {"qid": 1079, "query": "A player was bucking my style and I lost my temper. What can I do from here?", "score": 63, "views": 16536, "answer_pids": [35250, 35251, 35252, 35253, 35255, 35256, 35292], "question_author": "Don Kirkby"} {"qid": 1080, "query": "As a girl how can I roleplay a male character better?", "score": 63, "views": 15420, "answer_pids": [5583, 5587, 7214, 8818, 8839, 8877, 11298, 16282, 40028, 88980], "question_author": "tunaranch"} {"qid": 1081, "query": "Player has arachnophobia: what to use in place of spiders?", "score": 63, "views": 14475, "answer_pids": [39209, 39213, 39215, 39220, 39227, 39229, 39235, 39260, 39283, 39315], "question_author": ""} {"qid": 1082, "query": "What can I do to encourage my players to use their consumables?", "score": 63, "views": 8148, "answer_pids": [67825, 67826, 67827, 67828, 67839, 67843, 67859, 67872, 67898, 67929], "question_author": "HerbN"} {"qid": 1083, "query": "Aiming at specific body parts", "score": 62, "views": 75400, "answer_pids": [34671, 34673, 34674, 34676, 34681, 34685, 44510, 47023, 47908, 58057, 59194], "question_author": ""} {"qid": 1084, "query": "Weapon attacks compared with damaging cantrips?", "score": 62, "views": 35816, "answer_pids": [37990, 37998], "question_author": ""} {"qid": 1085, "query": "What options do I have for changing my characters sex?", "score": 62, "views": 25009, "answer_pids": [45852, 45853, 45856, 45860, 45863, 45876, 54166, 65805], "question_author": "Karl Bunyan"} {"qid": 1086, "query": "The PC claims to know an NPC they just met, how should I react as the DM?", "score": 62, "views": 17996, "answer_pids": [34739, 34741, 34746, 34747, 34759, 34760, 34766, 36952], "question_author": ""} {"qid": 1087, "query": "How can I handle a player who pre-plans arguments about my rulings on RAW?", "score": 62, "views": 13947, "answer_pids": [68136, 68138, 68149, 68150, 68209], "question_author": ""} {"qid": 1088, "query": "Does Burning Hands really require touching thumbs?", "score": 62, "views": 13693, "answer_pids": [23305, 23318, 23321, 23650], "question_author": ""} {"qid": 1089, "query": "Would forcing armour on a wizard using an illusion spell and Illusory Reality harm them?", "score": 62, "views": 5773, "answer_pids": [60445, 60446, 60462, 60463], "question_author": "Drew Hoskins"} {"qid": 1090, "query": "How much damage does Great Weapon Fighting add on average?", "score": 61, "views": 131434, "answer_pids": [23612, 23623, 24196, 29626, 46349, 51626], "question_author": ""} {"qid": 1091, "query": "What is railroading, and what are its pros and cons?", "score": 61, "views": 40237, "answer_pids": [410, 436, 444, 1351, 1357, 1714, 8884, 17529], "question_author": ""} {"qid": 1092, "query": "What are the differences between Holmes, Moldvay, and Mentzer D&D?", "score": 61, "views": 33436, "answer_pids": [1331, 1334, 1338, 1350, 1372, 49719, 65580], "question_author": "Numenetics"} {"qid": 1093, "query": "What does the abbreviation \u201cRAW\u201d mean?", "score": 61, "views": 21887, "answer_pids": [3994, 77212], "question_author": ""} {"qid": 1094, "query": "Does forcing a Vampire indoors count as an invitation?", "score": 61, "views": 14947, "answer_pids": [53400, 53407, 53409, 53419, 53424, 53425], "question_author": ""} {"qid": 1095, "query": "How do I handle a group that does not understand the assumption rule?", "score": 61, "views": 12768, "answer_pids": [11197, 11198, 11199, 11201, 11212, 11243, 11979, 17573, 22921], "question_author": "JSB\u0571\u0578\u0563\u0579"} {"qid": 1096, "query": "How can I roleplay a character more manipulative than myself?", "score": 61, "views": 12036, "answer_pids": [19954, 19958, 19963, 19965, 19966, 19967, 19969, 19971, 19989], "question_author": ""} {"qid": 1097, "query": "What do you do in Dungeon World when someone wants to perform an action they dont have a move for?", "score": 61, "views": 8106, "answer_pids": [10067, 10068, 10073, 12420], "question_author": ""} {"qid": 1098, "query": "How to get my Paranoia players to stop cooperating and start killing each other?", "score": 61, "views": 7513, "answer_pids": [16157, 16162, 16170, 16175, 59718, 59743], "question_author": "Mark Rogers"} {"qid": 1099, "query": "Can someone explain what the Proficiency Bonus is in D&D 5e/Next exactly?", "score": 60, "views": 915030, "answer_pids": [17759, 17763, 34280, 34281, 80314], "question_author": ""} {"qid": 1100, "query": "How does one craft Potions of Healing?", "score": 60, "views": 370759, "answer_pids": [23030, 26245, 28075, 42660, 60913], "question_author": "mmr"} {"qid": 1101, "query": "What spells are available to resurrect characters?", "score": 60, "views": 296035, "answer_pids": [57172, 85617], "question_author": ""} {"qid": 1102, "query": "Does casting a spell from an item allow you to apply class abilities that are used when casting a spell?", "score": 60, "views": 37782, "answer_pids": [25137, 25154, 29553, 50350], "question_author": ""} {"qid": 1103, "query": "What is GM/DM Fiat?", "score": 60, "views": 27668, "answer_pids": [17723, 17724, 17725, 17730, 17740, 68910], "question_author": "jpt"} {"qid": 1104, "query": "How do I learn to become a good GM?", "score": 60, "views": 18883, "answer_pids": [12978, 12986, 13136, 14192, 17356, 19550, 19604], "question_author": "Adam Franco"} {"qid": 1105, "query": "Are Imps incapable of speech or is my DM messing with me?", "score": 60, "views": 17007, "answer_pids": [52100, 52101, 52106, 52112], "question_author": ""} {"qid": 1106, "query": "What happens when initiative allows a player to act before the player that started the combat?", "score": 60, "views": 15027, "answer_pids": [34698, 34700, 34708, 34715, 34755, 80919, 88010], "question_author": ""} {"qid": 1107, "query": "What is a spell slot in-lore, and how does it justify the limits on casting spells?", "score": 60, "views": 12696, "answer_pids": [62435, 62439, 62443, 81251], "question_author": "tttppp"} {"qid": 1108, "query": "How can I tell if I am a problem player?", "score": 60, "views": 9865, "answer_pids": [66475, 66477, 66511, 66516, 66517, 66520], "question_author": "Eight Days of Malaise"} {"qid": 1109, "query": "What RPG concepts does rules as written encompass?", "score": 60, "views": 7038, "answer_pids": [49286, 49287, 49288, 49289, 49290], "question_author": "Salsa"} {"qid": 1110, "query": "Can wizards really cast spells in armor?", "score": 59, "views": 184193, "answer_pids": [23248, 23254], "question_author": "Karel"} {"qid": 1111, "query": "Are there ways to concentrate on more than one spell at a time?", "score": 59, "views": 131734, "answer_pids": [23697, 23723, 23740, 29525, 46368, 53006, 66487, 66753], "question_author": ""} {"qid": 1112, "query": "Does Power Word Kill kill druids in wild-shape?", "score": 59, "views": 26561, "answer_pids": [32407], "question_author": "Stephen Furlani"} {"qid": 1113, "query": "Should I boost a new players character level to match the rest of the group?", "score": 59, "views": 24671, "answer_pids": [41702, 41703, 41704, 41707, 41715, 41717, 41723, 41732, 41746, 41790, 41806], "question_author": "Iain M Norman"} {"qid": 1114, "query": "Players argue and dont accept rulings to the point of arguments", "score": 59, "views": 18183, "answer_pids": [56589, 56590, 56592, 56593, 56595, 56597, 56598, 56601, 56621, 56636, 56672], "question_author": ""} {"qid": 1115, "query": "As the Dungeon Master, how do I handle a player that insists on a specific class when I already know that choice will cause issues?", "score": 59, "views": 15976, "answer_pids": [72241, 72242, 72243, 72244, 72245, 72246, 72247, 72266, 72295, 72308], "question_author": ""} {"qid": 1116, "query": "How to handle a rules-lawyer player?", "score": 59, "views": 15161, "answer_pids": [72, 77, 78, 82, 94, 108, 114, 267, 1024, 1030, 1068, 1430, 1431, 1441, 16882, 66363, 66395, 68678], "question_author": ""} {"qid": 1117, "query": "GMing for a hyper optimized character", "score": 59, "views": 12065, "answer_pids": [59160, 59161, 59162, 59164, 59190, 59200, 59218], "question_author": ""} {"qid": 1118, "query": "For seasoned 3.X players and DMs, what is the whats new summary for 5e?", "score": 59, "views": 10380, "answer_pids": [22856, 22857, 22897], "question_author": ""} {"qid": 1119, "query": "How do I allow the Rogue opportunities to be stealthy with a Druid in the party?", "score": 59, "views": 9251, "answer_pids": [57877, 57878, 57879, 57880, 57882, 57883, 57885, 57906, 61475, 61477], "question_author": "Slick23"} {"qid": 1120, "query": "How do I keep presenting progressively more challenging encounters to my PCs without making them think that the world is gaining Levels as they are?", "score": 59, "views": 9098, "answer_pids": [57181, 57182, 57184, 57188, 57195, 57201], "question_author": "Slick23"} {"qid": 1121, "query": "Need out-of-game solution for spell choice paralysis", "score": 59, "views": 9081, "answer_pids": [54012, 54013, 54014, 54021, 54022, 54025, 54037, 54038, 54039, 54040, 54048, 54074, 54094, 54128], "question_author": "davr"} {"qid": 1122, "query": "Are babies of evil humanoid species inherently evil?", "score": 59, "views": 8936, "answer_pids": [68379, 68382, 68398, 68403], "question_author": ""} {"qid": 1123, "query": "What to do when a player character does something that seems suicidal?", "score": 59, "views": 8272, "answer_pids": [10085, 10086, 10087, 10106, 11083, 29365, 29404, 78016, 78022], "question_author": "Anna"} {"qid": 1124, "query": "How can I get players to realize that their enemies arent evil?", "score": 59, "views": 7392, "answer_pids": [21090, 21095, 21096, 21102, 21115, 21120, 21121, 21122, 21123, 21124, 21125, 21127, 21137, 21208], "question_author": "Mark Rogers"} {"qid": 1125, "query": "Can a creature take turns as normal if they are inside an Antimagic Field while another creature casts Time Stop?", "score": 59, "views": 4720, "answer_pids": [79467, 79493, 79512, 79522], "question_author": "vanden"} {"qid": 1126, "query": "Can you milk an Ivory Goat?", "score": 59, "views": 4631, "answer_pids": [54556, 54557], "question_author": ""} {"qid": 1127, "query": "Does D&D 5e have a rule for character knowledge about monsters?", "score": 58, "views": 101876, "answer_pids": [23302, 27914, 46590, 46777, 54534], "question_author": ""} {"qid": 1128, "query": "How to randomly generate a village or town for old school D&D game?", "score": 58, "views": 32447, "answer_pids": [964, 965, 976, 980, 38047], "question_author": "Adam Dray"} {"qid": 1129, "query": "Help, a player wont stop hitting on every female NPC!", "score": 58, "views": 19612, "answer_pids": [38311, 38312, 38318, 38320, 38321, 38323, 38325, 38336, 38338, 38345, 38354, 38414], "question_author": "cmcculloh"} {"qid": 1130, "query": "Can the DM forbid my character from using certain weapons because he doesnt like them?", "score": 58, "views": 18221, "answer_pids": [32358, 32359, 32360, 32371, 32384, 32406, 32413, 32432], "question_author": ""} {"qid": 1131, "query": "Can a raging barbarian carry live rabbits to kill them, in order to keep his rage going?", "score": 58, "views": 13323, "answer_pids": [59715, 59716, 59725, 59802], "question_author": ""} {"qid": 1132, "query": "How to deal with anti-authoritarian player characters?", "score": 58, "views": 12654, "answer_pids": [30206, 30207, 30208, 30210, 30212, 30217, 30221, 30222, 30225, 30227, 30232, 30233, 30249, 30253, 30272, 30292, 30342, 30364], "question_author": ""} {"qid": 1133, "query": "How do I rein in a player who talks over my descriptions?", "score": 58, "views": 11378, "answer_pids": [50271, 50273, 50275, 50276, 50280, 50310, 50319], "question_author": "cmcculloh"} {"qid": 1134, "query": "Is there \u201cflavor text\u201d in D&D 5e spells?", "score": 58, "views": 8432, "answer_pids": [36781, 36786, 36796], "question_author": ""} {"qid": 1135, "query": "I deciphered the meaning of a cryptic language out-of-game: should my character know what the meaning is?", "score": 58, "views": 8389, "answer_pids": [66535, 66553, 66559, 66567, 66568, 66579], "question_author": ""} {"qid": 1136, "query": "What happens to a Devil when its alignment is forcibly changed?", "score": 58, "views": 8015, "answer_pids": [48911, 85496], "question_author": "Slick23"} {"qid": 1137, "query": "Handling encounter elements that would realistically cause instant death", "score": 58, "views": 6496, "answer_pids": [66901, 66907, 66908, 66910, 66922, 66931], "question_author": ""} {"qid": 1138, "query": "Is there any known limit for how many dice RPG players are comfortable adding up?", "score": 58, "views": 6020, "answer_pids": [41984, 41993, 42834], "question_author": "cmcculloh"} {"qid": 1139, "query": "How do I make weather an important part of the game?", "score": 58, "views": 5843, "answer_pids": [6807, 6808, 6809, 6811, 6822, 7754, 8798, 9873, 10190, 10199, 10211], "question_author": ""} {"qid": 1140, "query": "Do 10 second turns/60 second rounds make spellcasters horrible?", "score": 58, "views": 5367, "answer_pids": [82139, 82140, 82218], "question_author": ""} {"qid": 1141, "query": "How does extra damage work for critical hits?", "score": 57, "views": 371454, "answer_pids": [28259, 50486, 50488], "question_author": ""} {"qid": 1142, "query": "Is it better to take the array and be Joe Average, or to roll for the odds of getting on average better scores?", "score": 57, "views": 221587, "answer_pids": [24220, 24221, 24222, 24225, 24247, 24248, 24290, 26341, 26345, 28210, 28217, 31055, 34594], "question_author": "daniel"} {"qid": 1143, "query": "How do I kill a 20th level raging Zealot Barbarian?", "score": 57, "views": 76759, "answer_pids": [61689, 61726, 64313, 69897, 69901, 84451], "question_author": "edelwater"} {"qid": 1144, "query": "How do I keep spellcasters from casting while in jail?", "score": 57, "views": 60729, "answer_pids": [27053, 27054, 27057, 27058, 27061, 27062, 27075, 28618, 68517, 68524, 69044], "question_author": "Adam Dray"} {"qid": 1145, "query": "With the lifespan of elves, would you ever encounter a low-level elf?", "score": 57, "views": 53929, "answer_pids": [26785, 26786, 26788, 26795, 26798, 26809, 26810, 26815, 26820, 26825, 26833, 36822, 49867, 86878, 88356], "question_author": "edelwater"} {"qid": 1146, "query": "The power level of the Sword of Sharpness doesnt justify its very rare rating - am I missing something?", "score": 57, "views": 32957, "answer_pids": [48650, 48651, 48665, 80712], "question_author": ""} {"qid": 1147, "query": "How can I tell the DM that I wasnt having fun and that I fault him for it?", "score": 57, "views": 32285, "answer_pids": [64421, 64422, 64423, 64424, 64425, 64426, 64427, 64428, 64430, 64452, 64478, 88247], "question_author": "LittleBobbyTables - Au Revoir"} {"qid": 1148, "query": "What is Sandbox play?", "score": 57, "views": 25889, "answer_pids": [393, 416, 420, 426, 8564, 9923], "question_author": ""} {"qid": 1149, "query": "Can characters reach Mach 1 and higher?", "score": 57, "views": 24733, "answer_pids": [9251, 9253, 9271, 14993, 15024, 15953, 28339], "question_author": ""} {"qid": 1150, "query": "Whats the point of having a locked door if the players can just destroy it?", "score": 57, "views": 24330, "answer_pids": [65907, 65908, 65909, 65923, 65927, 65931, 65934, 65938], "question_author": ""} {"qid": 1151, "query": "How do you tell if a D&D book is 3.0 or 3.5?", "score": 57, "views": 22003, "answer_pids": [687, 690, 703, 2792, 2816, 5890, 20152], "question_author": "Slick23"} {"qid": 1152, "query": "How to create a dungeon based on a realistic tomb, but still enjoyable to explore?", "score": 57, "views": 16302, "answer_pids": [24482, 24484, 24485, 24487, 24488, 24491, 24514, 24525, 24531, 24543, 24582, 24613], "question_author": ""} {"qid": 1153, "query": "My players went from 100% murderous cretins to 100% nonviolent diplomats; how can I achieve a middle ground?", "score": 57, "views": 15889, "answer_pids": [43644, 43645, 43646, 43656, 43661, 43664, 43678], "question_author": "Drew Hoskins"} {"qid": 1154, "query": "Dealing with fearless players", "score": 57, "views": 14079, "answer_pids": [20367, 20368, 20374, 20376, 20384, 20398, 20399, 20404, 20405, 20406, 20450, 20451, 20708, 20710, 37080], "question_author": ""} {"qid": 1155, "query": "How to get a new group into the spirit of the game Paranoia?", "score": 57, "views": 12616, "answer_pids": [4895, 4896, 4898, 4903, 4907, 4908, 5745, 6269, 26664, 70912], "question_author": ""} {"qid": 1156, "query": "Since advantage and disadvantage caused in a heavily obscured area cancel out, what effect does it have in combat?", "score": 57, "views": 12327, "answer_pids": [47070, 47071, 47075, 47076, 47078, 47084], "question_author": ""} {"qid": 1157, "query": "How can I play dumb?", "score": 57, "views": 10007, "answer_pids": [7563, 7564, 7565, 7566, 7567, 7569, 7573, 7575, 7578, 7589, 7591, 7622, 11299, 16584, 87284, 87309], "question_author": "Slick23"} {"qid": 1158, "query": "I unknowingly guessed what will happen next in the module and its angering the DM", "score": 57, "views": 9829, "answer_pids": [81375, 81376, 81378, 81379, 81381, 81387, 81402, 81404, 81405], "question_author": "chills42"} {"qid": 1159, "query": "Are all players supposed to be able to see each others character sheets?", "score": 57, "views": 9261, "answer_pids": [68337, 68338, 68340, 68341, 68342, 68355, 68412], "question_author": "HerbN"} {"qid": 1160, "query": "As a DM, how to avoid unconscious metagaming when dealing with a high AC character?", "score": 57, "views": 9056, "answer_pids": [72315, 72316, 72319, 72323, 72328, 72365, 72393], "question_author": ""} {"qid": 1161, "query": "Is a creature in the area of a Wall of Water spell trapped in ice when hit by the Ray of Frost cantrip?", "score": 57, "views": 8761, "answer_pids": [58963, 58964, 58965], "question_author": ""} {"qid": 1162, "query": "How can I DM a character with more social skills than me?", "score": 57, "views": 7237, "answer_pids": [27664, 27665, 27669, 27673, 27734], "question_author": "Kempeth"} {"qid": 1163, "query": "What to do if a player expected much more than DM gave?", "score": 57, "views": 7124, "answer_pids": [82181, 82182, 82187, 82188, 82195, 82215, 82260], "question_author": ""} {"qid": 1164, "query": "Does dealing 0 damage to a concentrating spellcaster require a saving throw?", "score": 57, "views": 6566, "answer_pids": [53307, 53310, 79379], "question_author": "Tony Meyer"} {"qid": 1165, "query": "What approaches are there to lessen or eliminate reliance upon dice in an RPG?", "score": 57, "views": 3793, "answer_pids": [169, 174, 177, 186, 194, 203, 305, 312, 329, 450, 721, 722, 723, 732, 1615, 1639, 25288], "question_author": ""} {"qid": 1166, "query": "How can I make my game more noir as a GM?", "score": 57, "views": 3448, "answer_pids": [307, 313, 829, 2721, 3055, 3056, 14232], "question_author": ""} {"qid": 1167, "query": "How can I persuade my DM to play my warlocks patron in a way that respects my characters concept?", "score": 56, "views": 26282, "answer_pids": [36330, 36333, 36351, 36359, 36374, 36401], "question_author": "Tony Meyer"} {"qid": 1168, "query": "How can we kick our novice roleplayer out of the group for being a poor match, without alienating them from the hobby?", "score": 56, "views": 15354, "answer_pids": [60306, 60311, 60341, 60343], "question_author": ""} {"qid": 1169, "query": "How to challenge a pacifist party", "score": 56, "views": 14072, "answer_pids": [51702, 51703, 51704, 51711, 51718, 51720, 51728, 51731, 51749, 51772, 51787, 54455, 63276], "question_author": "user259"} {"qid": 1170, "query": "Player wants to steal items from previous character", "score": 56, "views": 13563, "answer_pids": [57943, 57944, 57947, 57949, 57956, 57971, 57972, 58010, 58030], "question_author": ""} {"qid": 1171, "query": "What is the purpose of old-school D&D class level limitations?", "score": 56, "views": 12926, "answer_pids": [5142, 5146, 5166], "question_author": null} {"qid": 1172, "query": "How can I politely ask a player to leave my game?", "score": 56, "views": 9921, "answer_pids": [16996, 17003, 17486], "question_author": "Mark"} {"qid": 1173, "query": "What to do when players bypass plot hooks", "score": 56, "views": 9133, "answer_pids": [29169, 29170, 29171, 29173, 29180, 29184, 29186, 29193], "question_author": "Tobiasopdenbrouw"} {"qid": 1174, "query": "Can Raise Dead revive a character with zero maximum Hit Points?", "score": 56, "views": 8294, "answer_pids": [35570, 35571, 35573, 35575], "question_author": ""} {"qid": 1175, "query": "Techniques for allowing character defeat without character death", "score": 56, "views": 6957, "answer_pids": [4507, 4508, 4509, 4510, 4511, 4516, 4517, 4519, 4818], "question_author": ""} {"qid": 1176, "query": "How do I deal with PCs who use kidnap and torture as an investigative technique?", "score": 56, "views": 6230, "answer_pids": [8952, 8953, 8954, 8955, 8957, 8960, 8967, 8968, 25059], "question_author": ""} {"qid": 1177, "query": "How do I help my players not get caught up on smaller plot points?", "score": 56, "views": 2084, "answer_pids": [3552, 3553, 3554, 3556, 3557, 3558, 3560, 3561, 3562, 3563, 3564, 3581, 16995], "question_author": null} {"qid": 1178, "query": "Can the arcane/druidic focus staff double as quarterstaff?", "score": 55, "views": 81872, "answer_pids": [23360, 23423, 23852, 23853, 78058], "question_author": ""} {"qid": 1179, "query": "Can one enter Leomunds Tiny Hut from below?", "score": 55, "views": 24237, "answer_pids": [35140, 35149, 35156, 35157, 35179], "question_author": ""} {"qid": 1180, "query": "How to not let the Identify spell spoil everything?", "score": 55, "views": 22846, "answer_pids": [67840, 67841, 67844, 67847, 67858, 67867, 67873, 67906], "question_author": ""} {"qid": 1181, "query": "What is the etymology of the term Gish?", "score": 55, "views": 13543, "answer_pids": [17167, 17168, 76892], "question_author": ""} {"qid": 1182, "query": "Whats the best way to make time spent travelling more interesting?", "score": 55, "views": 11806, "answer_pids": [26858, 26862, 26874, 26904, 26913], "question_author": ""} {"qid": 1183, "query": "Can the DM change his/her mind after a player has rolled?", "score": 55, "views": 9400, "answer_pids": [60084, 60085, 60086, 60091, 60126], "question_author": "7wp"} {"qid": 1184, "query": "How to handle players who make unmotivated characters", "score": 55, "views": 8624, "answer_pids": [36291, 36293, 36297, 36298, 36308, 36311, 36314, 36596, 36598], "question_author": "Don Kirkby"} {"qid": 1185, "query": "Can a GM prohibit players from using external reference materials (like the PHB) during play?", "score": 55, "views": 8186, "answer_pids": [41526, 41528, 41533, 41543, 41551, 41569], "question_author": "MatthewMartin"} {"qid": 1186, "query": "What makes a character/creature/power/ability/etc. over-powered?", "score": 55, "views": 6005, "answer_pids": [12686, 12688, 12690, 12691, 12692, 12702, 12719], "question_author": "vanden"} {"qid": 1187, "query": "How can I check whether an extremely adult/mature plot idea would be accepted by players without revealing any details of the idea itself?", "score": 55, "views": 5807, "answer_pids": [13527, 13528, 13529, 13530, 13546, 13548], "question_author": ""} {"qid": 1188, "query": "GMs and MCs, When do you say, No, to your players?", "score": 55, "views": 3299, "answer_pids": [12421, 12429, 12506, 17165], "question_author": "Aristos"} {"qid": 1189, "query": "What are the essential features of a successful evil campaign?", "score": 54, "views": 43273, "answer_pids": [13631, 13632, 13633, 13634, 13635, 13639, 13648, 13695], "question_author": ""} {"qid": 1190, "query": "Can a multiclass character with 1 level of wizard copy any wizard spell they find into their spellbook?", "score": 54, "views": 40301, "answer_pids": [25323, 29449, 45243, 60807], "question_author": ""} {"qid": 1191, "query": "My DM has removed heavy armour from the game. How can I make a Paladin work or convince him not to do this?", "score": 54, "views": 22116, "answer_pids": [53689, 53690, 53697, 53701, 53704, 53716, 53722, 53724, 53751, 53756, 53769], "question_author": "Mark Rogers"} {"qid": 1192, "query": "How to enjoyably include a 4\u20135 year old in a D&D 5e game", "score": 54, "views": 19012, "answer_pids": [23570, 23571, 23573, 23611, 23614, 23691, 31393, 56768], "question_author": ""} {"qid": 1193, "query": "My 5e character isnt very fun. How do I kill them off?", "score": 54, "views": 17414, "answer_pids": [28983, 28985, 28987, 28990, 29029, 39397, 47941], "question_author": "LittleBobbyTables - Au Revoir"} {"qid": 1194, "query": "How does D&D Vancian magic make sense in-game?", "score": 54, "views": 17005, "answer_pids": [7843, 7844, 7845, 7853, 11966, 11973, 18236], "question_author": ""} {"qid": 1195, "query": "Are there any rules for a Cleric to change his faith?", "score": 54, "views": 15599, "answer_pids": [12138, 14910, 14912], "question_author": null} {"qid": 1196, "query": "What software is available for virtual table tops?", "score": 54, "views": 15494, "answer_pids": [2100, 2101, 2102, 2104, 2115, 2145, 2146, 2149, 2167, 2201, 2244, 5380, 9601], "question_author": ""} {"qid": 1197, "query": "Would striking someone at 0 hit points with an entire Magic Missile spell automatically kill them?", "score": 54, "views": 14079, "answer_pids": [34848, 52489, 53391], "question_author": ""} {"qid": 1198, "query": "My party bypassed a combat encounter. Should they still get XP?", "score": 54, "views": 13755, "answer_pids": [35060, 35061, 35066, 35069, 35112], "question_author": "Toon Krijthe"} {"qid": 1199, "query": "How to handle loot disputes as the DM?", "score": 54, "views": 13128, "answer_pids": [59000, 59001, 59006, 59011, 59014, 59024, 59025, 59042, 59056, 59065], "question_author": ""} {"qid": 1200, "query": "How can I end combat quickly when the outcome is inevitable?", "score": 54, "views": 12083, "answer_pids": [71108, 71109, 71110, 71116, 71127, 71134, 71144, 71178, 77994], "question_author": ""} {"qid": 1201, "query": "How do I deal with being envious of my own players?", "score": 54, "views": 9682, "answer_pids": [68182, 68183, 68200, 68208, 68271, 72044], "question_author": ""} {"qid": 1202, "query": "What is a social contract?", "score": 54, "views": 9453, "answer_pids": [12883, 12884, 12886, 17132], "question_author": ""} {"qid": 1203, "query": "How can I play a character who is scared without disrupting the gameplay?", "score": 54, "views": 9046, "answer_pids": [44435, 44436, 44439, 44442, 44455, 44457], "question_author": ""} {"qid": 1204, "query": "Is it possible to produce a bowl-shaped probability curve with dice rolls?", "score": 54, "views": 8987, "answer_pids": [10144, 10145, 10146, 10159, 10169, 10170, 11205], "question_author": "Maniero"} {"qid": 1205, "query": "How do you deal with metagaming based on die results?", "score": 54, "views": 8895, "answer_pids": [18244, 18245, 18246, 18250, 18256, 29799, 29813, 38444, 40290, 86870], "question_author": "Maciej Piechotka"} {"qid": 1206, "query": "GMing for [dis]abled players, specifically blind players?", "score": 54, "views": 6913, "answer_pids": [4165, 4166, 4168, 4169, 4267], "question_author": "Pr\u00e4riewolf"} {"qid": 1207, "query": "How do I prevent myself from freezing up during a game Im DMing?", "score": 54, "views": 5243, "answer_pids": [84556, 84565, 84578, 84586, 84590], "question_author": "Reinstate Monica - Goodbye SE"} {"qid": 1208, "query": "How to approximate rolls for potions of healing using only d6s?", "score": 54, "views": 5124, "answer_pids": [68095, 68103, 68106, 68108], "question_author": ""} {"qid": 1209, "query": "What characterizes a Modern role playing game or gaming system?", "score": 54, "views": 4605, "answer_pids": [4883, 4885, 4887, 4888, 4889, 4893, 4901], "question_author": "DavRob60"} {"qid": 1210, "query": "Can I Ready an action to do something Im not currently capable of doing?", "score": 54, "views": 4516, "answer_pids": [65135, 65137, 65138], "question_author": "Mark Green"} {"qid": 1211, "query": "How do I get the PCs to stop focusing on a red herring?", "score": 54, "views": 3957, "answer_pids": [4917, 4918, 4919, 4920, 4921, 4923, 4925, 4926, 4927, 4929, 4934, 4937, 4948, 5054], "question_author": null} {"qid": 1212, "query": "If someone is Out of Ammo, can they still shoot until compelled?", "score": 54, "views": 2247, "answer_pids": [13720, 13722, 19117], "question_author": ""} {"qid": 1213, "query": "What is the proper way to style references to D&D game elements?", "score": 54, "views": 1756, "answer_pids": [63416], "question_author": "Stu Pegg"} {"qid": 1214, "query": "When can I make opportunity attacks?", "score": 53, "views": 188284, "answer_pids": [22064], "question_author": ""} {"qid": 1215, "query": "What are the downsides to multi-classing?", "score": 53, "views": 63269, "answer_pids": [42647, 42650, 42653, 42680, 42694, 69290], "question_author": ""} {"qid": 1216, "query": "How is Polymorph broken?", "score": 53, "views": 28495, "answer_pids": [11703, 11706, 11707, 11714, 11767, 22133], "question_author": "Apreche"} {"qid": 1217, "query": "What can I use in place of Fudge Dice?", "score": 53, "views": 21783, "answer_pids": [12, 13, 15, 17, 18, 99, 244, 921, 1080, 1081, 1908, 1909, 2203, 3424, 13339, 13368, 14475, 14493, 16182, 16183, 16240, 16734, 34083], "question_author": "Apreche"} {"qid": 1218, "query": "What alternatives are there to replace orcs, kobolds, and hobgoblins in low-level encounters?", "score": 53, "views": 21739, "answer_pids": [293, 295, 296, 297, 298, 300, 303, 331, 345, 354, 454, 1148, 1150, 2330, 2373, 2377, 3488, 4942, 8363, 8365, 8381, 13984], "question_author": "Apreche"} {"qid": 1219, "query": "Is there a OGL or GSL license for D&D 5e?", "score": 53, "views": 19043, "answer_pids": [22416, 31895, 34788], "question_author": "Apreche"} {"qid": 1220, "query": "How can I keep a player from shutting down after a streak of bad rolls?", "score": 53, "views": 18934, "answer_pids": [48878, 48879, 48881, 48883, 48884, 48914, 48928, 48937, 48946, 48962, 48975, 48987], "question_author": "Davy Landman"} {"qid": 1221, "query": "How to break the Death Loop resulting from unfortunate initiative order", "score": 53, "views": 16607, "answer_pids": [63973, 63974, 63975, 63977, 63978, 63981, 63982, 63985, 63987, 63990, 64002, 64005], "question_author": "mafu"} {"qid": 1222, "query": "How can I reward a character that doesnt want magic items?", "score": 53, "views": 15269, "answer_pids": [7561, 7562, 7570, 7571, 7580, 8740, 8741, 8742, 12698, 24317], "question_author": "GWLlosa"} {"qid": 1223, "query": "How should I handle players who ignore the session zero agreement?", "score": 53, "views": 14084, "answer_pids": [67961, 67963, 67964, 67980, 67990], "question_author": ""} {"qid": 1224, "query": "Medieval problems in a magical world", "score": 53, "views": 12363, "answer_pids": [5675, 5676, 5678, 5679, 5680, 5681, 5682, 5683, 5684, 7370], "question_author": "idan315"} {"qid": 1225, "query": "How do I make interrogations interesting?", "score": 53, "views": 11635, "answer_pids": [26630, 26631, 26632, 26634, 26652, 26654], "question_author": ""} {"qid": 1226, "query": "How do I make an unwinnable fight fun?", "score": 53, "views": 11171, "answer_pids": [26819, 26821, 26827, 26842, 26847, 26867, 26869, 26873, 26882], "question_author": ""} {"qid": 1227, "query": "What inspired the D&D version of the Rakshasa?", "score": 53, "views": 10178, "answer_pids": [13585, 15670, 18266, 82422], "question_author": "Slick23"} {"qid": 1228, "query": "How do I know if an ability is magical?", "score": 53, "views": 9629, "answer_pids": [54118], "question_author": "David Sykes"} {"qid": 1229, "query": "How should I deal with a difficult group and a DM that doesnt help?", "score": 53, "views": 8949, "answer_pids": [41531, 41532, 41540, 41548, 41549], "question_author": ""} {"qid": 1230, "query": "Problems with minecraft happy druids", "score": 53, "views": 8168, "answer_pids": [36955, 36956, 36957, 36961, 36962, 36965, 36967, 36970, 36971, 36980, 36982, 36983, 36992, 36997, 37018, 37060, 37074, 37081], "question_author": "Peter McEvoy"} {"qid": 1231, "query": "Preventing saturation in a horror campaign", "score": 53, "views": 8103, "answer_pids": [10254, 10255, 10256, 10263, 10269, 10293], "question_author": ""} {"qid": 1232, "query": "Make the player roll even if there is no chance of success?", "score": 53, "views": 7035, "answer_pids": [36003, 36004, 36005, 36006, 36007, 36025, 36032, 36040, 36099], "question_author": "tttppp"} {"qid": 1233, "query": "Dealing with a Repeatedly Cheating Friend", "score": 53, "views": 6852, "answer_pids": [87775, 87776, 87782, 87789], "question_author": "Craig Nicholson"} {"qid": 1234, "query": "Is the same page tool overkill for brand new players?", "score": 53, "views": 4575, "answer_pids": [28545, 28547, 28548, 28549, 28553], "question_author": ""} {"qid": 1235, "query": "Running games for a constantly unreliable group", "score": 53, "views": 4511, "answer_pids": [8309, 8312, 8313, 8315, 8316, 8330, 8335, 8392, 8545], "question_author": ""} {"qid": 1236, "query": "How can I quickly get a player back into a session after their character dies?", "score": 53, "views": 3085, "answer_pids": [8766, 8767, 8769, 8770, 8790], "question_author": ""} {"qid": 1237, "query": "How much should NPCs charge for spells cast as services to PCs?", "score": 52, "views": 121602, "answer_pids": [52021], "question_author": "Marcin"} {"qid": 1238, "query": "Optimizing a D&D 3.5 Monk", "score": 52, "views": 121121, "answer_pids": [381, 1023, 1057, 1073, 11339, 11994, 12597, 14815, 15471, 15585, 24415], "question_author": ""} {"qid": 1239, "query": "Can a zero-HP, unconscious and stable character be woken up prematurely?", "score": 52, "views": 101446, "answer_pids": [27412, 27423, 27450, 40109, 43420, 43435], "question_author": ""} {"qid": 1240, "query": "Does passive perception supersede active perception?", "score": 52, "views": 54512, "answer_pids": [24217, 24218, 24219, 53249], "question_author": ""} {"qid": 1241, "query": "What do I know, when deciding whether to cast Counterspell?", "score": 52, "views": 35535, "answer_pids": [23438, 23460, 24046, 38554, 77078], "question_author": "Eight Days of Malaise"} {"qid": 1242, "query": "Is a fumble on a natural 1 an official rule?", "score": 52, "views": 29987, "answer_pids": [2, 3, 11, 54, 5576, 5579, 8061, 8067, 9537], "question_author": "Mark Rogers"} {"qid": 1243, "query": "Can you cast banishment on yourself?", "score": 52, "views": 29388, "answer_pids": [23761], "question_author": "Naseer"} {"qid": 1244, "query": "Does a natural 20 on the attack roll still automatically hit if the target is wearing adamantine armor?", "score": 52, "views": 26309, "answer_pids": [63684], "question_author": ""} {"qid": 1245, "query": "What games are out there that could be played in a single night, with no prep?", "score": 52, "views": 19956, "answer_pids": [2418, 2420, 2421, 2422, 2436, 2437, 2440, 2445, 2446, 2452, 2463, 2493, 2569, 2573, 2582, 2858, 3272, 3279, 3280, 3494, 4025, 4252, 4567, 7282, 14877, 17644], "question_author": ""} {"qid": 1246, "query": "My own Party are ruining the game for me. What should I do?", "score": 52, "views": 19211, "answer_pids": [78187, 78191, 78196, 78199, 78205, 78226, 78237, 78272], "question_author": "Jorge Castro"} {"qid": 1247, "query": "Is there a minimum damage rule?", "score": 52, "views": 17625, "answer_pids": [34962, 34963, 84845, 84846], "question_author": ""} {"qid": 1248, "query": "How much damage can a druid Wild Shaped into a spider take without dying instantly?", "score": 52, "views": 16370, "answer_pids": [27696, 27726, 27732, 78820, 78900], "question_author": "Mark Rogers"} {"qid": 1249, "query": "Player is obsessed with shaving my wookiee", "score": 52, "views": 16315, "answer_pids": [62006, 62013, 62016, 62017, 62018], "question_author": ""} {"qid": 1250, "query": "How to handle wealthy player characters as a GM?", "score": 52, "views": 16289, "answer_pids": [13087, 13091, 13096, 13098, 13110, 13112, 14244, 25896, 30141, 30152], "question_author": ""} {"qid": 1251, "query": "What does DMG mean?", "score": 52, "views": 16048, "answer_pids": [64728, 64729, 64734], "question_author": "Tony Meyer"} {"qid": 1252, "query": "What should be done if I suspect a player is using weighted dice?", "score": 52, "views": 14327, "answer_pids": [76408, 76409, 76411, 76418, 76423, 76443, 76445], "question_author": null} {"qid": 1253, "query": "How do I get better at improvising city features?", "score": 52, "views": 13984, "answer_pids": [46365, 46366, 46367, 46369, 46370, 46391, 46423, 46426, 46439, 46441, 47085, 47137, 66592], "question_author": ""} {"qid": 1254, "query": "How can I use powerful NPCs without overshadowing the player characters?", "score": 52, "views": 13067, "answer_pids": [47886, 47888, 47889, 47892, 47893, 47894, 47899, 47914, 47934], "question_author": "Naseer"} {"qid": 1255, "query": "How do I deal with a player whose character wanders off for no reason?", "score": 52, "views": 10452, "answer_pids": [46538, 46542, 46543, 46544, 46566, 46572, 46584, 46597, 46616], "question_author": "Drew Hoskins"} {"qid": 1256, "query": "When and how does combat start in a standoff?", "score": 52, "views": 6017, "answer_pids": [47944, 47950, 47954], "question_author": "badp"} {"qid": 1257, "query": "How can a GM prevent growing disillusioned with their own game?", "score": 52, "views": 5414, "answer_pids": [14652, 14653, 14654, 14655, 14656, 14658, 14660, 14661, 14665, 14667, 14867], "question_author": "Firefeather"} {"qid": 1258, "query": "What is a good way to become a better NPC/monster actor?", "score": 52, "views": 3148, "answer_pids": [1918, 1920, 1921, 1927, 1928, 1934, 22676], "question_author": "old_warbaker"} {"qid": 1259, "query": "Can you cast a spell learned from the Magic Initiate feat using spell slots?", "score": 51, "views": 59939, "answer_pids": [32512, 84010], "question_author": ""} {"qid": 1260, "query": "What are the major changes between Pathfinder first and second editions?", "score": 51, "views": 39379, "answer_pids": [72937, 72953], "question_author": ""} {"qid": 1261, "query": "How to build an evil campaign", "score": 51, "views": 27810, "answer_pids": [26273, 26274, 26285, 27754, 27761], "question_author": ""} {"qid": 1262, "query": "Why is this uncommon magic item better than this rare magic item?", "score": 51, "views": 25901, "answer_pids": [63239, 63261, 63264, 63273, 63283], "question_author": "keithjgrant"} {"qid": 1263, "query": "How to kill a character without making their player too angry", "score": 51, "views": 19964, "answer_pids": [12757, 12758, 12761, 12762, 13738, 39448, 42508], "question_author": "Ken Liu"} {"qid": 1264, "query": "Can I use my familiar from the Find Familiar spell as a safety deposit box?", "score": 51, "views": 19569, "answer_pids": [29715, 29717, 33852, 83785], "question_author": ""} {"qid": 1265, "query": "What to do with players who have done something very stupid, but unwittingly?", "score": 51, "views": 16712, "answer_pids": [48819, 48820, 48821, 48826, 48828, 48840, 48841, 48865, 49071], "question_author": ""} {"qid": 1266, "query": "Handling a character who frequently pickpockets teammates", "score": 51, "views": 15824, "answer_pids": [49736, 49737, 49738, 49742, 49747, 49752, 49774, 49777, 49784, 49795, 49811, 78188], "question_author": ""} {"qid": 1267, "query": "What are the statistical implications of doubling damage on crit instead of doubling the dice rolled?", "score": 51, "views": 15765, "answer_pids": [36940, 36948], "question_author": ""} {"qid": 1268, "query": "What makes the Monstrous Crab so dangerous?", "score": 51, "views": 13147, "answer_pids": [33750, 33777], "question_author": ""} {"qid": 1269, "query": "Risk of a TPK vs realistic NPCs", "score": 51, "views": 11958, "answer_pids": [34056, 34059, 34061, 34062, 34063, 34064, 34065, 34074, 34078, 34082, 34101, 34434], "question_author": "Vinko Vrsalovic"} {"qid": 1270, "query": "A player constantly uses his mobile phone during our game session", "score": 51, "views": 11612, "answer_pids": [32180, 32181, 32182, 32183, 32189, 32202, 32284], "question_author": "serg555"} {"qid": 1271, "query": "How can I convince players not to offload a seemingly useless weapon?", "score": 51, "views": 10068, "answer_pids": [43091, 43092, 43096, 43097, 43098, 43099, 43114, 43117, 43133], "question_author": ""} {"qid": 1272, "query": "NPC casting Suggestion on PC: who decides its reasonable?", "score": 51, "views": 9881, "answer_pids": [56257, 56264, 56265, 56271, 56288, 56300], "question_author": null} {"qid": 1273, "query": "Why wouldn\u2019t I just run away from the eldritch horror?", "score": 51, "views": 9617, "answer_pids": [25525, 25537, 25547, 25549, 25552], "question_author": "TALlama"} {"qid": 1274, "query": "What do jagged lines crossing a corridor mean on this old D&D map?", "score": 51, "views": 8214, "answer_pids": [47061, 47063, 47065], "question_author": ""} {"qid": 1275, "query": "Punishing a player for instant-actioning", "score": 51, "views": 8005, "answer_pids": [28008, 28009, 28011, 28013, 30958], "question_author": ""} {"qid": 1276, "query": "Can a new player join a group only when a new campaign starts?", "score": 51, "views": 7938, "answer_pids": [69617, 69620, 69635], "question_author": ""} {"qid": 1277, "query": "Should I prohibit a player from having a character goal that makes me uncomfortable?", "score": 51, "views": 7554, "answer_pids": [83599, 83601, 83615, 83616], "question_author": ""} {"qid": 1278, "query": "Does a PC know the general strengths and weaknesses of their stats?", "score": 51, "views": 6117, "answer_pids": [53829, 53832, 53833, 53840, 53842, 53853, 53854, 53913, 53949], "question_author": "jrista"} {"qid": 1279, "query": "How can I check if a new group is OK with a plotline from an ethical standpoint without spoiling the plot?", "score": 51, "views": 5952, "answer_pids": [81548, 81550, 81551, 81558, 81583, 81607, 81614, 81618], "question_author": ""} {"qid": 1280, "query": "How can GMs make their game worlds more inclusive?", "score": 51, "views": 5436, "answer_pids": [2346, 2348, 2349, 2354, 2358, 2359, 2362, 2368, 2372, 2391, 2757, 9015, 48758, 48768, 48810], "question_author": ""} {"qid": 1281, "query": "Can deaf people play RPGs?", "score": 51, "views": 4391, "answer_pids": [17665, 17666, 17669, 17670, 65962, 65964], "question_author": "Goodbye Stack Exchange"} {"qid": 1282, "query": "How do I make skill checks more than just sequences of die chucking?", "score": 51, "views": 3865, "answer_pids": [1199, 1200, 1202, 1214, 1215, 1217, 1237, 1242, 1266, 2351, 18911], "question_author": "Martha F."} {"qid": 1283, "query": "Your friend wants to learn the ways of the Dungeon Master, what do you do?", "score": 51, "views": 3568, "answer_pids": [221, 234, 245, 253, 315, 324, 1153, 1174, 1241, 4055, 6246, 20396, 22680, 35124], "question_author": ""} {"qid": 1284, "query": "What are the key considerations to creating compelling environments?", "score": 51, "views": 2505, "answer_pids": [13248, 13249, 13268, 13511, 13515, 13519], "question_author": ""} {"qid": 1285, "query": "Two-Weapon Fighting & Bonus Actions", "score": 50, "views": 706476, "answer_pids": [24935, 24937, 25933, 25934, 33740, 73715], "question_author": ""} {"qid": 1286, "query": "What can a familiar actually do?", "score": 50, "views": 210136, "answer_pids": [26092, 28939, 42175, 45454], "question_author": "Alan"} {"qid": 1287, "query": "Can spells with a verbal component be cast underwater if the caster cant breathe water?", "score": 50, "views": 62207, "answer_pids": [37050, 44977, 52942], "question_author": ""} {"qid": 1288, "query": "When are Opportunity Attacks provoked while holding a reach and a non-reach weapon?", "score": 50, "views": 35892, "answer_pids": [23996, 27059, 27060, 27063, 27066, 36307, 37596, 68307, 68525], "question_author": null} {"qid": 1289, "query": "Prop Coins: How to get many coins for little (real life) money?", "score": 50, "views": 26352, "answer_pids": [36386, 36387, 36388, 36390, 36391, 36517, 36535, 38211, 38220, 38356, 39460], "question_author": "Ioannis Karadimas"} {"qid": 1290, "query": "Which item gets your soul first, Blackrazor or a Ring of Mind Shielding?", "score": 50, "views": 19712, "answer_pids": [46708, 46709, 46710], "question_author": ""} {"qid": 1291, "query": "My Wizard is conjuring a piece of the sun. How will this affect my setting?", "score": 50, "views": 17681, "answer_pids": [47503, 47504, 47505, 47506, 47508, 47513], "question_author": ""} {"qid": 1292, "query": "Are features that allow \u22125 to attack to get +10 to damage mathematically sound?", "score": 50, "views": 16892, "answer_pids": [36063, 36067, 36071, 36077, 36254, 66310], "question_author": "Lance Roberts"} {"qid": 1293, "query": "How can I avoid problems that arise from rolling ability scores?", "score": 50, "views": 16477, "answer_pids": [63363, 63364, 63367, 63383, 63387, 63390, 63393, 63406, 63408, 63415, 63418, 63420, 63486, 63490, 63595, 63651, 63662, 85373], "question_author": "M4N"} {"qid": 1294, "query": "What rules say that summoning undead is evil, and what rules say why?", "score": 50, "views": 15418, "answer_pids": [21680, 21682, 52916], "question_author": "jfklein13"} {"qid": 1295, "query": "What to do about PC rerolling for great stats?", "score": 50, "views": 15358, "answer_pids": [63061, 63062, 63063, 63067, 63068, 63071, 63073, 63084, 63148, 63149, 63208, 63681], "question_author": ""} {"qid": 1296, "query": "How common is homosexuality in the Forgotten Realms?", "score": 50, "views": 13906, "answer_pids": [63650, 63652, 63655, 63720, 63829, 72617, 81133], "question_author": null} {"qid": 1297, "query": "Cut off hand with Ring of Regeneration on it", "score": 50, "views": 12620, "answer_pids": [20323, 20324, 20325, 20327, 20328, 20329, 20330, 20341, 20464], "question_author": "Pat Ludwig"} {"qid": 1298, "query": "How can I convince my DM that my diplomacy will work on his characters?", "score": 50, "views": 11330, "answer_pids": [52346, 52349, 52353, 52363, 52375, 57013], "question_author": "Pridkett"} {"qid": 1299, "query": "As a player, how do I deal with a spotlight hog?", "score": 50, "views": 9409, "answer_pids": [64938, 64941, 64945, 64949, 64972, 65000], "question_author": ""} {"qid": 1300, "query": "How can I deal with a soft alpha gamer player?", "score": 50, "views": 8821, "answer_pids": [87747, 87760, 87764, 87765, 87786], "question_author": ""} {"qid": 1301, "query": "What are the stand-out features of D&D 5e?", "score": 50, "views": 8130, "answer_pids": [21776, 21812, 21815, 21844], "question_author": ""} {"qid": 1302, "query": "Suggestions for decreasing metagaming and increasing player immersion?", "score": 50, "views": 7934, "answer_pids": [4795, 4796, 4798, 4799, 4800, 4810, 7360, 19424, 70732, 70770], "question_author": ""} {"qid": 1303, "query": "Should my fighter really punch that werewolf?", "score": 50, "views": 7888, "answer_pids": [64468, 64469], "question_author": ""} {"qid": 1304, "query": "What are the dangers of renaming spells and changing their flavour, without changing mechanics at all?", "score": 50, "views": 7592, "answer_pids": [59758, 59759, 59760, 59761, 59762, 59766, 59770, 86514], "question_author": "Andrew Vandever"} {"qid": 1305, "query": "How can I make my enemies feel real and make combat more engaging?", "score": 50, "views": 6935, "answer_pids": [67626, 67629, 67637, 67661, 67683, 67710], "question_author": "Rog"} {"qid": 1306, "query": "How do you discourage player knowledge as a GM?", "score": 50, "views": 6400, "answer_pids": [5080, 5081, 5082, 5083, 5084, 5086, 5091, 5098, 17091, 38636, 83025], "question_author": "Imageree"} {"qid": 1307, "query": "How should I prevent a player from instantly recognizing a magical impostor without making them feel cheated?", "score": 50, "views": 6089, "answer_pids": [85779, 85780, 85784, 85786, 85788, 85792, 85801], "question_author": ""} {"qid": 1308, "query": "Which Article Helped Get Rid of Technobabble in RPGs?", "score": 50, "views": 4181, "answer_pids": [68765], "question_author": ""} {"qid": 1309, "query": "How do I deal with GM burnout?", "score": 50, "views": 3468, "answer_pids": [1469, 1476, 1478, 1480, 1482, 1489, 8403, 8451, 13616, 20348], "question_author": ""} {"qid": 1310, "query": "Best answer for people concerned about RPG activities being occult or dangerous?", "score": 50, "views": 2413, "answer_pids": [855, 864, 865, 866, 906, 1911, 20585, 20669, 70958], "question_author": ""} {"qid": 1311, "query": "Does 5th edition have the equivalent of Taking 20?", "score": 49, "views": 60924, "answer_pids": [23981, 24001, 87812], "question_author": ""} {"qid": 1312, "query": "Do you take falling damage after a high jump of over 10 feet?", "score": 49, "views": 53082, "answer_pids": [27814, 27815, 27817, 27835, 48099], "question_author": ""} {"qid": 1313, "query": "What is D&D Adventurers League and How Does It Work?", "score": 49, "views": 50657, "answer_pids": [31599, 45079], "question_author": ""} {"qid": 1314, "query": "Can I be a follower of an evil god and not be evil?", "score": 49, "views": 21713, "answer_pids": [32031, 32032, 32033, 32034, 32035, 32036, 32039, 32042, 32046, 32050, 35489], "question_author": ""} {"qid": 1315, "query": "Need way to physically track initiative in face-to-face D&D game", "score": 49, "views": 21693, "answer_pids": [54144, 54147, 54148, 54149, 54165, 54168, 54169, 54170, 54191, 54193, 54209, 54278, 54297, 54337, 54957, 78688], "question_author": ""} {"qid": 1316, "query": "How does one handle a bottleneck of enemies in a doorway?", "score": 49, "views": 15904, "answer_pids": [25459, 25460, 25463, 25464, 25465, 25490, 36226], "question_author": "Don"} {"qid": 1317, "query": "Is 3d6 the same as 1d18?", "score": 49, "views": 13997, "answer_pids": [18292, 18293, 18294, 18295, 18298, 18313], "question_author": ""} {"qid": 1318, "query": "Am I overreacting or seeing things where they dont exist?", "score": 49, "views": 11093, "answer_pids": [76303, 76312], "question_author": "vanden"} {"qid": 1319, "query": "Is it appropriate to disallow the players building the world?", "score": 49, "views": 9716, "answer_pids": [36872, 36873, 36875, 36878, 36879, 36880, 36881, 36884, 36906, 36918], "question_author": ""} {"qid": 1320, "query": "At level one, can you really never recover hit dice?", "score": 49, "views": 9351, "answer_pids": [22239], "question_author": "RunnerRick"} {"qid": 1321, "query": "Is a mushroom a plant for the purposes of Speak With Plants?", "score": 49, "views": 8908, "answer_pids": [65262, 65263, 65282], "question_author": "Marcin"} {"qid": 1322, "query": "What happens when a Mage Hand touches a mimic?", "score": 49, "views": 7785, "answer_pids": [46955, 46958, 46969, 46974], "question_author": ""} {"qid": 1323, "query": "How can mimic phobia be cured or prevented?", "score": 49, "views": 7547, "answer_pids": [68806, 68807, 68809, 68812, 68813, 68821, 68851, 68860, 79372], "question_author": "Jason Sundram"} {"qid": 1324, "query": "What is the d20 bust, and what does post-d20 game mean?", "score": 49, "views": 6555, "answer_pids": [18920, 18922, 18926, 18928], "question_author": "Reinstate Monica - Goodbye SE"} {"qid": 1325, "query": "How to resolve multiple start of turn triggers", "score": 49, "views": 6209, "answer_pids": [55766, 55767, 55768, 55778], "question_author": "Firefeather"} {"qid": 1326, "query": "How do I handle when my players overestimate the importance of an encounter/a location?", "score": 49, "views": 4836, "answer_pids": [82840, 82842, 82843, 82844, 82845, 82858, 82860, 82869, 82886], "question_author": "Jan Goyvaerts"} {"qid": 1327, "query": "How do you balance a group with both charismatic players who enjoy role-playing and introverted players who dont?", "score": 49, "views": 4189, "answer_pids": [76997, 76998, 77000, 77011, 77013, 77020], "question_author": ""} {"qid": 1328, "query": "As a GM, how does one deal with all of the players playing nearly identical characters?", "score": 49, "views": 2847, "answer_pids": [5811, 5812, 5813, 5816, 5818, 5820, 5821, 5822, 5825, 5827, 5829, 6293, 13885, 24559], "question_author": "Numenetics"} {"qid": 1329, "query": "How many attacks can a fighter with two-weapon fighting make at 5th level?", "score": 48, "views": 122851, "answer_pids": [26949, 26950], "question_author": ""} {"qid": 1330, "query": "Handy Haversack vs. Bag of Holding", "score": 48, "views": 84771, "answer_pids": [26211, 42214, 59510, 59512], "question_author": ""} {"qid": 1331, "query": "What happens if I get the same skill from both my background and my class?", "score": 48, "views": 70124, "answer_pids": [11848, 27160, 50414], "question_author": "Numenetics"} {"qid": 1332, "query": "Can a D&D Necromancer be of a good alignment?", "score": 48, "views": 64242, "answer_pids": [10527, 10528, 10530, 10531, 10533, 10554, 10556, 10569, 10579, 10587, 12628, 12629, 12633, 16611, 19655], "question_author": ""} {"qid": 1333, "query": "Are there any good tabletop RPGs for young beginning players?", "score": 48, "views": 54175, "answer_pids": [64, 80, 110, 259, 319, 788, 1134, 2878, 2906, 7652, 7688, 7706, 9024, 9142, 9917, 11631, 13627, 14769, 14770, 14779, 19009, 19011], "question_author": "tttppp"} {"qid": 1334, "query": "How can I speed up combat?", "score": 48, "views": 31617, "answer_pids": [59687, 59689, 59695, 59696, 59721], "question_author": "Mark Rogers"} {"qid": 1335, "query": "Can clerics use swords?", "score": 48, "views": 20801, "answer_pids": [37919], "question_author": ""} {"qid": 1336, "query": "How should I go about stopping metagaming based on Roll20 showing unexplored areas?", "score": 48, "views": 15306, "answer_pids": [57424, 57425, 57426, 57427, 57431, 57438, 57452, 57454, 57473, 57477, 57526], "question_author": ""} {"qid": 1337, "query": "With the 2019 Sage Advice Compendium release, are Jeremy Crawfords tweets considered to be official rulings?", "score": 48, "views": 14761, "answer_pids": [66980, 67689], "question_author": ""} {"qid": 1338, "query": "My ranger has tamed an absurd number of Animal Companions with Handle Animal", "score": 48, "views": 12179, "answer_pids": [20860, 20861, 32914], "question_author": ""} {"qid": 1339, "query": "How can a boss enemy prevent being the most obvious target?", "score": 48, "views": 11038, "answer_pids": [57826, 57827, 57828, 57829, 57832, 57834, 57835, 57836, 57854, 57857, 57860, 57862, 57888, 57923, 58039], "question_author": "Simon"} {"qid": 1340, "query": "How to communicate to my GM that not being allowed to use stealth isnt fun for me?", "score": 48, "views": 9481, "answer_pids": [71124, 71125, 71130, 71147, 71153, 71159, 71190, 71236], "question_author": ""} {"qid": 1341, "query": "After successfully hitting with an attack roll, when rolling for damage, if you roll a 1, is that a negative crit?", "score": 48, "views": 9038, "answer_pids": [64669, 64681, 64690, 64695, 64727, 64754], "question_author": ""} {"qid": 1342, "query": "I want to run vast and realistic dungeons, but my players get frustrated trying to search every room", "score": 48, "views": 8230, "answer_pids": [35389, 35397, 35403, 35404, 35406, 35410, 35465, 71364], "question_author": ""} {"qid": 1343, "query": "What is the easiest way a Wizard can copy-protect the scrolls he makes?", "score": 48, "views": 8193, "answer_pids": [39228, 39230, 39231, 39232, 39234, 39241, 39278], "question_author": "Daniel Bingham"} {"qid": 1344, "query": "Im a DM and I want a hiatus but dont know how to tell my players", "score": 48, "views": 7427, "answer_pids": [80850, 80851, 80865, 80970], "question_author": ""} {"qid": 1345, "query": "Is it better to dash or dodge across an open field?", "score": 48, "views": 6128, "answer_pids": [79004, 79006, 79019], "question_author": "RunnerRick"} {"qid": 1346, "query": "Identifying my late fathers D&D stuff found in the attic", "score": 48, "views": 5761, "answer_pids": [70046, 70055], "question_author": "Don"} {"qid": 1347, "query": "Is it fair to press the players for decisions during combat?", "score": 48, "views": 2541, "answer_pids": [7147, 7148, 7149, 7150, 7151, 7153, 7157, 7159, 7161, 7162], "question_author": "David Wolever"} {"qid": 1348, "query": "How can I DM with aphantasia?", "score": 48, "views": 1868, "answer_pids": [40727], "question_author": ""} {"qid": 1349, "query": "Shame him into not shooting: multiple stress tracks in a single conflict", "score": 48, "views": 1253, "answer_pids": [12741, 12742, 12779], "question_author": ""} {"qid": 1350, "query": "Whats the starting wealth for higher levels?", "score": 47, "views": 372164, "answer_pids": [32827, 49558], "question_author": "Gordon Gustafson"} {"qid": 1351, "query": "How does wizard & cleric spell preparation and casting work?", "score": 47, "views": 225953, "answer_pids": [22063, 22094, 22268], "question_author": "jrista"} {"qid": 1352, "query": "What happens if you dont sleep?", "score": 47, "views": 96909, "answer_pids": [32919, 33956, 52373, 61125], "question_author": ""} {"qid": 1353, "query": "Can spells be cast through a Wall of Force?", "score": 47, "views": 75877, "answer_pids": [29079, 29080, 29082, 35883, 62785, 87715], "question_author": ""} {"qid": 1354, "query": "Does Repelling Blast work once per spell or once per beam?", "score": 47, "views": 64724, "answer_pids": [40960, 40966, 41140, 82764], "question_author": ""} {"qid": 1355, "query": "Does the Clone spell make someone effectively immortal?", "score": 47, "views": 55788, "answer_pids": [25383, 25384, 25385], "question_author": ""} {"qid": 1356, "query": "When exactly can a caster stop concentrating on a spell?", "score": 47, "views": 41931, "answer_pids": [66615, 66616, 66658, 66663], "question_author": ""} {"qid": 1357, "query": "When and how should I introduce my kids to RPGs?", "score": 47, "views": 22545, "answer_pids": [1306, 1307, 1308, 1314, 1315, 1316, 1353, 1959, 3346, 12273, 34441], "question_author": null} {"qid": 1358, "query": "What would happen if you opened a Bag of Holding underwater?", "score": 47, "views": 20566, "answer_pids": [37526, 37531, 37543, 37943, 83866], "question_author": ""} {"qid": 1359, "query": "Necromancer PC Insisting On Disguising Undead Minions", "score": 47, "views": 19072, "answer_pids": [67432, 67433, 67435, 67436, 67450, 67456, 67457, 67467, 67469], "question_author": "HorusKol"} {"qid": 1360, "query": "Power player is a problem. Any fix?", "score": 47, "views": 16706, "answer_pids": [31188, 31189, 31192, 31194, 31197, 31218, 31237, 31265, 31302], "question_author": ""} {"qid": 1361, "query": "Can a permanent Teleportation Circle be made on a moving vehicle?", "score": 47, "views": 15060, "answer_pids": [48755, 48766], "question_author": "bitbonk"} {"qid": 1362, "query": "Does the UA Centaur stack actually work?", "score": 47, "views": 13286, "answer_pids": [58566, 58567, 80245], "question_author": ""} {"qid": 1363, "query": "How to secretly talk to one of the players while DMing?", "score": 47, "views": 11478, "answer_pids": [19419, 19420, 19421, 19422, 19423, 19425, 19448, 19477, 19481, 19482, 19506, 19535], "question_author": "bitbonk"} {"qid": 1364, "query": "How do I deal with players persistently arguing for rules loopholes, even after Ive tried to finalise my rulings?", "score": 47, "views": 11191, "answer_pids": [52387, 52388, 52393, 52408, 52416, 52446, 64152], "question_author": ""} {"qid": 1365, "query": "Is it appropriate to punish a PC who snatches things from under the other PCs?", "score": 47, "views": 10365, "answer_pids": [35436, 35438, 35439, 35440, 35441, 35447, 35448, 35449, 35451, 35452, 35454, 35457, 35498, 35537], "question_author": "Tyler Carter"} {"qid": 1366, "query": "How did D&D end up so focused on fighting, when its inspirations werent?", "score": 47, "views": 9267, "answer_pids": [23993, 23997, 24009, 24032, 55708], "question_author": "Wookai"} {"qid": 1367, "query": "Metagaming when there isnt actually anything there?", "score": 47, "views": 9163, "answer_pids": [20480, 20481, 20482, 20483, 20484, 20489, 20491, 20495, 20511, 20521, 20524, 20536], "question_author": "Erick Robertson"} {"qid": 1368, "query": "Whats the simplest class for a child?", "score": 47, "views": 8272, "answer_pids": [58403, 58406, 58407, 58482, 58539, 58555, 58631, 65772], "question_author": "Wookai"} {"qid": 1369, "query": "How do I avoid lecturing my players", "score": 47, "views": 7964, "answer_pids": [58715, 58716, 58717, 58731, 58757], "question_author": "LittleBobbyTables - Au Revoir"} {"qid": 1370, "query": "What to do when you seem like you do not fit in with a group?", "score": 47, "views": 6480, "answer_pids": [7235, 7237, 7238, 7239, 7242, 8525, 32648], "question_author": "Hamish Downer"} {"qid": 1371, "query": "Keeping the world alive whilst PCs take a rest mid-adventure?", "score": 47, "views": 6254, "answer_pids": [59942, 59943, 59947, 59950, 61528], "question_author": ""} {"qid": 1372, "query": "Maximum bookkeeping fantasy game with magic", "score": 47, "views": 6230, "answer_pids": [26236, 26237, 26242, 26248, 26255, 26266, 26283, 26292, 26305, 26310, 26312], "question_author": ""} {"qid": 1373, "query": "Whats the difference between a roleplaying game and a board game?", "score": 47, "views": 5691, "answer_pids": [12047, 12048, 12050, 12052, 12053, 12059, 12061, 12062, 12095, 27038, 29462], "question_author": "Vinko Vrsalovic"} {"qid": 1374, "query": "Does an Ooze (Gelatinous Cube) float?", "score": 47, "views": 5412, "answer_pids": [51956, 75368], "question_author": "Codemwnci"} {"qid": 1375, "query": "How to get out of a story deadlock", "score": 47, "views": 4776, "answer_pids": [8319, 8320, 8321, 8322, 8323, 8324, 8325, 8326, 8329, 8340, 8342, 8391, 11813, 11817], "question_author": ""} {"qid": 1376, "query": "If you start casting a new concentration spell while concentrating on Haste, does the wave of lethargy interrupt the casting of the new spell?", "score": 47, "views": 4296, "answer_pids": [78862, 78863, 78865, 78866, 80762, 80787], "question_author": ""} {"qid": 1377, "query": "One of my friends/players committed suicide. How do I continue the game from here?", "score": 47, "views": 3092, "answer_pids": [46375, 46380, 46382, 46386, 46582], "question_author": ""} {"qid": 1378, "query": "What sort of cheat sheets for a new DM lead to a better flowing 5e game of Dungeons and Dragons?", "score": 46, "views": 125321, "answer_pids": [29420, 29421, 29438, 29448, 44379, 74752], "question_author": ""} {"qid": 1379, "query": "Is an Unarmed Strike considered a Weapon attack?", "score": 46, "views": 104946, "answer_pids": [30436, 30441, 41386], "question_author": ""} {"qid": 1380, "query": "How does the Divination wizards Portent feature interact with Advantage and Disadvantage?", "score": 46, "views": 76295, "answer_pids": [25477, 25478, 25479, 28334, 31723, 57402, 66301, 83724], "question_author": ""} {"qid": 1381, "query": "Can I choose to delay my turn in combat?", "score": 46, "views": 71316, "answer_pids": [24184, 51154, 51858], "question_author": "blueberryfields"} {"qid": 1382, "query": "To What Extent Can The Keen Mind Feat Replace a Wizards Spellbook?", "score": 46, "views": 56748, "answer_pids": [36371, 36372, 36378], "question_author": "Brenton Taylor"} {"qid": 1383, "query": "Players constantly capture enemies and question them, how to deal with it?", "score": 46, "views": 14556, "answer_pids": [61543, 61544, 61554, 61559, 61572, 61629], "question_author": "Alan"} {"qid": 1384, "query": "What do I do with a player whos always trying to go out on their own to farm XP?", "score": 46, "views": 13878, "answer_pids": [39559, 39561, 39562, 39568, 39569, 39570, 39583, 39605, 39664, 39676, 39683], "question_author": ""} {"qid": 1385, "query": "How to deal with a player who says no all the time?", "score": 46, "views": 13546, "answer_pids": [77734, 77735, 77736, 77752, 77756, 77784, 77803, 77824], "question_author": "blueberryfields"} {"qid": 1386, "query": "How can I explain how to play D&D to new players without them falling asleep from boredom?", "score": 46, "views": 13282, "answer_pids": [66185, 66187, 66188, 66189, 66190, 66191, 66198, 66206, 66212, 66214, 66340], "question_author": ""} {"qid": 1387, "query": "How to deal with a PC being played as homophobic?", "score": 46, "views": 12261, "answer_pids": [74249, 74250, 74251, 74253, 74254, 74257, 74260, 74268, 74282], "question_author": "Reid"} {"qid": 1388, "query": "How do I explain to an AD&D player that items that increase Armor Class in D&D 5e are overpowered?", "score": 46, "views": 10884, "answer_pids": [83851, 83852, 83859, 83861, 83867, 83869, 88323], "question_author": ""} {"qid": 1389, "query": "Techniques for making undead scary again", "score": 46, "views": 9645, "answer_pids": [17459, 17462, 17463, 17464, 17465, 17466, 17469, 17478, 17485, 17488, 61056], "question_author": ""} {"qid": 1390, "query": "How do I handle a player exploring the entire dungeon with his familiar?", "score": 46, "views": 9629, "answer_pids": [83026, 83027, 83029, 83032, 83033, 83036, 83055, 83061, 83062], "question_author": "Dimitri C."} {"qid": 1391, "query": "Can party betrayal be done successfully?", "score": 46, "views": 8992, "answer_pids": [5621, 5622, 5626, 5629, 5638, 5693], "question_author": "blueberryfields"} {"qid": 1392, "query": "What sound does an owlbear make?", "score": 46, "views": 8757, "answer_pids": [52336, 52337], "question_author": ""} {"qid": 1393, "query": "I Roll To Seduce The Dragon! (or, Do doublings of proficiency bonus stack?)", "score": 46, "views": 8196, "answer_pids": [32019, 40995, 43626], "question_author": "alexandrul"} {"qid": 1394, "query": "How can I replace a player character with an impostor without alerting the group?", "score": 46, "views": 8149, "answer_pids": [13433, 13434, 13436, 13438, 13439, 13440, 13442, 13443, 13445, 23017, 23021, 23062], "question_author": "blueberryfields"} {"qid": 1395, "query": "As DM, how do I deal with PCs that expect everything in the game to be relevant to the story?", "score": 46, "views": 8021, "answer_pids": [64574, 64575, 64578, 64588, 64612, 64631, 64673, 64678], "question_author": ""} {"qid": 1396, "query": "Can multiple Bags of Holding banish Tiamat?", "score": 46, "views": 8004, "answer_pids": [65026, 65027, 65038], "question_author": "Marcin"} {"qid": 1397, "query": "Resources for Complex Puzzle-Traps", "score": 46, "views": 7228, "answer_pids": [13652, 13654, 13655, 13657], "question_author": ""} {"qid": 1398, "query": "Skills the player has, but the character doesn\u2019t vs. skills the character has, but the player doesn\u2019t", "score": 46, "views": 6724, "answer_pids": [66953, 66954, 66972, 66976, 66988], "question_author": "blueberryfields"} {"qid": 1399, "query": "Can a red dragon be boiled to death?", "score": 46, "views": 6268, "answer_pids": [35671, 35674, 36936], "question_author": "axk"} {"qid": 1400, "query": "Problems Regarding a Mary-Sue type Player Character", "score": 46, "views": 6153, "answer_pids": [12869, 12870, 12873, 12874, 12875, 12879, 12894], "question_author": ""} {"qid": 1401, "query": "How do you handle greedy shadowrunners?", "score": 46, "views": 5499, "answer_pids": [33113, 33115, 33123, 33128, 33139, 35616, 35788], "question_author": "no thanks"} {"qid": 1402, "query": "What makes the Forgotten Realms forgotten?", "score": 46, "views": 4917, "answer_pids": [68053, 68067], "question_author": "Simon"} {"qid": 1403, "query": "Does the Rules Compendium overrule the core books?", "score": 46, "views": 4897, "answer_pids": [20429, 21906, 31284], "question_author": ""} {"qid": 1404, "query": "How do I avoid clich\u00e9s while improvising?", "score": 46, "views": 3827, "answer_pids": [12985, 12988, 12989, 12990, 13021, 13073, 13078, 21359], "question_author": ""} {"qid": 1405, "query": "Does the Dueling Fighting Style prevent using a shield?", "score": 45, "views": 115325, "answer_pids": [22457, 41368], "question_author": ""} {"qid": 1406, "query": "What do MAD and SAD mean?", "score": 45, "views": 72401, "answer_pids": [61660, 61661, 61662, 61664], "question_author": "Mark Rogers"} {"qid": 1407, "query": "When do Lair Actions occur?", "score": 45, "views": 62422, "answer_pids": [25052, 25053, 55157], "question_author": "George Stocker"} {"qid": 1408, "query": "Clarification of Blink Mechanics, or Should Someone have Punched in my Sandwich?", "score": 45, "views": 50088, "answer_pids": [35167, 58825], "question_author": "alexandrul"} {"qid": 1409, "query": "Can an unconscious person drink a potion?", "score": 45, "views": 35266, "answer_pids": [39481, 39482, 39485, 39486, 39489, 84625], "question_author": ""} {"qid": 1410, "query": "How far does a rogue move when dashing twice using Cunning Action?", "score": 45, "views": 31038, "answer_pids": [27853], "question_author": ""} {"qid": 1411, "query": "How loud/obvious is a wizard casting a spell?", "score": 45, "views": 30553, "answer_pids": [25588, 25609, 67408], "question_author": ""} {"qid": 1412, "query": "How can a character escape after being engulfed by a shambling mound?", "score": 45, "views": 26085, "answer_pids": [37314, 37334, 63641], "question_author": ""} {"qid": 1413, "query": "What happens when you put a Portable Hole inside a Bag of Holding while on the Astral plane?", "score": 45, "views": 23315, "answer_pids": [39947], "question_author": ""} {"qid": 1414, "query": "What happens to dying PCs when combat ends?", "score": 45, "views": 18765, "answer_pids": [46281, 46285, 46286, 46307], "question_author": "thesunneversets"} {"qid": 1415, "query": "What is a spellcaster for the purpose of magic item attunement?", "score": 45, "views": 18428, "answer_pids": [26560, 26570, 35992, 36023], "question_author": ""} {"qid": 1416, "query": "Do teleportation spells conserve momentum?", "score": 45, "views": 12706, "answer_pids": [51661, 51664, 51666, 51667, 51733, 51739, 61825, 87852], "question_author": ""} {"qid": 1417, "query": "How do I know which printing of the PHB I have?", "score": 45, "views": 12230, "answer_pids": [31610, 49980], "question_author": "bitbonk"} {"qid": 1418, "query": "How can I, as DM, avoid the Conga Line of Death occurring when implementing some form of flanking rule?", "score": 45, "views": 11930, "answer_pids": [68657, 68659, 68667, 68710, 68712, 68713, 68715], "question_author": ""} {"qid": 1419, "query": "Creating the personality of a character different from myself", "score": 45, "views": 11789, "answer_pids": [38494, 38496, 38497, 38499, 38511, 38517, 38528, 38542, 38549, 38568, 38582, 38656, 42410], "question_author": ""} {"qid": 1420, "query": "How many editions of Dungeons & Dragons are there?", "score": 45, "views": 11604, "answer_pids": [180, 278, 321, 4462], "question_author": ""} {"qid": 1421, "query": "Should I take a side in an external player conflict, or let my game die?", "score": 45, "views": 11015, "answer_pids": [76766, 76777, 76778, 76780, 76783, 76786, 76798, 76819, 76831, 76844], "question_author": "jrista"} {"qid": 1422, "query": "How to deal with sniping spellcaster PCs?", "score": 45, "views": 10800, "answer_pids": [53196, 53197, 53198, 53200, 53203, 53208, 53220, 53221, 53228, 53244, 53300], "question_author": ""} {"qid": 1423, "query": "Can massive damage knock out rather than causing instant death?", "score": 45, "views": 10578, "answer_pids": [36163, 36164, 36169, 57892], "question_author": ""} {"qid": 1424, "query": "How should I handle retconning (or not) a session with a drunk player and character deaths?", "score": 45, "views": 9999, "answer_pids": [20646, 20647, 20648, 20649, 20650, 20651, 20657, 20661, 20665, 20681], "question_author": "Wookai"} {"qid": 1425, "query": "Who actually makes the Magic Books that boost stats?", "score": 45, "views": 9965, "answer_pids": [8863, 8868, 8869, 8872, 8900, 8906, 9045, 9051, 9053, 10625, 10636, 28432, 83243], "question_author": "RunnerRick"} {"qid": 1426, "query": "Unhate-able Villains", "score": 45, "views": 8816, "answer_pids": [17768, 17769, 17770, 17774, 17775, 17776, 17787, 17801, 17805, 17807, 17817], "question_author": "keithjgrant"} {"qid": 1427, "query": "My DM asked me to stop roleplaying hostility toward another PC, but it fits my character", "score": 45, "views": 8754, "answer_pids": [32922, 32923, 32927, 32928, 32953, 32965], "question_author": ""} {"qid": 1428, "query": "Can you cast Invisibility on a large box and hide inside?", "score": 45, "views": 8390, "answer_pids": [17446, 17449, 17450, 17453, 17456, 17460, 17472, 25656], "question_author": ""} {"qid": 1429, "query": "How can I prevent a TPK without obvious Deus Ex Machina?", "score": 45, "views": 8343, "answer_pids": [8162, 8163, 8166, 8169, 8170, 8171, 8172, 8173, 8184, 8185, 26537], "question_author": ""} {"qid": 1430, "query": "Im feeling like my character doesnt fit the campaign", "score": 45, "views": 7443, "answer_pids": [72025, 72027, 72028, 72030, 72035, 72038, 72055, 72096, 72106, 72110], "question_author": ""} {"qid": 1431, "query": "Can you cast Teleport to teleport to a ship, even if its not in a fixed location?", "score": 45, "views": 7392, "answer_pids": [62227, 62234, 62235], "question_author": ""} {"qid": 1432, "query": "8 hour sessions require a meal! How can we incorporate meal breaks into the game itself?", "score": 45, "views": 7099, "answer_pids": [33275, 33283, 33292, 33296, 33309], "question_author": "CoderHawk"} {"qid": 1433, "query": "How To Divide Up Loot?", "score": 45, "views": 6530, "answer_pids": [13834, 13835, 13836, 13837, 13838, 14182, 14522], "question_author": ""} {"qid": 1434, "query": "How does failure work in Dungeon World? How does it move the story forward?", "score": 45, "views": 6436, "answer_pids": [11363, 11365, 16095, 23440, 71682], "question_author": "Ry St"} {"qid": 1435, "query": "Can you escape a Maze spell by turning into a Minotaur?", "score": 45, "views": 6075, "answer_pids": [59692, 59694, 59708], "question_author": "LittleBobbyTables - Au Revoir"} {"qid": 1436, "query": "What happens if a caster is surprised while casting a spell with a long casting time?", "score": 45, "views": 4436, "answer_pids": [71823, 71826, 71829, 71890], "question_author": "Igor Oks"} {"qid": 1437, "query": "How do I cater to the Three Stooges and good roleplayers at the same time?", "score": 45, "views": 2774, "answer_pids": [30562, 30563, 30574], "question_author": ""} {"qid": 1438, "query": "Whose portrait is printed on the Waterdhavian gold coin?", "score": 45, "views": 1543, "answer_pids": [67029], "question_author": ""} {"qid": 1439, "query": "Does flanking always grant advantage, or is it up to discussion?", "score": 44, "views": 263528, "answer_pids": [9344, 9349, 10147, 26493, 31566, 59336], "question_author": ""} {"qid": 1440, "query": "Casting multiple spells in a single round", "score": 44, "views": 193802, "answer_pids": [24860, 24863, 47817, 48910, 50345], "question_author": ""} {"qid": 1441, "query": "Is a critical failure on a natural 1 a rule or house rule?", "score": 44, "views": 96855, "answer_pids": [47208], "question_author": "John"} {"qid": 1442, "query": "Is the Warlocks Awakened Mind telepathy two-way, or only one-way?", "score": 44, "views": 93116, "answer_pids": [25910, 25912, 25916, 25919, 25920, 25937, 45712, 73884], "question_author": "Andrei Vajna II"} {"qid": 1443, "query": "When rolling percentages, do 1d100 and two d10s (percentiles) share the same probabilities?", "score": 44, "views": 68950, "answer_pids": [18372, 18378, 18379, 18384, 18397, 18414, 32581], "question_author": ""} {"qid": 1444, "query": "Does a bard have to play a musical instrument when using it as a spellcasting focus?", "score": 44, "views": 49061, "answer_pids": [37577, 37578], "question_author": "CoderHawk"} {"qid": 1445, "query": "Is the omission of a racial +2 to wisdom intentional?", "score": 44, "views": 40793, "answer_pids": [26633, 26670, 41051, 50591], "question_author": ""} {"qid": 1446, "query": "Do magic items really cost at least as much to create as they do to purchase?", "score": 44, "views": 39477, "answer_pids": [29471, 29473, 29477, 29486, 89055], "question_author": ""} {"qid": 1447, "query": "Im a quarter Draconian, a quarter Dragonborn and half Half-Dragon?", "score": 44, "views": 34191, "answer_pids": [23329, 23330, 23338, 23382, 46041], "question_author": "Stefano Borini"} {"qid": 1448, "query": "Does Evasion allow half damage even when unconscious?", "score": 44, "views": 28041, "answer_pids": [58066, 58067, 60529, 83512, 87444], "question_author": ""} {"qid": 1449, "query": "How do I deal with players who make friends with goblins?", "score": 44, "views": 22029, "answer_pids": [64007, 64008, 64009, 64010, 64013, 64022, 64054, 64076, 64080, 64092], "question_author": "Stefano Borini"} {"qid": 1450, "query": "How do we feed a character whose mouth is magically sewn shut?", "score": 44, "views": 15091, "answer_pids": [63495, 63496, 63500, 63503, 63504, 63505, 63508, 63509, 63517, 63543], "question_author": ""} {"qid": 1451, "query": "If a secret door is noticed with a DC 10 Wisdom (Perception) roll, dont most characters notice it passively?", "score": 44, "views": 14940, "answer_pids": [33919, 33920], "question_author": "RunnerRick"} {"qid": 1452, "query": "I killed a PCs animal companion at the end of last session, but later realized it should have survived; what are my options?", "score": 44, "views": 13227, "answer_pids": [67327, 67343, 67349, 67351], "question_author": ""} {"qid": 1453, "query": "Techniques for making high level D&D play more manageable?", "score": 44, "views": 13027, "answer_pids": [5738, 5741, 5742, 5744, 5753, 9370], "question_author": "RunnerRick"} {"qid": 1454, "query": "Is a Dragonwrought Kobold a True Dragon?", "score": 44, "views": 12932, "answer_pids": [10822, 14979, 15853, 52924], "question_author": "Leif"} {"qid": 1455, "query": "Our group keeps dying during the Lost Mine of Phandelver campaign. What are we doing wrong?", "score": 44, "views": 12434, "answer_pids": [73045, 73059, 73065, 73079], "question_author": "Mike King"} {"qid": 1456, "query": "What happens if you cast Resurrection on a found body part?", "score": 44, "views": 11835, "answer_pids": [47320, 47322, 47330], "question_author": ""} {"qid": 1457, "query": "Sentinel Raven uses too much awesome magic", "score": 44, "views": 11668, "answer_pids": [57708, 57748, 57756], "question_author": ""} {"qid": 1458, "query": "Can you play pen & paper roleplaying games over the Internet?", "score": 44, "views": 10939, "answer_pids": [37365, 37366, 37367, 37368, 37507], "question_author": "Reinstate Monica - Goodbye SE"} {"qid": 1459, "query": "In what medium does D&D get played nowadays?", "score": 44, "views": 10363, "answer_pids": [48347, 48348, 48349, 48350, 48351, 48352, 48353, 48354, 48362, 48373], "question_author": ""} {"qid": 1460, "query": "How to deal with a player who \u201cjumps the gun\u201d with premature rules-lawyering?", "score": 44, "views": 9995, "answer_pids": [37045, 37046, 37048, 37053, 37054, 37067, 39801], "question_author": "C. Ross"} {"qid": 1461, "query": "Randomly-generating weather for a sandbox campaign", "score": 44, "views": 9501, "answer_pids": [8684, 8685, 8686, 8687, 8690, 8713, 8775, 9018, 9020], "question_author": "Toon Krijthe"} {"qid": 1462, "query": "Does a short combat or casting one spell interrupt a Long Rest?", "score": 44, "views": 9485, "answer_pids": [21976], "question_author": ""} {"qid": 1463, "query": "A player made a 4th-level character for our new game and didnt tell me. Now what?", "score": 44, "views": 9222, "answer_pids": [58389, 58392, 58586], "question_author": "thesunneversets"} {"qid": 1464, "query": "How do I play a quiet, lone wolf character but also engage my group?", "score": 44, "views": 8335, "answer_pids": [21607, 21608, 21615, 21631, 21646], "question_author": ""} {"qid": 1465, "query": "Can a lich redeem itself?", "score": 44, "views": 8334, "answer_pids": [13401, 13402, 13403, 13404, 16884], "question_author": "Reinstate Monica - Goodbye SE"} {"qid": 1466, "query": "How to avoid the determinism of Passive Perception", "score": 44, "views": 8176, "answer_pids": [44565, 44571, 44572, 44573, 44577, 44631, 73472], "question_author": "FAE"} {"qid": 1467, "query": "How do I keep my players from focusing too deeply on unimportant details?", "score": 44, "views": 7064, "answer_pids": [65942, 65943, 65944, 65945, 65957, 65967], "question_author": "ICodeForCoffee"} {"qid": 1468, "query": "How to avoid favoritism when only one PC pulls on plot hooks?", "score": 44, "views": 6822, "answer_pids": [59854, 59873, 59874, 59881], "question_author": ""} {"qid": 1469, "query": "How much do I tell new players about new monsters?", "score": 44, "views": 6592, "answer_pids": [26238, 26239, 26258, 26267, 26279, 88831], "question_author": "Eddie Deyo"} {"qid": 1470, "query": "How to play a devious character when you are not personally devious?", "score": 44, "views": 6250, "answer_pids": [74904, 74905, 74927, 74932, 74960, 74966], "question_author": ""} {"qid": 1471, "query": "How do I know which edition of Dungeons and Dragons (D&D) the books Im looking at are for?", "score": 44, "views": 6219, "answer_pids": [64518, 66100], "question_author": ""} {"qid": 1472, "query": "Can a D&D campaign be successfully used as a Sunday school teaching instrument?", "score": 44, "views": 5836, "answer_pids": [40420, 40423, 40425], "question_author": "tonylo"} {"qid": 1473, "query": "Good examples of RAWs advantages over JPEG?", "score": 305, "views": 59726, "answer_pids": [213807, 213808, 213812, 214235, 215326, 215501, 215986, 215987, 216039, 224688, 231552, 245413, 246841, 248354, 256463], "question_author": "Lasse V. Karlsen"} {"qid": 1474, "query": "What do all those cryptic number and letter codes in a lens name mean?", "score": 196, "views": 130909, "answer_pids": [212299, 212305, 212467, 244957, 244958, 244959, 244960, 244961, 244962, 244963, 244964, 244965, 244966, 244967, 247912], "question_author": "Eddie Deyo"} {"qid": 1475, "query": "What are the best techniques for photographing overweight people?", "score": 168, "views": 35590, "answer_pids": [216270, 216271, 216274, 216292, 225927, 227454, 227455, 251768], "question_author": "aperkins"} {"qid": 1476, "query": "What is the exposure triangle?", "score": 154, "views": 57338, "answer_pids": [216353, 219961, 220005, 220025, 221864, 221872, 239211, 251861, 251862], "question_author": ""} {"qid": 1477, "query": "What is ISO on a digital camera?", "score": 129, "views": 17982, "answer_pids": [216363, 216365, 216380, 216413, 238845, 256892, 258336], "question_author": ""} {"qid": 1478, "query": "When do the differences between APS-C and full frame sensors matter, and why?", "score": 121, "views": 50461, "answer_pids": [214682, 214684, 214685, 219006, 229414, 255204], "question_author": "Zoot"} {"qid": 1479, "query": "What is bokeh, exactly?", "score": 121, "views": 12386, "answer_pids": [213859, 213860, 213861, 213862, 213863, 213865, 236395], "question_author": ""} {"qid": 1480, "query": "How and why do you use an image histogram?", "score": 120, "views": 8908, "answer_pids": [212268, 212269, 212391, 212414, 213312, 213682, 253906], "question_author": ""} {"qid": 1481, "query": "What are your easiest photography beginner tips?", "score": 117, "views": 11461, "answer_pids": [212079, 212097, 212102, 212108, 212112, 212117, 212125, 212126, 212186, 212189, 212193, 212196, 212210, 212249, 212322, 212330, 212480, 212481, 212483, 212654, 212673, 212735, 212747, 212749, 212756, 212761, 212842, 213060, 213062, 213070, 213075, 213384, 213395, 213412, 213522, 214178, 214188, 215391, 231608], "question_author": ""} {"qid": 1482, "query": "In portrait photography, what is broad lighting? What is short lighting?", "score": 113, "views": 18749, "answer_pids": [216284], "question_author": ""} {"qid": 1483, "query": "What blogs should I be following if I am learning photography?", "score": 113, "views": 12250, "answer_pids": [212225, 212228, 212229, 212230, 212250, 212254, 212255, 212317, 212318, 212319, 212384, 212424, 212428, 212529, 212567, 212568, 212570, 212574, 212661, 212848, 213335, 213396, 213492, 215525, 216295, 218781, 219200, 219201, 220315, 220324, 231633], "question_author": ""} {"qid": 1484, "query": "How do I approach people for a natural look in street photography?", "score": 113, "views": 7843, "answer_pids": [212387, 212396, 212410, 212899, 212913, 213426, 213955, 217801, 217802, 219667, 221952], "question_author": "chills42"} {"qid": 1485, "query": "How can I better organise and file my photos?", "score": 111, "views": 8035, "answer_pids": [211977, 211993, 212001, 212038, 212070, 212588, 212589, 212590, 212659, 212944, 214028, 214306], "question_author": "Justin Hamilton"} {"qid": 1486, "query": "What is RAW, technically?", "score": 107, "views": 7459, "answer_pids": [212988, 212989, 212990, 212992, 217497], "question_author": ""} {"qid": 1487, "query": "Why do breathtaking views turn into boring photos, and how can I do better?", "score": 103, "views": 19273, "answer_pids": [236263, 236264, 236266, 236275, 236276, 236277, 236278, 236279, 236288, 236291, 236297, 236300, 236308, 236325, 236330, 236331, 236341, 236494, 236506, 236786, 245304, 247625], "question_author": ""} {"qid": 1488, "query": "I know what a bad photo is, so why do I keep taking them?", "score": 102, "views": 22215, "answer_pids": [250718, 250721, 250724, 250725, 250727, 250729, 250731, 250732, 250739, 250740, 250745, 250747, 250752, 250753, 250761, 250763, 250781, 250782, 250786, 250816], "question_author": ""} {"qid": 1489, "query": "What can I do to get models to take my small camera more seriously?", "score": 101, "views": 27912, "answer_pids": [253312, 253313, 253314, 253315, 253316, 253318, 253319, 253326, 253336, 253343, 253344, 253345, 253349, 253350, 253351, 253362, 253369, 253370, 253374, 253376], "question_author": ""} {"qid": 1490, "query": "Why are lenses always round in shape?", "score": 101, "views": 18816, "answer_pids": [233202, 233203, 233206, 233207, 233210, 233211, 233212, 233224, 233227, 233236, 233261, 234310], "question_author": ""} {"qid": 1491, "query": "Can the sun damage the camera sensor? Under what conditions?", "score": 94, "views": 133442, "answer_pids": [214698, 214701, 215813, 215814, 218415, 223962, 232584, 250552], "question_author": "Stefano Borini"} {"qid": 1492, "query": "What does an unprocessed RAW file look like?", "score": 94, "views": 15781, "answer_pids": [256551, 256554, 256560], "question_author": ""} {"qid": 1493, "query": "How do you photograph rain?", "score": 88, "views": 13375, "answer_pids": [212840, 212843, 212844, 212846, 212850, 212855, 212860, 213014, 213641], "question_author": "RunnerRick"} {"qid": 1494, "query": "What is angle of view in photography?", "score": 86, "views": 43134, "answer_pids": [215913, 215914, 215917, 219969], "question_author": "blueberryfields"} {"qid": 1495, "query": "What historic reasons are there for common aspect ratios?", "score": 86, "views": 14203, "answer_pids": [221684, 223242, 236139, 251013], "question_author": "blueberryfields"} {"qid": 1496, "query": "How should I photograph falling snow?", "score": 83, "views": 25601, "answer_pids": [215509, 215510, 215511, 215513, 215523, 215537, 215563, 215673, 215856, 235257, 259251], "question_author": ""} {"qid": 1497, "query": "How much do lens lineups vary across DSLR platforms?", "score": 83, "views": 7082, "answer_pids": [218107, 218108, 218111, 218113, 218115, 218117, 218122, 221371, 235088, 237158, 244085], "question_author": "puri"} {"qid": 1498, "query": "What is Rembrandt lighting, and when do I use it?", "score": 82, "views": 83369, "answer_pids": [216355, 216364, 216965, 231260], "question_author": ""} {"qid": 1499, "query": "Do fluorescent lighting and shutter speed create a problem with color cast?", "score": 82, "views": 38038, "answer_pids": [214759, 214763, 214774, 225263, 236768, 246334], "question_author": "Alan"} {"qid": 1500, "query": "Is it worth trying to process film shot in 1989? (Fall of the Berlin Wall)", "score": 81, "views": 8482, "answer_pids": [252319, 252321, 252324, 252325], "question_author": "benhowdle89"} {"qid": 1501, "query": "How can I get my portrait subjects to look natural and drop the cheesy smile?", "score": 81, "views": 6732, "answer_pids": [212684, 212687, 212689, 212697, 212716, 212740, 212760, 212781, 218162, 218163, 242144, 242146, 262386], "question_author": ""} {"qid": 1502, "query": "How do I generate high quality prints with an ink jet printer?", "score": 78, "views": 59398, "answer_pids": [213179, 213185, 213881, 214679, 261865], "question_author": "Zoot"} {"qid": 1503, "query": "What advantages does manual mode have over aperture priority mode?", "score": 78, "views": 19701, "answer_pids": [212609, 212612, 212613, 212614, 212636, 212725, 212772, 213526, 213741, 213742, 229242, 256601, 256602], "question_author": "matt burns"} {"qid": 1504, "query": "How do I convert lens focal length (mm) to x-times optical zoom?", "score": 77, "views": 455014, "answer_pids": [220728, 220729, 228090, 228091, 228099], "question_author": "WireGuy"} {"qid": 1505, "query": "How does one focus for landscape photos in very dark conditions?", "score": 76, "views": 18220, "answer_pids": [213232, 213233, 213234, 213235, 213241, 213303, 213307, 213622, 213689, 215519, 215529, 215530, 227567, 227568, 234838], "question_author": "Karel"} {"qid": 1506, "query": "Is it lenses which make your photographs, not camera bodies?", "score": 75, "views": 30608, "answer_pids": [223378, 223379, 223380, 223386, 223394, 223397, 223399, 223402, 223435, 223469, 240322, 240329, 248362, 256316, 256324, 257968], "question_author": "Stefano Borini"} {"qid": 1507, "query": "How do I set the proper exposure for nighttime moon photos?", "score": 74, "views": 143611, "answer_pids": [212273, 212277, 212285, 212289, 216472, 216478, 221873, 230319, 239578, 239593, 239595, 259754], "question_author": "Codemwnci"} {"qid": 1508, "query": "What kind of photography is still better done with film cameras?", "score": 74, "views": 7722, "answer_pids": [212577, 212579, 212581, 212582, 212639, 212719, 212752, 212774, 212894, 214000, 215405, 215451, 218468, 224525, 224529, 224535, 224540, 224559, 251741], "question_author": ""} {"qid": 1509, "query": "How is ISO implemented in digital cameras?", "score": 71, "views": 22158, "answer_pids": [214022, 214024, 214031, 214047, 242832], "question_author": "jalf"} {"qid": 1510, "query": "What am I photographing INSIDE my camera?", "score": 71, "views": 8650, "answer_pids": [240054, 240057, 240060, 240061, 240074, 240077], "question_author": ""} {"qid": 1511, "query": "Is there a noticeable difference between 1.8 and 1.4 prime lenses?", "score": 70, "views": 87929, "answer_pids": [211986, 211990, 211992, 212012, 212023, 212162, 212304, 212523, 212688], "question_author": ""} {"qid": 1512, "query": "What is the Golden Ratio and why is it better than the Rule of Thirds?", "score": 70, "views": 69144, "answer_pids": [217862, 217864, 217867, 217873, 217875, 217909, 218017, 218083, 235942], "question_author": "Alastair"} {"qid": 1513, "query": "Are there any downsides to using a good-quality UV filter?", "score": 70, "views": 12819, "answer_pids": [212016, 212021, 212031, 219387, 219390, 219498, 226366, 252785], "question_author": ""} {"qid": 1514, "query": "Is there any significant difference between Nikon and Canon?", "score": 69, "views": 14347, "answer_pids": [220151, 220157, 220159, 220162, 220174, 222783, 224319, 224322, 224346, 224352, 229353, 237554, 251359], "question_author": "John Cavan"} {"qid": 1515, "query": "What exactly determines depth of field?", "score": 69, "views": 10133, "answer_pids": [218260, 218262, 218263, 220723, 252604], "question_author": ""} {"qid": 1516, "query": "Where can I find reviews of lenses?", "score": 69, "views": 9041, "answer_pids": [213035, 213036, 213037, 213049, 213050, 213204, 214633, 217618, 217619, 217634, 219032, 219056, 220070, 222698, 222710, 231146], "question_author": ""} {"qid": 1517, "query": "When is black and white likely to enhance an image?", "score": 69, "views": 4674, "answer_pids": [213212, 213213, 213214, 213215, 213217, 213218, 213225, 214354, 214356, 216710, 216721, 221485, 221506, 221544], "question_author": "PearsonArtPhoto"} {"qid": 1518, "query": "What does f-stop mean?", "score": 68, "views": 152381, "answer_pids": [212371, 212421, 233946, 238869], "question_author": ""} {"qid": 1519, "query": "How to make female chest look bigger?", "score": 68, "views": 20872, "answer_pids": [235608, 235609, 235619, 235626, 235627, 235943], "question_author": "Kempeth"} {"qid": 1520, "query": "Why are wide-angle lenses so much more expensive?", "score": 68, "views": 14030, "answer_pids": [215008, 215009, 215010, 215011, 215043, 215055], "question_author": "eruditass"} {"qid": 1521, "query": "How to get the best results for landscape and stars photographs?", "score": 67, "views": 29555, "answer_pids": [213480, 213486, 213521, 231452, 236483], "question_author": ""} {"qid": 1522, "query": "What is a diffraction limit?", "score": 67, "views": 22170, "answer_pids": [217463, 217466, 217468, 217473, 217482, 234066], "question_author": "Daniel Bingham"} {"qid": 1523, "query": "What should I look for when shopping for my first DSLR?", "score": 67, "views": 9001, "answer_pids": [213974, 213975, 214128, 214131, 221665, 232179, 235346, 236953], "question_author": "keithjgrant"} {"qid": 1524, "query": "Why do some lenses cost 10 times as much when the specs are very close?", "score": 67, "views": 7898, "answer_pids": [227036, 227037, 227040, 227041, 227043, 227050, 227100], "question_author": ""} {"qid": 1525, "query": "What are the key photography-related features from Photoshop that are missing in GIMP?", "score": 67, "views": 5175, "answer_pids": [212344, 217047, 217048, 218182, 221572, 225248, 240273, 243474], "question_author": ""} {"qid": 1526, "query": "How to compose flowering plants?", "score": 67, "views": 4423, "answer_pids": [247759, 247762, 247773, 247774, 247782], "question_author": "Didier Trosset"} {"qid": 1527, "query": "Why are SLR mechanisms still prevalent among high-end digital cameras?", "score": 66, "views": 9940, "answer_pids": [244241, 244242, 244243, 244244, 244254, 244259, 244264, 244294], "question_author": ""} {"qid": 1528, "query": "Are there reasons to use colour filters with digital cameras?", "score": 65, "views": 25796, "answer_pids": [212364, 212365, 212368, 212379, 212382, 212386, 238041, 246461], "question_author": "Stefano Borini"} {"qid": 1529, "query": "How do I get started in Astrophotography?", "score": 64, "views": 5807, "answer_pids": [212726, 212782, 212788, 212799, 213502], "question_author": "Shalom Craimer"} {"qid": 1530, "query": "Is watermarking worth it?", "score": 64, "views": 4220, "answer_pids": [212559, 212561, 212562, 213348, 213394, 213476, 213735], "question_author": ""} {"qid": 1531, "query": "What is something you learned recently about your gear that you wish you discovered earlier?", "score": 64, "views": 3754, "answer_pids": [212007, 212018, 212026, 212198, 213157, 213162, 213163, 213164, 213165, 213166, 216978, 216979, 216982, 216986, 216989, 217030, 217045, 217065, 217072, 217075, 217077, 217078, 217163, 217364, 217400, 222237], "question_author": ""} {"qid": 1532, "query": "What tools are available for RAW image processing in Linux?", "score": 62, "views": 91850, "answer_pids": [212286, 212287, 212359, 212539, 212662, 212672, 214305, 216964, 218246, 218335, 235794, 238002, 240193, 241979], "question_author": "Zoot"} {"qid": 1533, "query": "Arent all digital images ultimately just pixel values between 0 - 255?", "score": 62, "views": 24855, "answer_pids": [253600, 253602, 253603, 253604, 253605, 253606, 253608, 253611, 253612, 253620, 253625, 253629, 253631], "question_author": ""} {"qid": 1534, "query": "What should I pay attention to when choosing a tripod?", "score": 62, "views": 4117, "answer_pids": [213728, 213730, 214753], "question_author": "Daniel Bingham"} {"qid": 1535, "query": "Is there a free program to (batch) change photo files date to match EXIF?", "score": 61, "views": 114510, "answer_pids": [228231, 235455, 238273, 238939, 241763, 247051, 250602, 252922, 259509, 261329, 261930, 262077, 262893], "question_author": "Daniel Bingham"} {"qid": 1536, "query": "Is there development in the world of lenses?", "score": 61, "views": 2639, "answer_pids": [222590, 222592, 222593, 222834, 223289], "question_author": "JYelton"} {"qid": 1537, "query": "What is moir\u00e9? How can we avoid it?", "score": 60, "views": 76600, "answer_pids": [219635, 219636, 219637, 235012], "question_author": "keithjgrant"} {"qid": 1538, "query": "Why dont most cameras support PNG format?", "score": 60, "views": 21690, "answer_pids": [219500, 219502, 219504, 219506, 219511, 219535], "question_author": ""} {"qid": 1539, "query": "Whats a good strategy for choosing which photos to keep?", "score": 60, "views": 15341, "answer_pids": [217412, 217414, 217415, 217416, 217420, 217423, 217452, 217454, 217480, 218316, 218317, 218318, 218321, 218322, 218327, 218332, 218343, 218353, 218388, 218396, 222052], "question_author": ""} {"qid": 1540, "query": "What makes this interior photo look unrealistic?", "score": 60, "views": 11558, "answer_pids": [256325, 256326, 256329, 256357, 256383], "question_author": ""} {"qid": 1541, "query": "Why do pro photographers use huge lenses to take portrait shots?", "score": 59, "views": 64157, "answer_pids": [221919, 221920, 221922, 221924, 221926, 233125], "question_author": "jfklein13"} {"qid": 1542, "query": "Whats the difference between saturation and vibrance in Lightroom?", "score": 59, "views": 18024, "answer_pids": [218128, 218130, 228354], "question_author": "Joe Hopfgartner"} {"qid": 1543, "query": "What is Ansel Adams Zone System?", "score": 59, "views": 7697, "answer_pids": [216449, 216457, 216495], "question_author": ""} {"qid": 1544, "query": "What are the first few photography books someone should read?", "score": 59, "views": 4336, "answer_pids": [211983, 211988, 212095, 212276, 212281, 212321, 212496, 213406, 213407, 213413, 215198, 218798, 218894, 218896, 218899, 224831, 233692], "question_author": "Mnementh"} {"qid": 1545, "query": "Are Windows Photo Viewer rotations lossless?", "score": 58, "views": 18601, "answer_pids": [219916], "question_author": ""} {"qid": 1546, "query": "What is aperture, and how does it affect my photographs?", "score": 57, "views": 143143, "answer_pids": [212019, 212022, 212030, 212449, 216362, 233803], "question_author": ""} {"qid": 1547, "query": "What does expanded ISO mean?", "score": 57, "views": 43840, "answer_pids": [214735, 214736, 214738, 214739, 214765, 217338, 217342, 217344], "question_author": "Sam"} {"qid": 1548, "query": "Why do light sources appear as stars sometimes?", "score": 57, "views": 38111, "answer_pids": [216357, 216358, 216359, 216361, 238159], "question_author": "morganpdx"} {"qid": 1549, "query": "Is camera flash actually harmful to infants or newborns?", "score": 56, "views": 316938, "answer_pids": [222296, 222298, 222421, 233579, 233581, 233583, 233593, 256118], "question_author": "Dima"} {"qid": 1550, "query": "What is the difference between a pentaprism and a pentamirror?", "score": 56, "views": 50481, "answer_pids": [219616, 219618, 260017], "question_author": ""} {"qid": 1551, "query": "Why are Red, Green, and Blue the primary colors of light?", "score": 56, "views": 25512, "answer_pids": [254135, 254137, 254138, 254145, 254146, 254152, 260654], "question_author": "Kempeth"} {"qid": 1552, "query": "What methods can be used to micro-adjust autofocus of a camera body to a particular lens?", "score": 56, "views": 23850, "answer_pids": [211994, 212005, 218480, 220210, 243828], "question_author": ""} {"qid": 1553, "query": "Who, or what, is an Uncle Bob?", "score": 56, "views": 21704, "answer_pids": [251700, 251701, 251704, 251737], "question_author": ""} {"qid": 1554, "query": "Manual photography cheat sheet \u2014 where can I find one, or what should be on it?", "score": 56, "views": 17500, "answer_pids": [214785, 214786, 214787, 214789, 214871, 216482, 221599, 228715, 256176], "question_author": "Felix T. Katt"} {"qid": 1555, "query": "Why arent all viewfinders 100%?", "score": 56, "views": 8185, "answer_pids": [219621, 219622, 219630, 231258], "question_author": "Reid"} {"qid": 1556, "query": "What can be done using a 18-55mm lens?", "score": 55, "views": 310365, "answer_pids": [224809, 224810, 224812, 224815, 224823, 225120, 225146, 239847, 239848], "question_author": ""} {"qid": 1557, "query": "What is the best way to remove texture from a scanned textured photo paper?", "score": 54, "views": 107455, "answer_pids": [226190, 226212, 226254, 229977, 229980, 234361, 234372, 237520], "question_author": ""} {"qid": 1558, "query": "What is tone mapping? How does it relate to HDR?", "score": 54, "views": 48530, "answer_pids": [217005, 233050], "question_author": "dbkk"} {"qid": 1559, "query": "How can I backup my RAW photos while travelling without Internet access?", "score": 54, "views": 40226, "answer_pids": [212603, 212605, 212608, 212622, 212627, 212698, 212707, 227356, 227357, 227358, 227363, 228532, 230478, 230483], "question_author": ""} {"qid": 1560, "query": "Where to get community-based feedback on my photos online?", "score": 54, "views": 13000, "answer_pids": [213978, 213980, 213982, 213985, 213990, 218820, 218824, 219016, 219028, 219157, 219158, 219159, 219160, 219161, 221150, 221459, 223083, 223087, 223105], "question_author": "user666"} {"qid": 1561, "query": "Why is a dark-room safelight safe?", "score": 54, "views": 12686, "answer_pids": [245732, 245733, 245734], "question_author": ""} {"qid": 1562, "query": "What should I look for in a monitor for photo processing?", "score": 54, "views": 2163, "answer_pids": [213153, 213158, 213161, 213168, 213177], "question_author": ""} {"qid": 1563, "query": "Is there a general formula for image size vs. print size?", "score": 52, "views": 188573, "answer_pids": [212274, 217980, 218925, 236510, 244224], "question_author": "Roddy"} {"qid": 1564, "query": "Which software to assemble a time-lapse from images?", "score": 52, "views": 130187, "answer_pids": [212839, 226760, 226769, 228841, 228842, 233646, 233655, 241118, 241119, 242221, 242223, 258597, 260117], "question_author": "Ian"} {"qid": 1565, "query": "Do megapixels matter with modern sensor technology?", "score": 52, "views": 10124, "answer_pids": [221351, 221354, 221364, 224835, 225796, 238111], "question_author": ""} {"qid": 1566, "query": "Why is the 50mm prime lens the most standard?", "score": 51, "views": 13802, "answer_pids": [214076, 214082, 214090, 215210, 215229, 216500, 216502], "question_author": "Borror0"} {"qid": 1567, "query": "Which tools are good for creating panoramas/stitching multiple photos?", "score": 50, "views": 23398, "answer_pids": [212120, 212122, 212123, 212181, 212288, 212389, 212783, 212784, 212800, 213906, 214564, 214569, 222336], "question_author": ""} {"qid": 1568, "query": "How can one learn to shoot with both eyes open, and what are the advantages?", "score": 50, "views": 15403, "answer_pids": [217746, 217747, 217748, 218816], "question_author": "Adam Flynn"} {"qid": 1569, "query": "Why do photo cameras lose focus when you zoom, when movie or TV cameras keep it?", "score": 50, "views": 11873, "answer_pids": [251754, 251760, 251771], "question_author": ""} {"qid": 1570, "query": "Whats the difference between CCD and CMOS image sensors?", "score": 50, "views": 11009, "answer_pids": [217957, 217964, 218936, 230816, 239878], "question_author": "Sam"} {"qid": 1571, "query": "What is the best way to synchronize Adobe Lightroom databases between two computers?", "score": 50, "views": 8308, "answer_pids": [213069, 214170, 224705, 225532], "question_author": ""} {"qid": 1572, "query": "What quality to choose when converting to JPG?", "score": 49, "views": 191733, "answer_pids": [229307, 229311, 229314, 229315, 229316, 229333], "question_author": ""} {"qid": 1573, "query": "Can anyone suggest books/resources on the artistic side of photography?", "score": 49, "views": 9069, "answer_pids": [213064, 213065, 213066, 213623, 213627, 215212, 215214, 215215, 215216, 215232, 215255, 226320, 226341], "question_author": "Jeff"} {"qid": 1574, "query": "What image-quality characteristics make a lens good or bad?", "score": 49, "views": 8938, "answer_pids": [227293, 229787, 246800], "question_author": "Brisbe42"} {"qid": 1575, "query": "What does STM mean on a Canon lens?", "score": 48, "views": 394460, "answer_pids": [226535, 226536, 226566], "question_author": null} {"qid": 1576, "query": "What apertures are required to enable autofocus, including cross-type or high-precision focusing, on Canon DSLR cameras?", "score": 48, "views": 38101, "answer_pids": [222289, 229509, 239109], "question_author": "Sam"} {"qid": 1577, "query": "Why do we use RGB instead of wavelengths to represent colours?", "score": 48, "views": 16675, "answer_pids": [249925, 249928, 249929, 249938, 249940, 249943, 249960, 249968], "question_author": "Dimitrios Mistriotis"} {"qid": 1578, "query": "How can I take pictures through a skyscraper, airplane, or train window?", "score": 48, "views": 7436, "answer_pids": [215061, 215062, 215070, 215093, 217878, 221618, 227091], "question_author": "LifeH2O"} {"qid": 1579, "query": "What is the best way to clean lenses and filters?", "score": 48, "views": 5713, "answer_pids": [212950, 212951, 212953, 212954, 212956, 212965, 212970, 214202, 248046], "question_author": "Sam"} {"qid": 1580, "query": "What are good photography projects?", "score": 48, "views": 4145, "answer_pids": [212640, 212641, 212642, 212643, 212644, 212646, 212647, 212649, 212650, 212653, 212656, 212658, 212660, 213745, 214009, 214010, 217042, 217043, 217783, 217784, 217786, 219640, 223161, 223162], "question_author": ""} {"qid": 1581, "query": "How to shift EXIF date/time created by time in days, hours, minutes?", "score": 47, "views": 67719, "answer_pids": [217204, 217206, 217210, 217212, 217234, 221428, 223588, 232941, 252712, 255687, 257524], "question_author": ""} {"qid": 1582, "query": "What is the Rule of 600 in astrophotography?", "score": 47, "views": 50020, "answer_pids": [229319, 229320, 229321, 242890, 247402], "question_author": ""} {"qid": 1583, "query": "Do professionals really never use a 24-70 lens?", "score": 47, "views": 49439, "answer_pids": [239392, 239393, 239403, 239410, 239419, 239420, 239425, 239437, 250165], "question_author": ""} {"qid": 1584, "query": "What is the Rule of Odds?", "score": 47, "views": 31180, "answer_pids": [219357], "question_author": ""} {"qid": 1585, "query": "Which focal-length lens is usually used for portrait photography, and why?", "score": 47, "views": 30708, "answer_pids": [212597, 212598, 212606, 212764, 221203, 243187, 247484], "question_author": "thesunneversets"} {"qid": 1586, "query": "How can I get photos showing the shaft of light effect?", "score": 47, "views": 18014, "answer_pids": [215389, 215396, 216749, 236453], "question_author": "morganpdx"} {"qid": 1587, "query": "How do I achieve the technique which creates the illusion of busy locations being empty?", "score": 47, "views": 12256, "answer_pids": [251258, 251259, 251262, 251263, 251264, 251272, 251289, 251291], "question_author": ""} {"qid": 1588, "query": "How did photography work before auto-focus was invented?", "score": 47, "views": 9066, "answer_pids": [246849, 246850, 246851, 246860, 246870, 246888, 252295], "question_author": "Canon Gangsta"} {"qid": 1589, "query": "How many photos should I keep?", "score": 47, "views": 6035, "answer_pids": [216670, 216671, 216672, 216673, 216674, 216675, 216677, 216693, 232407, 232408, 233639, 233649], "question_author": "Dimitri C."} {"qid": 1590, "query": "What technique and camera settings should I use to capture lightning strikes?", "score": 47, "views": 3811, "answer_pids": [213010, 213011, 213013, 213016, 218791, 218983], "question_author": "Ian"} {"qid": 1591, "query": "Should I buy an original manufacturer battery, or is a generic brand OK?", "score": 46, "views": 118740, "answer_pids": [212903, 212904, 212906, 212908, 212909, 212919, 212920, 213624, 215227, 221149, 223466, 229204, 229358, 230677, 231919], "question_author": "Ian"} {"qid": 1592, "query": "Is it better to shoot with a higher ISO, or use lower ISO and raise the exposure in post-processing?", "score": 46, "views": 84000, "answer_pids": [230856, 230858, 230862, 230867, 230913], "question_author": "Ian"} {"qid": 1593, "query": "How can I maximize the blurry background, sharp subject (bokeh) effect?", "score": 46, "views": 16723, "answer_pids": [211967, 211974, 211975, 211980, 211982, 211985, 212119, 212306, 221560, 221562, 222467, 222514, 230704, 245571, 245572, 245715, 256423, 260380], "question_author": "Canon Gangsta"} {"qid": 1594, "query": "HDR then Panorama, or Panorama then HDR?", "score": 46, "views": 12844, "answer_pids": [219367, 219370, 219371, 223543, 223559, 226649, 234225, 235521], "question_author": "gomad"} {"qid": 1595, "query": "How do constant aperture zoom lenses work?", "score": 46, "views": 11741, "answer_pids": [216820, 216821, 216983, 216994, 217102, 258316, 259042], "question_author": ""} {"qid": 1596, "query": "What is crop factor and how does it relate to focal length?", "score": 46, "views": 11423, "answer_pids": [212056, 212057, 212075, 212149, 212156, 212160, 212830, 228184, 228186, 228194, 243563], "question_author": ""} {"qid": 1597, "query": "Has my idea for a time lapse technique been used before?", "score": 46, "views": 5867, "answer_pids": [259994, 259996, 260005, 260014, 260015], "question_author": ""} {"qid": 1598, "query": "Does this telescope only have a 4 blade aperture?", "score": 46, "views": 3271, "answer_pids": [249635, 249650], "question_author": "the_e"} {"qid": 1599, "query": "Why does fungus form in lenses, and how to get rid of it?", "score": 45, "views": 151954, "answer_pids": [221363, 221368, 222735, 230346, 234018, 240278, 241665, 248962, 251020, 257515, 261857], "question_author": ""} {"qid": 1600, "query": "What is the Rule of Thirds?", "score": 45, "views": 9533, "answer_pids": [212320, 212324, 218035], "question_author": ""} {"qid": 1601, "query": "Whats the best way to get constructive critique of your photographs?", "score": 45, "views": 7660, "answer_pids": [215144, 215145, 216479, 220610, 220639, 240745], "question_author": ""} {"qid": 1602, "query": "Should I keep Image Stabilization (IS) off when I do not actually need it?", "score": 45, "views": 5890, "answer_pids": [217761, 217762, 217766, 217771, 217772, 217773, 217806, 217874, 223336], "question_author": "John Rudy"} {"qid": 1603, "query": "Should higher ISOs really be preferred (all other things being equal)?", "score": 45, "views": 2709, "answer_pids": [231681, 231682, 231684, 231692, 231694, 231772, 237666, 254279], "question_author": ""} {"qid": 1604, "query": "Is switching to DNG worthwhile?", "score": 45, "views": 1636, "answer_pids": [212017, 212029, 212049, 212176, 213333, 224492, 224507], "question_author": ""} {"qid": 1605, "query": "Does Photoshop have an equivalent to Gimps Color to Alpha?", "score": 44, "views": 88092, "answer_pids": [213945, 214106, 214281, 234343, 260448], "question_author": "Brisbe42"} {"qid": 1606, "query": "How can I get dramatic shallow DOF with a kit lens?", "score": 44, "views": 41125, "answer_pids": [215651, 215652, 215653, 215661, 215686, 229303, 229310, 229326, 229331, 233870], "question_author": "Canon Gangsta"} {"qid": 1607, "query": "What does the number of elements and groups in a lens mean?", "score": 44, "views": 40025, "answer_pids": [221240, 221241], "question_author": ""} {"qid": 1608, "query": "What are the best sites to share photo galleries with friends?", "score": 44, "views": 26642, "answer_pids": [212163, 212164, 212167, 212194, 212197, 212272, 213309, 214014, 214017, 214111, 214367, 214911, 214917, 214924, 214927, 214928, 214930, 214939, 214944, 214948, 220230, 220231], "question_author": ""} {"qid": 1609, "query": "Why dont DSLRs come equipped with classic focusing screens (split prism, etc)?", "score": 44, "views": 17381, "answer_pids": [219457, 219458, 219460, 219462, 244756], "question_author": "Reinstate Monica - Goodbye SE"} {"qid": 1610, "query": "What does it mean for a photograph to be high key?", "score": 44, "views": 16229, "answer_pids": [218626, 218628, 218648, 223601, 236335], "question_author": ""} {"qid": 1611, "query": "What photo management software would you recommend for Linux?", "score": 44, "views": 13999, "answer_pids": [212188, 212192, 212797, 212808, 212817, 212821, 216957, 216958, 216999, 218987, 226445, 226452], "question_author": "Hamish Downer"} {"qid": 1612, "query": "Are there cameras that can photograph Wi-Fi/WLAN or mobile phone radiation?", "score": 44, "views": 10699, "answer_pids": [242597, 242602, 242604, 242605, 242625, 242640], "question_author": ""} {"qid": 1613, "query": "What are the pros and cons when shooting in RAW vs JPEG?", "score": 44, "views": 8990, "answer_pids": [211976, 211978, 211979, 211981, 211995, 212036, 212037, 213620, 214226, 214229, 214237, 217745], "question_author": "Mnementh"} {"qid": 1614, "query": "Is digital zoom really useful?", "score": 44, "views": 8658, "answer_pids": [220010, 220011, 220012, 220013, 220014, 220026, 220295, 220309, 221793, 224068, 224074, 234896], "question_author": ""} {"qid": 1615, "query": "Is there any photography equipment designed to ease use for the disabled?", "score": 44, "views": 7437, "answer_pids": [212528, 212534, 212550, 214121, 217479, 220223, 220229, 231674], "question_author": "Canon Gangsta"} {"qid": 1616, "query": "How does aperture work without cropping the image hitting the sensor?", "score": 44, "views": 7335, "answer_pids": [213760, 229719, 229721], "question_author": "Goodbye Stack Exchange"} {"qid": 1617, "query": "Unsharp mask - what do the parameters do?", "score": 44, "views": 5744, "answer_pids": [212922, 212927, 215890, 252846], "question_author": "Chris Persichetti"} {"qid": 1618, "query": "What are common beginner mistakes in photography?", "score": 44, "views": 4869, "answer_pids": [219670, 219671, 219672, 219673, 219674, 219675, 219676, 219678, 219679, 219684, 219687, 219690, 219693, 222284], "question_author": "Viktor Haag"} {"qid": 1619, "query": "How do I calculate the distance of an object in a photo?", "score": 43, "views": 187736, "answer_pids": [219958, 219960, 238452, 242116], "question_author": "Peter Boughton"} {"qid": 1620, "query": "What is the difference between EF and EF-S lenses?", "score": 43, "views": 130829, "answer_pids": [212220, 212223, 212226, 212227, 217902, 217903, 217904, 225564, 225565], "question_author": "Canon Gangsta"} {"qid": 1621, "query": "What are the dangers of removing a lens while the camera is on?", "score": 43, "views": 33881, "answer_pids": [216101, 216102, 216105, 216107, 216112, 216117, 238494, 239415, 239433], "question_author": ""} {"qid": 1622, "query": "Any good tutorials for learning how to post-process images?", "score": 43, "views": 14329, "answer_pids": [212073, 212080, 212510, 212819, 212835, 212836, 214283, 214287, 214325], "question_author": ""} {"qid": 1623, "query": "Is there a sane reason why \u00b9\u2044\u2081\u2082\u2085 is not, instead, exactly half of \u00b9\u2044\u2086\u2080?", "score": 43, "views": 13553, "answer_pids": [237036, 237037, 237039, 237060, 237062, 240858], "question_author": ""} {"qid": 1624, "query": "How to explain that the .jpg (digital file) of my photograph isnt free?", "score": 43, "views": 8559, "answer_pids": [242692, 242694, 242696, 242698, 242700, 242713], "question_author": ""} {"qid": 1625, "query": "Can you tell from a blurry photo if focus was too close or too far?", "score": 43, "views": 7988, "answer_pids": [256614, 256615, 256616, 256617, 256625, 256626], "question_author": ""} {"qid": 1626, "query": "How can I backup my photos while travelling?", "score": 43, "views": 7630, "answer_pids": [212326, 212331, 212336, 212337, 212339, 212354, 212617, 214364, 214946, 214961, 215457, 220421, 220430, 220435, 220446], "question_author": "Reinstate Monica - Goodbye SE"} {"qid": 1627, "query": "What is Hyperfocal Distance?", "score": 43, "views": 7404, "answer_pids": [212004, 212008, 212010, 213181, 216120, 216131, 217084], "question_author": ""} {"qid": 1628, "query": "When best to use Multi-Zone/Matrix, Spot, or Center-Weight exposure metering modes?", "score": 43, "views": 7208, "answer_pids": [215141, 215146, 215148], "question_author": "Bart"} {"qid": 1629, "query": "Are there disadvantages to a prosumer camera for a beginner, aside from cost?", "score": 43, "views": 6753, "answer_pids": [215894, 215895, 215896, 215899, 215908, 216466, 218441, 218450, 222466, 226828], "question_author": ""} {"qid": 1630, "query": "Why do it in-camera rather than in post-processing?", "score": 43, "views": 6573, "answer_pids": [217349, 217350, 217351, 217352, 217353, 217354, 217355, 217357, 217358, 217360, 217370], "question_author": "Canon Gangsta"} {"qid": 1631, "query": "Is crop-factor a bad thing?", "score": 43, "views": 5841, "answer_pids": [213748, 213749, 213750, 219129], "question_author": ""} {"qid": 1632, "query": "How do I tell which point-and-shoot cameras take good low light photos?", "score": 43, "views": 4850, "answer_pids": [212924, 212929, 212932, 225279, 228699], "question_author": ""} {"qid": 1633, "query": "How many actuations are too many actuations?", "score": 42, "views": 132366, "answer_pids": [214570, 214572, 214574, 234135], "question_author": ""} {"qid": 1634, "query": "How do I combine multiple exposures for action shots?", "score": 42, "views": 71817, "answer_pids": [219829, 219887, 219892, 219893], "question_author": ""} {"qid": 1635, "query": "What should I consider when using DSLRs in cold weather?", "score": 42, "views": 50954, "answer_pids": [215758, 215759, 215760, 215822, 215830, 223088, 223089, 223092], "question_author": ""} {"qid": 1636, "query": "What can a fisheye lens be useful for?", "score": 42, "views": 50870, "answer_pids": [213244, 213245, 213247, 213276, 213337, 244528, 249418], "question_author": ""} {"qid": 1637, "query": "Why is a 1 sensor actually 13.2 \u00d7 8.8mm?", "score": 42, "views": 34089, "answer_pids": [226994, 226995], "question_author": ""} {"qid": 1638, "query": "What are the advantages and disadvantages of using auto ISO on a DSLR?", "score": 42, "views": 21553, "answer_pids": [225208, 225209, 225210, 225226, 228472, 228836], "question_author": ""} {"qid": 1639, "query": "Why is there an ISO setting when shooting raw?", "score": 42, "views": 16410, "answer_pids": [228638, 228640, 228641, 238314, 238317, 259174], "question_author": "Canon Gangsta"} {"qid": 1640, "query": "How does one create the Dave Hill effect for portraits/action shots?", "score": 42, "views": 12911, "answer_pids": [215155], "question_author": ""} {"qid": 1641, "query": "What does it really mean that telephoto lenses flatten scenes?", "score": 42, "views": 11147, "answer_pids": [223261, 223272, 223276], "question_author": ""} {"qid": 1642, "query": "Why use a dedicated light meter instead of the one built into the camera?", "score": 42, "views": 10124, "answer_pids": [216501, 216503, 216504, 216505, 216508, 220794, 251177], "question_author": ""} {"qid": 1643, "query": "Why dont cameras offer more than 3 colour channels? (Or do they?)", "score": 42, "views": 7978, "answer_pids": [247434, 247435, 247436, 247438], "question_author": "benstraw"} {"qid": 1644, "query": "How do you photograph a lunar eclipse?", "score": 42, "views": 5822, "answer_pids": [215900, 215905, 216198], "question_author": ""} {"qid": 1645, "query": "How were photographs printed in newspapers in 1929?", "score": 41, "views": 26397, "answer_pids": [239867, 248564], "question_author": ""} {"qid": 1646, "query": "What software can show my most frequently used focal length?", "score": 41, "views": 19991, "answer_pids": [216752, 216754, 216766, 236319, 240570, 251390, 251391], "question_author": "Brisbe42"} {"qid": 1647, "query": "Why is giving clients RAW files such a sensitive matter among photographers?", "score": 41, "views": 18881, "answer_pids": [236186, 236187, 236188, 236190, 236193, 236197, 250711], "question_author": ""} {"qid": 1648, "query": "Should I get DX for my APS-C camera, or FX lenses in case I upgrade to full-frame in the future?", "score": 41, "views": 5053, "answer_pids": [212545, 212546, 212547, 217086], "question_author": ""} {"qid": 1649, "query": "What is the best way for a hobbyist to start generating income from photos?", "score": 41, "views": 1964, "answer_pids": [217958, 217966, 217978, 218008, 227069, 234217, 234224], "question_author": ""} {"qid": 1650, "query": "Is there good free software for editing Canon RAW files?", "score": 40, "views": 144301, "answer_pids": [215802, 215803, 215805, 215815, 215832, 215843, 216591, 224320, 225143, 240700, 243118, 243124], "question_author": "Powerlord"} {"qid": 1651, "query": "Whats the best way to store over 400 GB of digital photos?", "score": 40, "views": 17368, "answer_pids": [255784, 255785, 255786, 255787, 255790, 255791, 255792, 255794, 255796, 255798, 255799, 255807, 255808, 255997], "question_author": ""} {"qid": 1652, "query": "If an image is rotated losslessly, why does the file size change?", "score": 40, "views": 17199, "answer_pids": [247591, 247592, 247603, 247604, 247615], "question_author": "jrista"} {"qid": 1653, "query": "Will too much time-lapse photography reduce a DSLRs life, or cause any other damage?", "score": 40, "views": 15687, "answer_pids": [216064, 216076, 216078, 229021, 237860], "question_author": "Dr G"} {"qid": 1654, "query": "What methods do macro photographers use to get so close to butterflies, bees, insects and the like?", "score": 40, "views": 14610, "answer_pids": [216860, 216861, 216862, 216870, 216880, 216984, 254758], "question_author": ""} {"qid": 1655, "query": "What is the point of the depressible peg on a quick release plate?", "score": 40, "views": 11742, "answer_pids": [251253, 251254], "question_author": ""} {"qid": 1656, "query": "Whats the point of capturing 14 bit images and editing on 8 bit monitors?", "score": 40, "views": 10156, "answer_pids": [243597, 243599, 243600, 243601], "question_author": ""} {"qid": 1657, "query": "Why do stars appear as circles, not points?", "score": 40, "views": 9291, "answer_pids": [250940, 250941, 250943, 250963], "question_author": ""} {"qid": 1658, "query": "How to get a miniature effect on pictures without special equipment?", "score": 40, "views": 7223, "answer_pids": [222029, 222030, 222031, 222033, 222089], "question_author": "joshperry"} {"qid": 1659, "query": "What are the must-have filters for a new photographer?", "score": 40, "views": 5752, "answer_pids": [212314, 212349, 212350, 212628, 212703, 212778, 212826, 212829, 215015, 215016, 224724, 224732], "question_author": "Marco Mustapic"} {"qid": 1660, "query": "How can I get sharp manual focus despite poor eyesight?", "score": 39, "views": 25673, "answer_pids": [220437, 220439, 220440, 220488, 222933], "question_author": ""} {"qid": 1661, "query": "Does my crop sensor camera actually turn my lenses into a longer focal length?", "score": 39, "views": 20426, "answer_pids": [219273, 219279, 222657, 222661, 244171], "question_author": "AJ Finch"} {"qid": 1662, "query": "What is the best way to clean the sensor on a interchangeable lens camera (mirrorless or digital SLR)?", "score": 39, "views": 10634, "answer_pids": [211989, 211997, 213145, 213317, 213320, 217637, 231877, 231880, 242125, 243514, 250545, 256721], "question_author": "keithjgrant"} {"qid": 1663, "query": "What should one look for when buying a used lens?", "score": 39, "views": 9132, "answer_pids": [212704, 212705, 212710, 212884, 212888], "question_author": "Peter Boughton"} {"qid": 1664, "query": "What are alternatives to a tripod when I cant take one along?", "score": 39, "views": 8930, "answer_pids": [211968, 211969, 211971, 211972, 212000, 212003, 212054, 212077, 212293, 212311, 212356, 212845, 218413], "question_author": "Peter Boughton"} {"qid": 1665, "query": "In which situations should I use my circular polarizing filter?", "score": 39, "views": 8710, "answer_pids": [212411, 212412, 212413, 212415, 212416], "question_author": ""} {"qid": 1666, "query": "What is the difference between a linear and a circular polarizer?", "score": 39, "views": 8014, "answer_pids": [212923, 212928, 212943, 257379], "question_author": ""} {"qid": 1667, "query": "Why no zoom lenses with a maximum aperture wider than f/2.8?", "score": 39, "views": 7927, "answer_pids": [225416], "question_author": ""} {"qid": 1668, "query": "Focus problem vs. motion blur vs. camera shake - how to tell the difference?", "score": 39, "views": 6847, "answer_pids": [217488, 217489, 217491, 217504], "question_author": "LittleBobbyTables - Au Revoir"} {"qid": 1669, "query": "Why are some big telephoto lenses so expensive compared to telescopes?", "score": 39, "views": 4951, "answer_pids": [240886, 240891], "question_author": "Todd"} {"qid": 1670, "query": "Why are some lens hoods petal shaped and others not?", "score": 38, "views": 34408, "answer_pids": [212232, 212235, 212248, 233824, 233831, 236545, 242847], "question_author": "Mark"} {"qid": 1671, "query": "What is the difference between digital high ISO noise and film grain?", "score": 38, "views": 28548, "answer_pids": [212664, 212665, 212666, 212670], "question_author": ""} {"qid": 1672, "query": "Why does a bigger sensor lead to a shallower depth of field?", "score": 38, "views": 27446, "answer_pids": [219746, 219748, 219749, 219755, 232512], "question_author": ""} {"qid": 1673, "query": "How can I get a good HDR image from a single RAW file?", "score": 38, "views": 14052, "answer_pids": [212433, 212435, 213399, 214385, 214386, 224280, 224284, 224295, 228210, 244621, 251690], "question_author": ""} {"qid": 1674, "query": "Are equipment reviews from websites like kenrockwell.com and bythom.com reliable?", "score": 38, "views": 13248, "answer_pids": [219047, 219048, 219049, 219074, 219082, 219086, 219125, 219162, 219214], "question_author": "morganpdx"} {"qid": 1675, "query": "If I swallowed my SD Card would it be readable when it emerged?", "score": 38, "views": 12722, "answer_pids": [230214, 230217, 230218, 230228, 230234, 234827, 237547], "question_author": ""} {"qid": 1676, "query": "Why do people recommend 50mm or other prime lenses as starting lenses for learning photography?", "score": 38, "views": 10679, "answer_pids": [217847, 217848, 217849, 217853, 217857, 227833, 227836, 236880], "question_author": "Bryant"} {"qid": 1677, "query": "Why is The Steerage hailed one of the greatest photographs of all time?", "score": 38, "views": 9417, "answer_pids": [256184, 256185], "question_author": "Mark42"} {"qid": 1678, "query": "How to reproduce a photo with silhouette and bokeh?", "score": 38, "views": 5831, "answer_pids": [254905, 254906, 254913], "question_author": "morganpdx"} {"qid": 1679, "query": "How can I show a slope? Even mountain roads appear flat in my photos", "score": 38, "views": 5108, "answer_pids": [219589, 219590, 219591], "question_author": "Shalom Craimer"} {"qid": 1680, "query": "What is back-focusing?", "score": 37, "views": 96575, "answer_pids": [212586, 212591, 212593, 212596], "question_author": "matt burns"} {"qid": 1681, "query": "How to nail focus for DSLR astrophotography?", "score": 37, "views": 33114, "answer_pids": [226468, 226469, 226470, 226969, 240866], "question_author": "Iszi"} {"qid": 1682, "query": "What is a macro lens?", "score": 37, "views": 21550, "answer_pids": [213490, 213493, 213496, 233455, 260995], "question_author": "javamonkey79"} {"qid": 1683, "query": "What makes a photo a good photo?", "score": 37, "views": 12617, "answer_pids": [213588, 213589, 213591, 213592, 213594, 213642, 218412, 218419, 254672], "question_author": ""} {"qid": 1684, "query": "Should I be shooting in RAW even though Im a beginner that doesnt know how to retouch photos yet?", "score": 37, "views": 9607, "answer_pids": [217698, 217699, 217700, 217701, 217703, 217706, 217709, 217723, 229653], "question_author": ""} {"qid": 1685, "query": "Why do flashes make a whistling sound when recycling?", "score": 37, "views": 7757, "answer_pids": [215865, 215867], "question_author": "Uri"} {"qid": 1686, "query": "Does Auto White-Balance Really Work? How?", "score": 37, "views": 5448, "answer_pids": [215702, 215703, 250966], "question_author": "Alan"} {"qid": 1687, "query": "When you zoom in with a lens on an SLR why does the lens go in then out?", "score": 37, "views": 4453, "answer_pids": [223415, 223418, 223422], "question_author": "Karel"} {"qid": 1688, "query": "How do I interpret an MTF Chart?", "score": 37, "views": 4069, "answer_pids": [213154, 213155, 244137, 251175], "question_author": "Todd"} {"qid": 1689, "query": "How do I give someone a critique of their photograph?", "score": 37, "views": 3768, "answer_pids": [217179, 230597, 242468], "question_author": ""} {"qid": 1690, "query": "How can I take advantage of fog in my photographs?", "score": 37, "views": 3064, "answer_pids": [216872, 216873, 216881, 216884], "question_author": ""} {"qid": 1691, "query": "Why dont they make all DSLRs with an electronic shutter?", "score": 37, "views": 2658, "answer_pids": [212376, 212378, 240756], "question_author": "fluxd"} {"qid": 1692, "query": "Whats a concise Im not dangerous phrase for a business card?", "score": 37, "views": 1523, "answer_pids": [221451, 221460, 221465, 221467, 221477], "question_author": "J. Walton"} {"qid": 1693, "query": "Is GPU or CPU more important for Photoshop and Lightroom?", "score": 36, "views": 265099, "answer_pids": [231662, 231663, 231666, 231669, 234140], "question_author": ""} {"qid": 1694, "query": "What are the major differences between these camera series by Canon?", "score": 36, "views": 56007, "answer_pids": [212504, 212513, 212517, 213295, 213353, 226124, 226125, 237101, 255801], "question_author": ""} {"qid": 1695, "query": "How to have colors in Milky-way?", "score": 36, "views": 28077, "answer_pids": [234261, 234290, 234351, 234352, 251629], "question_author": "WOPR"} {"qid": 1696, "query": "How can I take shallow depth of field photos with a point-and-shoot camera?", "score": 36, "views": 16157, "answer_pids": [211984, 211987, 212521, 212525, 212544, 213133, 214104, 214112, 219178, 219179, 219180, 222852], "question_author": ""} {"qid": 1697, "query": "How can I test a new lens to make sure it is operating correctly?", "score": 36, "views": 11030, "answer_pids": [219820, 219830, 219840], "question_author": ""} {"qid": 1698, "query": "Why are there no dark yellows, or bright violets?", "score": 36, "views": 8986, "answer_pids": [218717, 218719, 218721, 218723, 218733, 224861, 224883, 256712], "question_author": "warbaker"} {"qid": 1699, "query": "Image rejected by Shutterstock because the main subject is out of focus and it contains noise", "score": 36, "views": 8195, "answer_pids": [260889, 260890, 260891, 260892, 260893, 260894], "question_author": ""} {"qid": 1700, "query": "Do sensors wear out?", "score": 36, "views": 7536, "answer_pids": [221573, 221576, 221602, 221619, 221658, 221668], "question_author": "warbaker"} {"qid": 1701, "query": "Why take diagonal photos?", "score": 36, "views": 7343, "answer_pids": [215909, 215910, 215911, 215912, 215998, 218357], "question_author": ""} {"qid": 1702, "query": "What is bokeh panorama (also called the Brenizer method)?", "score": 36, "views": 7130, "answer_pids": [217585, 241161], "question_author": ""} {"qid": 1703, "query": "How can I use my dSLR or mirrorless camera as a webcam?", "score": 36, "views": 6566, "answer_pids": [260550, 260556, 260573, 260652], "question_author": "ankitjaininfo"} {"qid": 1704, "query": "Is version control software used for photos?", "score": 36, "views": 4576, "answer_pids": [224417, 224422, 224425, 224428, 224432, 224444, 226415, 236394], "question_author": ""} {"qid": 1705, "query": "What situations are best suited for spot metering?", "score": 36, "views": 4486, "answer_pids": [212372, 212373, 212377, 212401, 215285, 216518, 216524, 222943, 222955, 223042, 223070], "question_author": ""} {"qid": 1706, "query": "How does one develop good photographic vision and style?", "score": 36, "views": 4004, "answer_pids": [212856, 212857, 212858, 212861, 212864, 212865, 212866], "question_author": "Kristo"} {"qid": 1707, "query": "Whats the best number of shots to combine to produce an HDR photo?", "score": 36, "views": 2897, "answer_pids": [213440, 213446, 213451, 213461, 213466, 213956, 213957, 213959, 214709, 214713], "question_author": ""} {"qid": 1708, "query": "What should one consider when choosing a style of tripod head?", "score": 36, "views": 2194, "answer_pids": [212497, 212501, 222932], "question_author": "Jeff"} {"qid": 1709, "query": "Is there any practical difference between saving at 300ppi or 72ppi?", "score": 35, "views": 22208, "answer_pids": [235495, 235497, 235501, 235512, 262736], "question_author": ""} {"qid": 1710, "query": "How do I capture the milky way?", "score": 35, "views": 22196, "answer_pids": [231435, 231441, 231442, 231445, 231447, 231667], "question_author": "fluxd"} {"qid": 1711, "query": "How to white-balance photos shot in mixed-lighting environments?", "score": 35, "views": 18637, "answer_pids": [217631, 217636, 228980], "question_author": ""} {"qid": 1712, "query": "Should I shoot underexposed photos?", "score": 35, "views": 16294, "answer_pids": [218996, 218997, 219003, 219005, 219018, 224873, 231735], "question_author": "cmcculloh"} {"qid": 1713, "query": "How do I copyright my photographs?", "score": 35, "views": 15424, "answer_pids": [216553, 216554, 216561, 216593, 229754], "question_author": ""} {"qid": 1714, "query": "How do I use spot metering?", "score": 35, "views": 15123, "answer_pids": [216792, 216794, 216795, 216796, 221419, 223102], "question_author": ""} {"qid": 1715, "query": "Better to buy an older high-level camera or newer entry-level camera?", "score": 35, "views": 14363, "answer_pids": [236202, 236204, 236205, 236206, 236207, 236208, 236209, 236222, 236225, 240569], "question_author": "jrista"} {"qid": 1716, "query": "How does autofocus work?", "score": 35, "views": 13003, "answer_pids": [216533, 230289], "question_author": ""} {"qid": 1717, "query": "Whats the difference between using black and white mode in-camera, and converting in post?", "score": 35, "views": 12192, "answer_pids": [216849, 216852, 216853, 216867, 219358, 219373, 219855, 221757, 224944], "question_author": "alexandrul"} {"qid": 1718, "query": "Why are tripods forbidden in some public spaces and how to deal with it?", "score": 35, "views": 11326, "answer_pids": [219899, 219900, 219901, 219902, 219904, 219907, 219909, 219913, 219924], "question_author": ""} {"qid": 1719, "query": "Can I publish photos taken in public legally?", "score": 35, "views": 10300, "answer_pids": [213195, 213197, 213198, 213199, 213200, 213203, 213359], "question_author": "RoboShop"} {"qid": 1720, "query": "How are SpaceX rocket launches filmed?", "score": 35, "views": 9432, "answer_pids": [249612, 252637], "question_author": ""} {"qid": 1721, "query": "How can I know what speed card to get for my camera?", "score": 35, "views": 8573, "answer_pids": [212191, 212216, 235332], "question_author": "Numenetics"} {"qid": 1722, "query": "What are some tips for shooting in low light?", "score": 35, "views": 4015, "answer_pids": [212201, 212202, 212205, 212208, 212209, 212215, 212221, 212252, 214565, 214581], "question_author": ""} {"qid": 1723, "query": "Do good lenses really last a lifetime?", "score": 35, "views": 3483, "answer_pids": [215930, 215931, 215933, 215944, 215951, 231991], "question_author": ""} {"qid": 1724, "query": "What software is focused on reviewing and organizing images?", "score": 35, "views": 3191, "answer_pids": [214821, 214822, 214826, 214827, 214831, 214832, 214837, 220511, 220512, 220543, 220545, 220754, 230577], "question_author": "rem"} {"qid": 1725, "query": "Should changing the shutter speed affect the moons color?", "score": 35, "views": 2675, "answer_pids": [244816, 244822], "question_author": "timday"} {"qid": 1726, "query": "What attributes should I look for when buying batteries for a flash?", "score": 35, "views": 2461, "answer_pids": [212360, 212388, 212392, 212450, 214839, 214847, 214849, 221236], "question_author": null} {"qid": 1727, "query": "Is there a trick to shooting large group portraits without blinkers?", "score": 35, "views": 2079, "answer_pids": [213238, 213240, 213246, 216243, 216246], "question_author": ""} {"qid": 1728, "query": "Is there a software to automatically crop a scan of multiple images?", "score": 34, "views": 109810, "answer_pids": [222616, 222621, 222626, 222666, 222669, 229822, 231779, 234936, 236587], "question_author": "jfklein13"} {"qid": 1729, "query": "What is one stop?", "score": 34, "views": 46316, "answer_pids": [221903, 221904, 221905, 221911, 221914, 221960], "question_author": "Adam Dray"} {"qid": 1730, "query": "How do I choose a title for a photograph?", "score": 34, "views": 43941, "answer_pids": [219139, 219140, 219143, 219144, 219147, 219181, 231680, 231688, 231690, 231711], "question_author": ""} {"qid": 1731, "query": "What do I need to get photos with a unifom black background (not with post)?", "score": 34, "views": 42398, "answer_pids": [217104, 217106, 217107, 217108, 217109, 217111, 230307, 230308, 230311, 230318, 230320, 233001], "question_author": "Reinstate Monica - Goodbye SE"} {"qid": 1732, "query": "Lightroom vs. Elements vs. Photoshop: Side-by-side comparison?", "score": 34, "views": 29089, "answer_pids": [214420, 214428, 214433, 228853, 232001], "question_author": "OpaCitiZen"} {"qid": 1733, "query": "Technically, why is the out of focus area blurred more when using a bigger aperture?", "score": 34, "views": 26818, "answer_pids": [220578, 220580, 221896, 221897, 230966, 253046], "question_author": "chills42"} {"qid": 1734, "query": "Does it make sense to put an expensive lens on a cheap camera?", "score": 34, "views": 25453, "answer_pids": [244470, 244471, 244472, 244475, 244476, 244478, 244479, 244487, 244495, 244508], "question_author": ""} {"qid": 1735, "query": "Why is 18% grey considered to be in the middle for photography?", "score": 34, "views": 24897, "answer_pids": [240928, 240934, 240937, 240940, 240949, 248404, 255752, 257218], "question_author": "Robert"} {"qid": 1736, "query": "Whats the difference between Adobe RGB and sRGB and which should I set in my camera?", "score": 34, "views": 17279, "answer_pids": [220031, 220032, 220033, 220040, 222800, 222878], "question_author": "Ifi"} {"qid": 1737, "query": "Do super-super-lightweight tripods exist?", "score": 34, "views": 13727, "answer_pids": [212013, 212583, 213764, 213771, 213772, 213819, 217434, 217438, 221649, 226580, 231555], "question_author": "Cyric"} {"qid": 1738, "query": "Why store both JPEG and raw?", "score": 34, "views": 11931, "answer_pids": [218655, 218656, 218659, 218662, 218667, 235801, 237755, 237794, 240304, 246143], "question_author": "Todd"} {"qid": 1739, "query": "How do I keep the horizon straight when taking a photo (without a tripod)?", "score": 34, "views": 10477, "answer_pids": [221535, 221536, 221538, 221541, 221543, 221546, 253518], "question_author": "aperkins"} {"qid": 1740, "query": "What kind of camera do I need to capture deep space images?", "score": 34, "views": 8392, "answer_pids": [224245, 224510], "question_author": "Brenton Taylor"} {"qid": 1741, "query": "How do you find out the sweet spot of a lens?", "score": 34, "views": 7478, "answer_pids": [214001, 214002, 214006, 214008], "question_author": "keithjgrant"} {"qid": 1742, "query": "How can I hold my camera steady?", "score": 34, "views": 5607, "answer_pids": [221468, 221469, 221470, 221471, 221481, 221484, 221525, 221533, 221550, 221586, 221608, 221614, 223165, 223166, 223167, 223169, 223176, 223185, 224583], "question_author": "keithjgrant"} {"qid": 1743, "query": "Without modification, is the built-in pop-up flash EVER appropriate?", "score": 34, "views": 5089, "answer_pids": [225527, 225528, 225531, 256953], "question_author": null} {"qid": 1744, "query": "What makes a strong wide-angle composition?", "score": 34, "views": 4928, "answer_pids": [220116, 220119, 220124], "question_author": "DavRob60"} {"qid": 1745, "query": "(How) should I start with film photography?", "score": 34, "views": 4811, "answer_pids": [218381, 218387, 218389, 218391], "question_author": ""} {"qid": 1746, "query": "What are good gifts between $50 and $100 for photographers?", "score": 34, "views": 3129, "answer_pids": [215364, 215365, 215366, 215376, 215392, 215567, 216473, 220524, 236327], "question_author": "Martha F."} {"qid": 1747, "query": "What is the definition of a prime lens?", "score": 34, "views": 2844, "answer_pids": [213329, 213330, 213331, 213332, 214220, 237641], "question_author": "dan_waterworth"} {"qid": 1748, "query": "Help motivate me to use my SLRs manual mode", "score": 34, "views": 1453, "answer_pids": [215690, 215692, 215693, 215694, 215695, 215697, 215698, 215750, 216468, 216483], "question_author": "rem"} {"qid": 1749, "query": "Why do higher end lenses use USM instead of STM?", "score": 33, "views": 193931, "answer_pids": [239652, 239653], "question_author": ""} {"qid": 1750, "query": "How does automatic sensor cleaning work?", "score": 33, "views": 54514, "answer_pids": [232116], "question_author": ""} {"qid": 1751, "query": "How does the dynamic range of the human eye compare to that of digital cameras?", "score": 33, "views": 45782, "answer_pids": [225166, 225168, 225169, 225657], "question_author": "olajostein"} {"qid": 1752, "query": "How can I calculate what the effect of an extension tube will be?", "score": 33, "views": 32498, "answer_pids": [215719, 215720, 215733, 250105], "question_author": ""} {"qid": 1753, "query": "Should I buy a camera with kit lens, or body plus lens separately?", "score": 33, "views": 30684, "answer_pids": [212505, 212508, 212509, 212511, 212514, 212516, 212533, 212541, 226392, 232822, 232823, 246147], "question_author": "Shabbyrobe"} {"qid": 1754, "query": "What options are there for a camera bag that does not look like one?", "score": 33, "views": 19381, "answer_pids": [213032, 213033, 213041, 213042, 213118, 213205, 214135, 214137, 214163, 218577, 227009, 227105, 227112, 232627], "question_author": "Robert"} {"qid": 1755, "query": "Why am I unable to capture as many stars as I can see with my eyes with my new DSLR?", "score": 33, "views": 17600, "answer_pids": [235076, 235078, 235080, 235087], "question_author": "Kjartan \u00de\u00f3r Kjartansson"} {"qid": 1756, "query": "What are the best techniques for making great photographs of LEGO creations?", "score": 33, "views": 15202, "answer_pids": [222487, 222488, 222498, 226092, 226106, 251317], "question_author": ""} {"qid": 1757, "query": "Why does storing a long exposure photo take almost as long as the exposure time itself?", "score": 33, "views": 14928, "answer_pids": [227612, 227613, 227614], "question_author": "Hamish Downer"} {"qid": 1758, "query": "Why do people like square photos?", "score": 33, "views": 13312, "answer_pids": [243313, 243314, 243316, 243317, 243321, 243332, 243352, 243354], "question_author": ""} {"qid": 1759, "query": "How much benefit can one expect from a monopod?", "score": 33, "views": 12651, "answer_pids": [214049, 214053, 214056, 214059, 214064, 214169, 220655, 220664, 231243], "question_author": ""} {"qid": 1760, "query": "How does Kelvin for color temperature relate to Kelvin for actual temperature?", "score": 33, "views": 8552, "answer_pids": [243917, 243918, 243919, 243920, 243922], "question_author": "Brenton Taylor"} {"qid": 1761, "query": "Do I really need a fast 50mm lens?", "score": 33, "views": 7273, "answer_pids": [221071, 221072, 221077, 221079, 225743, 226841, 234473], "question_author": ""} {"qid": 1762, "query": "What is the Diagonal Method and should I use it instead of The Rule of Thirds?", "score": 33, "views": 7156, "answer_pids": [219105, 219107, 219108, 219124], "question_author": "thesunneversets"} {"qid": 1763, "query": "What factors cause or prevent generational loss when JPEGs are recompressed multiple times?", "score": 33, "views": 6429, "answer_pids": [254080, 254081, 254084, 254085], "question_author": null} {"qid": 1764, "query": "How can I stop my HDR shots looking so fake?", "score": 33, "views": 6320, "answer_pids": [212020, 212045, 212053, 213400, 217132, 217139, 218868], "question_author": ""} {"qid": 1765, "query": "How was photo data processed and transferred back to Earth from satellites before digital photography became commonplace?", "score": 33, "views": 6080, "answer_pids": [247284, 247290, 247298, 247299], "question_author": "spong"} {"qid": 1766, "query": "What do Pentax and Sigma DSLRs offer that differs from Canon and Nikon?", "score": 33, "views": 5565, "answer_pids": [224327, 224358, 225394, 225409, 236503], "question_author": null} {"qid": 1767, "query": "What is the advantage to back-button autofocus?", "score": 33, "views": 5141, "answer_pids": [217740, 217741, 217742, 217744, 217749, 217750, 217755], "question_author": "Tony Meyer"} {"qid": 1768, "query": "What is an efficient way to digitize a family photo collection?", "score": 33, "views": 3857, "answer_pids": [256561, 256563, 256565, 256574], "question_author": "Tony Meyer"} {"qid": 1769, "query": "How can knowledge of human perception of color be used in photography?", "score": 33, "views": 3481, "answer_pids": [217282, 217429, 217449, 217541, 217581, 218926], "question_author": "rem"} {"qid": 1770, "query": "What does a lens hood do?", "score": 33, "views": 2065, "answer_pids": [212418], "question_author": ""} {"qid": 1771, "query": "Are stock photo sites a viable source of income?", "score": 33, "views": 1855, "answer_pids": [213873, 213874, 213879, 224587], "question_author": ""} {"qid": 1772, "query": "Is it worth to upgrade from Canon 50mm f/1.8 to Canon 50mm f/1.4 on 550d?", "score": 32, "views": 31048, "answer_pids": [216182, 216185, 216236, 216269, 231139, 232795], "question_author": "Wok"} {"qid": 1773, "query": "How to batch edit a collection of raw files in Darktable?", "score": 32, "views": 30492, "answer_pids": [232313, 232338, 250924, 250932], "question_author": ""} {"qid": 1774, "query": "How to capture the scene exactly as my eyes can see?", "score": 32, "views": 30322, "answer_pids": [217804, 217815, 217816, 217832], "question_author": "keithjgrant"} {"qid": 1775, "query": "How do Micro 4/3s cameras compare with DSLR cameras?", "score": 32, "views": 28443, "answer_pids": [217362, 217363, 217365, 217366, 217369, 217397, 217421, 218370, 218409, 223427, 224466, 230759, 232066], "question_author": ""} {"qid": 1776, "query": "Whats the best way to scan in hundreds of pictures?", "score": 32, "views": 25052, "answer_pids": [215456, 215468, 215505, 215581, 233610], "question_author": "Shabbyrobe"} {"qid": 1777, "query": "CSI image resolution enhance: How real is it?", "score": 32, "views": 23588, "answer_pids": [222692, 222697, 222737, 222754, 237691], "question_author": ""} {"qid": 1778, "query": "Does a photographer take pictures or make pictures?", "score": 32, "views": 18750, "answer_pids": [215923, 215924, 215926, 215928, 215932, 219075], "question_author": ""} {"qid": 1779, "query": "How can I create this medieval look using an entry-level camera like the Nikon D3300?", "score": 32, "views": 9163, "answer_pids": [253212, 253230, 254143, 254166], "question_author": "fbrereto"} {"qid": 1780, "query": "How do you make the most stunning sunrise and sunset photos?", "score": 32, "views": 8733, "answer_pids": [215259, 215260, 215261, 215263, 215275, 215280, 215388, 215390, 215397, 215402, 239342], "question_author": "fbrereto"} {"qid": 1781, "query": "What are the best online photo classes?", "score": 32, "views": 8251, "answer_pids": [213334, 213339, 213344, 213350, 213355, 227979, 234017, 235919], "question_author": "BoltBait"} {"qid": 1782, "query": "Is it normal business practice for a contest site to solicit submissions and then, later, ask for a fee?", "score": 32, "views": 6229, "answer_pids": [254016, 254017, 254018], "question_author": ""} {"qid": 1783, "query": "What license do I use when I dont want stock image companies charging people money for photos?", "score": 32, "views": 5654, "answer_pids": [259252, 259253, 259254, 259257, 259259, 259264, 259266], "question_author": "Canon Gangsta"} {"qid": 1784, "query": "Why cant the ISO level on most digital cameras be set below 80?", "score": 32, "views": 4992, "answer_pids": [218053, 218054, 218055], "question_author": "sbi"} {"qid": 1785, "query": "What do you learn in a BA in Photography that cant be self taught?", "score": 32, "views": 4629, "answer_pids": [216687, 216689, 216692, 216694, 216702, 216711], "question_author": "Iszi"} {"qid": 1786, "query": "How do I get over my shyness when taking photos of strangers?", "score": 32, "views": 4319, "answer_pids": [222312, 222317, 222322, 222324, 222390, 222659, 222679, 222682, 224508, 237146], "question_author": ""} {"qid": 1787, "query": "How does Image Stabilization / Vibration Reduction work?", "score": 32, "views": 3822, "answer_pids": [212518, 212527, 213116], "question_author": "TALlama"} {"qid": 1788, "query": "What is the difference between in-lens image stabilizing and sensor-based image stabilizing?", "score": 32, "views": 3813, "answer_pids": [212540, 212543, 212548, 212553, 212587, 221747, 221748, 231248], "question_author": ""} {"qid": 1789, "query": "Why does every digital camera save photos in a directory called DCIM?", "score": 32, "views": 3715, "answer_pids": [235360], "question_author": "Jeff Swensen"} {"qid": 1790, "query": "What is the best technique for long-exposure photography in good light?", "score": 32, "views": 2730, "answer_pids": [212946, 212947, 212948, 213063, 219113, 219116], "question_author": "rem"} {"qid": 1791, "query": "What is the difference between a telephoto lens and a zoom lens?", "score": 31, "views": 129112, "answer_pids": [219174, 219175, 219176, 219199, 238614], "question_author": ""} {"qid": 1792, "query": "Are airport x-ray scanners safe for DSLRs / lenses / storage media?", "score": 31, "views": 53140, "answer_pids": [216232, 216233, 216235, 216238, 224109, 226210, 227653], "question_author": ""} {"qid": 1793, "query": "What am I losing when using extension tubes instead of a macro lens?", "score": 31, "views": 52870, "answer_pids": [217987, 217988, 217990, 217991, 217995, 220628, 242552], "question_author": ""} {"qid": 1794, "query": "What does frequency mean in an image?", "score": 31, "views": 44038, "answer_pids": [232965, 248171, 258886], "question_author": ""} {"qid": 1795, "query": "How can I tell exactly what changed between two images?", "score": 31, "views": 39585, "answer_pids": [219533, 219534, 219542, 227551, 252911], "question_author": "Goran Jovic"} {"qid": 1796, "query": "Why are my photos not crisp?", "score": 31, "views": 37082, "answer_pids": [237094, 237095, 237096, 237099, 237100, 253755, 254526], "question_author": ""} {"qid": 1797, "query": "Is it better to store edited photos as PSD or TIFF files?", "score": 31, "views": 32214, "answer_pids": [220091, 220096, 233620], "question_author": ""} {"qid": 1798, "query": "How do mountains affect lighting at sunrise and sunset?", "score": 31, "views": 26242, "answer_pids": [218350, 218351, 218371, 231360, 236305, 237553], "question_author": "Grant Palin"} {"qid": 1799, "query": "What is T-number / T-stop?", "score": 31, "views": 25891, "answer_pids": [225373, 225374], "question_author": ""} {"qid": 1800, "query": "Which lenses should be included in a travel photography kit?", "score": 31, "views": 21946, "answer_pids": [219650, 219651, 219654, 219655, 219658, 219662, 219663, 219692, 219694, 219698, 223636, 237001], "question_author": "ruds"} {"qid": 1801, "query": "Can I show two sides of a card in one photo?", "score": 31, "views": 18802, "answer_pids": [249121, 249123, 249124, 249126, 249127, 249128, 249130, 249133, 249136, 249137, 249139, 249149, 249153, 249172, 249190], "question_author": ""} {"qid": 1802, "query": "What is a zone plate?", "score": 31, "views": 16252, "answer_pids": [219408, 219424], "question_author": "johnc"} {"qid": 1803, "query": "What cheap colour calibrators are available for Linux?", "score": 31, "views": 15967, "answer_pids": [215408, 215410, 215413, 225702], "question_author": ""} {"qid": 1804, "query": "If I paid for photography am I entitled to raw images?", "score": 31, "views": 14094, "answer_pids": [234951, 234952, 234959, 234966, 234969, 237621], "question_author": "user677"} {"qid": 1805, "query": "Can anyone recommend a good open-source photo management platforms for power users?", "score": 31, "views": 8807, "answer_pids": [215547, 215550, 215554, 215562, 215587, 241368], "question_author": "mmr"} {"qid": 1806, "query": "Why is the blue channel the noisiest?", "score": 31, "views": 8326, "answer_pids": [218918, 218924, 219080], "question_author": "Rodger Cooley"} {"qid": 1807, "query": "What does a hexagonal sun tell us about the camera lens/sensor?", "score": 31, "views": 8123, "answer_pids": [218245, 218248, 218250], "question_author": ""} {"qid": 1808, "query": "Is it true that the best images from all digital cameras can be obtained at ISO 200?", "score": 31, "views": 8118, "answer_pids": [251332, 251333, 251343, 251348], "question_author": ""} {"qid": 1809, "query": "When to use shutter priority instead of aperture priority?", "score": 31, "views": 7223, "answer_pids": [216069, 216070, 216071, 216077, 217555], "question_author": ""} {"qid": 1810, "query": "How do I make my photos sharp enough for print or selling online?", "score": 31, "views": 6613, "answer_pids": [244098, 244099, 244100, 244102, 244104, 244107, 254334], "question_author": "yhw42"} {"qid": 1811, "query": "What is a pancake lens?", "score": 31, "views": 6466, "answer_pids": [213552, 222453], "question_author": ""} {"qid": 1812, "query": "Do I lose anything converting to DNG?", "score": 31, "views": 5990, "answer_pids": [216094, 216097, 216098, 235082], "question_author": "Daisy Sophia Hollman"} {"qid": 1813, "query": "Is there a rule of thumb that I can use to estimate depth of field while shooting?", "score": 31, "views": 5458, "answer_pids": [216005, 216006, 216008, 216012, 243446], "question_author": "Alan"} {"qid": 1814, "query": "Why would I want to select an autofocus point?", "score": 31, "views": 5435, "answer_pids": [219779, 219780, 219781, 219782, 219783, 219784, 219790, 219792, 219801], "question_author": ""} {"qid": 1815, "query": "What options are there for good, cheap online backup of photos?", "score": 31, "views": 4610, "answer_pids": [217642, 217644, 217648, 217658, 217661, 217779, 217780, 217817, 228057, 228355, 229262, 231612, 259848], "question_author": ""} {"qid": 1816, "query": "What is noise in a digital photograph?", "score": 31, "views": 4204, "answer_pids": [219634], "question_author": ""} {"qid": 1817, "query": "Why cant I see bokeh in the viewfinder?", "score": 31, "views": 3407, "answer_pids": [224911], "question_author": "DarenW"} {"qid": 1818, "query": "What is this physical filter, shaped like a shallow pyramid?", "score": 31, "views": 3223, "answer_pids": [249224], "question_author": ""} {"qid": 1819, "query": "Tips for photographing a wedding", "score": 31, "views": 1720, "answer_pids": [215293, 215299, 215300, 215302], "question_author": ""} {"qid": 1820, "query": "Why do RAW images look worse than JPEGs in editing programs?", "score": 30, "views": 58904, "answer_pids": [233426, 233429, 233431], "question_author": "DarenW"} {"qid": 1821, "query": "Can one tell, just by looking at a picture, if its a sunset or a sunrise?", "score": 30, "views": 52292, "answer_pids": [227965, 227968], "question_author": "DarenW"} {"qid": 1822, "query": "What is the advantage of the huge number of AF points?", "score": 30, "views": 36066, "answer_pids": [225714, 225721, 225724], "question_author": "Boris Callens"} {"qid": 1823, "query": "What are the names for Magics different colour combinations?", "score": 146, "views": 1370173, "answer_pids": [94593, 94597, 109075], "question_author": ""} {"qid": 1824, "query": "What cooperative board games are there that I could safely play with my wife?", "score": 112, "views": 12696, "answer_pids": [89070, 89071, 89076, 89083, 89102, 89108, 89124, 89155, 89158, 89162, 89213, 89214, 89215, 89216, 89282, 89290, 89335, 89696, 90104, 90236, 90473, 91348], "question_author": "bryanjonker"} {"qid": 1825, "query": "How can I beat Big Money in Dominion?", "score": 109, "views": 23746, "answer_pids": [89461, 89463, 89464, 89468, 89965, 90767, 90901, 93234, 93745, 98920], "question_author": "drewm"} {"qid": 1826, "query": "Why would you want to play with a deck bigger than 60 cards in MTG?", "score": 77, "views": 98315, "answer_pids": [89168, 89169, 89192, 89273, 89278, 96170, 97878, 102308, 102531, 102536, 108592, 108600], "question_author": ""} {"qid": 1827, "query": "Is pretending to want to trade before playing a monopoly card objectionable?", "score": 74, "views": 10188, "answer_pids": [89398, 89399, 89402, 89405, 89414, 89427, 89457, 89547, 89877, 89878, 89879, 90577, 92208, 97703, 97708], "question_author": "Darius"} {"qid": 1828, "query": "Taking back turns, and forgotten privileges", "score": 74, "views": 3533, "answer_pids": [89123, 89126, 89127, 89128, 89130, 89133, 89178, 89228, 89658, 89791, 89793, 89876, 89931], "question_author": ""} {"qid": 1829, "query": "Does a player win Catan if they forget to count their points?", "score": 68, "views": 10068, "answer_pids": [90452, 90453, 90455, 90459, 90461, 90472, 90483, 90933], "question_author": "mmr"} {"qid": 1830, "query": "Boardgaming Online", "score": 67, "views": 10262, "answer_pids": [89950, 89953, 89955, 89956, 89958, 89959, 89960, 89961, 89973, 90820, 90829, 90917, 91811, 93731], "question_author": "keithjgrant"} {"qid": 1831, "query": "Countermeasures to analysis paralysis?", "score": 66, "views": 4950, "answer_pids": [89132, 89134, 89135, 89139, 89181, 89288, 89299, 89408, 89561, 90102, 93310, 93733, 94795, 99468], "question_author": ""} {"qid": 1832, "query": "Is kingmaking in multiplayer games a problem that can be fixed?", "score": 65, "views": 8798, "answer_pids": [93055, 93056, 93059, 93061, 94872], "question_author": ""} {"qid": 1833, "query": "Are Bishops more valuable than Knights?", "score": 64, "views": 29433, "answer_pids": [90853, 90858, 90994, 91850, 91854, 91856, 91858, 97172, 97537, 99747], "question_author": "Kristo"} {"qid": 1834, "query": "Can every game of Klondike-Solitaire be solved?", "score": 61, "views": 209213, "answer_pids": [91916, 91917, 91918, 93032, 93033, 93627], "question_author": ""} {"qid": 1835, "query": "Should I trash Copper or Silver with Mine?", "score": 61, "views": 5170, "answer_pids": [90876, 90877, 90878, 90881, 90893, 90910, 91525, 92723, 93998], "question_author": ""} {"qid": 1836, "query": "Do my own villages/cities interrupt my longest road in Catan?", "score": 59, "views": 75173, "answer_pids": [89080, 89084], "question_author": ""} {"qid": 1837, "query": "Is it possible to design a two player game of skill with absolutely no luck?", "score": 58, "views": 22512, "answer_pids": [102874, 102875, 102877, 102879, 102880, 102881, 102882, 102883, 102886, 102887, 102890, 102891, 103165], "question_author": ""} {"qid": 1838, "query": "In MTG, what happens when a player loses in multiplayer?", "score": 56, "views": 68752, "answer_pids": [91542, 103328], "question_author": "Rabbi David"} {"qid": 1839, "query": "How do you make Settlers of Catan work well for 2 players? (Problems and play-tested solution described, alternatives requested)", "score": 55, "views": 225937, "answer_pids": [92289, 92304, 92320, 92330, 93031, 95172, 95195, 96188, 98962, 98964, 102818, 108151, 108825], "question_author": "Borror0"} {"qid": 1840, "query": "Which property group colour gives the best ROI , undeveloped or fully developed, in Monopoly?", "score": 55, "views": 34654, "answer_pids": [89106, 91115], "question_author": "DampeS8N"} {"qid": 1841, "query": "What are some good games or techniques for helping kids learn to lose gracefully?", "score": 55, "views": 5642, "answer_pids": [89534, 89537, 89551, 89553, 89558, 89562, 89635, 102565], "question_author": "Mnementh"} {"qid": 1842, "query": "What happens if both players have the ability to pump their creature an unlimited number of times?", "score": 55, "views": 3871, "answer_pids": [96083, 96086, 100005, 101705, 101717, 102216], "question_author": "heidiphoto"} {"qid": 1843, "query": "Monopoly: Do I Collect Rent If I Am In Jail?", "score": 53, "views": 105495, "answer_pids": [103436, 103442, 103475], "question_author": "Sebastian Hoitz"} {"qid": 1844, "query": "There seem to be several different scoring systems for fields/cities in Carcassonne -- which is correct?", "score": 53, "views": 15965, "answer_pids": [89072, 89332], "question_author": "the_e"} {"qid": 1845, "query": "How can I tell the difference between early Magic: The Gathering card editions?", "score": 52, "views": 218187, "answer_pids": [91745], "question_author": "Jorge Flor\u00eancio"} {"qid": 1846, "query": "How do you calculate the likelihood of drawing certain cards in your opening hand?", "score": 52, "views": 122174, "answer_pids": [93256, 93258, 93259], "question_author": "heidiphoto"} {"qid": 1847, "query": "What exactly was so broken about Urzas Block?", "score": 52, "views": 37606, "answer_pids": [94055, 94066, 96228, 98717, 102730], "question_author": ""} {"qid": 1848, "query": "In Magic, when can I tap my opponents creatures to prevent them from attacking?", "score": 52, "views": 35860, "answer_pids": [89365, 89369, 89867], "question_author": ""} {"qid": 1849, "query": "What are mistakes beginners should avoid in Dominion?", "score": 50, "views": 25458, "answer_pids": [90883, 90884, 90885, 90887, 90891, 96231, 101422, 102906], "question_author": "thesunneversets"} {"qid": 1850, "query": "How can I improve my deck building skills?", "score": 50, "views": 5760, "answer_pids": [91402, 91404, 91407, 91408, 103481], "question_author": "jfklein13"} {"qid": 1851, "query": "In Settlers of Catan, how do you overcome bad initial settlement placement?", "score": 50, "views": 5697, "answer_pids": [89099, 89105, 89176, 90579, 91873, 95196], "question_author": "Billy Jo"} {"qid": 1852, "query": "Must a player take a jump in checkers?", "score": 49, "views": 246648, "answer_pids": [90252, 90253, 90323, 96079], "question_author": ""} {"qid": 1853, "query": "Munchkin as a two player game", "score": 48, "views": 100929, "answer_pids": [92128, 92149, 92155, 92535, 92720, 94430, 94433, 95996, 96954, 97906, 105137], "question_author": "Darius"} {"qid": 1854, "query": "Games that have no element of luck.", "score": 48, "views": 40987, "answer_pids": [89662, 89665, 89666, 89667, 89668, 89669, 89670, 89671, 89673, 89675, 89676, 89678, 89679, 89682, 89684, 89685, 89687, 89688, 89689, 89694, 89723, 89734, 89772, 89775, 89776, 89777, 89779, 89898, 90191, 90197, 90219, 90220, 90222, 90223, 90246, 90823, 90825, 90828, 90831, 90851, 91590], "question_author": ""} {"qid": 1855, "query": "Is there much strategy and planning in 7-Wonders or is mostly luck?", "score": 47, "views": 59338, "answer_pids": [90150, 90163, 90954, 90970, 92663, 93456, 93474, 93754], "question_author": ""} {"qid": 1856, "query": "Is giving resources away for free allowed?", "score": 46, "views": 18257, "answer_pids": [92234, 92235, 92236, 105369], "question_author": ""} {"qid": 1857, "query": "Is pawn promotion to rook or bishop something that is seen in play?", "score": 45, "views": 11165, "answer_pids": [92635, 92636, 108108, 108191], "question_author": "Linnaeus"} {"qid": 1858, "query": "How to mitigate bandwagon attacking from players?", "score": 45, "views": 8870, "answer_pids": [105900, 105906, 105907, 105909, 105913, 105915, 105916], "question_author": ""} {"qid": 1859, "query": "How can I play games with closed information with players who cheat?", "score": 45, "views": 5118, "answer_pids": [92884, 92885, 92887, 92889, 92890, 92903], "question_author": ""} {"qid": 1860, "query": "Land ratio vs mana curve", "score": 44, "views": 173226, "answer_pids": [93113, 93114, 105667], "question_author": ""} {"qid": 1861, "query": "How much would it cost to get started with Magic: The Gathering?", "score": 44, "views": 119979, "answer_pids": [91167, 91168, 91171, 91174, 91177, 91178, 94799, 94803, 96741, 98649], "question_author": "Nick DeVore"} {"qid": 1862, "query": "What are good standard strategies for Carcassonne ?", "score": 44, "views": 45346, "answer_pids": [89507, 89511, 89512, 89515, 89517, 89649, 89702, 89703, 89715, 90417, 90418, 93757, 97706, 100587], "question_author": "Alan"} {"qid": 1863, "query": "What are good general strategies for Pandemic?", "score": 43, "views": 186034, "answer_pids": [90408, 90412, 96631, 96641, 97913, 101265, 101268], "question_author": ""} {"qid": 1864, "query": "What techniques should I use to teach my 5-year old to play chess?", "score": 43, "views": 13700, "answer_pids": [91615, 91619, 91624, 91625, 91680, 92108, 92744, 94719, 96249, 101254], "question_author": "Andrew Vandever"} {"qid": 1865, "query": "In Magic: The Gathering, can I include a card in my deck that I cant play based on my land setup?", "score": 43, "views": 9930, "answer_pids": [104743, 104744, 105861], "question_author": "heidiphoto"} {"qid": 1866, "query": "Am I technically allowed to look up cards during a Magic tournament match?", "score": 43, "views": 8634, "answer_pids": [104503, 104504, 109334], "question_author": ""} {"qid": 1867, "query": "Why do some people use fetchlands in mono-colored decks?", "score": 42, "views": 10399, "answer_pids": [104458, 104459, 104461, 104478, 104480], "question_author": ""} {"qid": 1868, "query": "Does newer card text on a Magic: The Gathering card override the old wording?", "score": 42, "views": 4713, "answer_pids": [92426, 92435, 93664], "question_author": ""} {"qid": 1869, "query": "Can I cast a spell after a certain part of the stack is resolved in MTG?", "score": 41, "views": 37622, "answer_pids": [92238, 92239, 92240, 98057], "question_author": ""} {"qid": 1870, "query": "Which resources are better at the start of a Catan game?", "score": 41, "views": 5654, "answer_pids": [89651, 89653, 89655, 89757, 89758, 90116, 90128, 90578, 91702, 108851], "question_author": ""} {"qid": 1871, "query": "Can a settlement disrupt roads in Settlers of Catan", "score": 40, "views": 131776, "answer_pids": [89605, 100083], "question_author": ""} {"qid": 1872, "query": "Whats the deal with the new Wastes land and the diamond symbol on it?", "score": 40, "views": 79049, "answer_pids": [100022], "question_author": "readdear"} {"qid": 1873, "query": "What are good placement strategies for Stratego?", "score": 40, "views": 55817, "answer_pids": [89189, 89249, 89250, 92242, 92296, 94091, 94786, 95452, 96146, 96671], "question_author": ""} {"qid": 1874, "query": "Do you collect $200 in Monopoly if you dont ask for it?", "score": 40, "views": 27821, "answer_pids": [103227, 103256], "question_author": ""} {"qid": 1875, "query": "Beginner Power Grid strategy", "score": 40, "views": 27445, "answer_pids": [89411, 89413, 89417, 89528, 95858, 95932, 106171], "question_author": ""} {"qid": 1876, "query": "Board games for two players", "score": 40, "views": 21268, "answer_pids": [89183, 89185, 89188, 89191, 89194, 89197, 89200, 89203, 89211, 89221, 89223, 89232, 89240, 89241, 89254, 89354, 89356, 89357, 89455, 89489, 89847, 90429, 90436, 91113], "question_author": ""} {"qid": 1877, "query": "Is prompting players with alternate moves poor etiquette?", "score": 40, "views": 8659, "answer_pids": [101379, 101380, 101382, 101383, 101385, 101386, 101388, 101390, 101394, 101399, 101402], "question_author": ""} {"qid": 1878, "query": "Can infinite scry 1 be shortcut by searching through your deck?", "score": 39, "views": 5350, "answer_pids": [101000], "question_author": "Apreche"} {"qid": 1879, "query": "Why is Pot of Greed banned?", "score": 38, "views": 136034, "answer_pids": [101594, 101595, 101597, 101612, 104251, 105018], "question_author": ""} {"qid": 1880, "query": "What is summoning sickness and how does it affect my creatures?", "score": 38, "views": 49451, "answer_pids": [99235], "question_author": "Igor Zinov'yev"} {"qid": 1881, "query": "In Catan, can I ask how many cards are in a players hand?", "score": 38, "views": 20390, "answer_pids": [91829, 91847], "question_author": ""} {"qid": 1882, "query": "How can I tell when to take a mulligan in Magic?", "score": 38, "views": 12655, "answer_pids": [91413, 91422, 98143], "question_author": ""} {"qid": 1883, "query": "Games that are quick to teach to new people, yet still fun and strategical", "score": 38, "views": 6829, "answer_pids": [89832, 89833, 89834, 89835, 89836, 89837, 89838, 89840, 89841, 89842, 89862, 89868, 89869, 89974, 89976, 90103, 90313, 90324, 90531, 90582, 92048], "question_author": ""} {"qid": 1884, "query": "Does my opponent need to prove his creature has morph?", "score": 38, "views": 5499, "answer_pids": [106927, 106928], "question_author": ""} {"qid": 1885, "query": "Does an ability resolve if the source of the ability leaves the battlefield?", "score": 37, "views": 87593, "answer_pids": [93287, 93288], "question_author": ""} {"qid": 1886, "query": "Good board games for single player", "score": 37, "views": 34720, "answer_pids": [89066, 89067, 89090, 89097, 89165, 89170, 89174, 89304, 89324, 89458, 89508, 89510, 89612, 89615, 89980, 89989, 90008, 90012, 90440, 91149, 91892], "question_author": "Daniel Rodriguez"} {"qid": 1887, "query": "How do you place your initial settlements in Settlers of Catan?", "score": 37, "views": 25578, "answer_pids": [89172, 89182, 91377, 95450, 98792, 98796, 98976], "question_author": ""} {"qid": 1888, "query": "What is the optimal strategy in Quantum Tic Tac Toe?", "score": 37, "views": 6391, "answer_pids": [93789], "question_author": "Shabbyrobe"} {"qid": 1889, "query": "Setup for more than 4 players in Dominion?", "score": 36, "views": 69211, "answer_pids": [92953, 92967], "question_author": ""} {"qid": 1890, "query": "Which of the 5 resources in Settlers of Catan (base version) is most important? Why?", "score": 36, "views": 44003, "answer_pids": [92423, 92424, 93086, 109218], "question_author": ""} {"qid": 1891, "query": "How can I estimate my chances to win a Risk battle?", "score": 36, "views": 23454, "answer_pids": [90940, 90943, 90944, 90955, 97556, 98704], "question_author": ""} {"qid": 1892, "query": "What happens if the legend rule doesnt apply to only one of a pair of legendary creatures with the same name?", "score": 36, "views": 22880, "answer_pids": [102188, 102189, 102190, 102191, 109151], "question_author": "chris"} {"qid": 1893, "query": "Which program to chose when designing my own cards?", "score": 36, "views": 16521, "answer_pids": [94932, 94942, 94943, 94963, 95648, 95650, 97064, 97669, 97672, 97687, 97704], "question_author": ""} {"qid": 1894, "query": "What are some alternatives to using dice in Settlers of Catan?", "score": 36, "views": 13035, "answer_pids": [89377, 89379, 89543, 89660, 89721, 91004, 92077, 97561, 98444, 101544, 106075], "question_author": ""} {"qid": 1895, "query": "Whats the difference between a tactic and a strategy?", "score": 36, "views": 6293, "answer_pids": [89509, 89513, 89518, 89520, 89630, 91394, 106502], "question_author": ""} {"qid": 1896, "query": "How is Test of Faith resolved?", "score": 36, "views": 3191, "answer_pids": [91388, 91390], "question_author": ""} {"qid": 1897, "query": "Does a creature with Lifelink provide lifegain equal to its power, or the defenders toughness?", "score": 35, "views": 83720, "answer_pids": [92206, 92207], "question_author": ""} {"qid": 1898, "query": "When should I buy copper in Dominion?", "score": 35, "views": 9608, "answer_pids": [91737, 91739, 91740, 91766, 91791, 91903, 96232], "question_author": "Borror0"} {"qid": 1899, "query": "Why is the strong 1NT so prevalent in Bridge?", "score": 35, "views": 8756, "answer_pids": [90037, 90062, 90077, 90121, 90591, 90722, 90992, 91755, 96288, 98254, 99542, 101663], "question_author": ""} {"qid": 1900, "query": "Which alternative methods of choosing a starting player actually WORK?", "score": 35, "views": 7429, "answer_pids": [90202, 90205, 90207, 90210, 90212, 90213, 90214, 90216, 90218, 90407, 90410, 90413, 90420, 90424, 90431, 90432, 91750, 93822, 94376, 99062, 105544], "question_author": "Rodger Cooley"} {"qid": 1901, "query": "Are the A-sides and B-sides of the wonder boards really balanced?", "score": 35, "views": 3446, "answer_pids": [92071, 92673, 99844, 103951], "question_author": "mmr"} {"qid": 1902, "query": "What do you recommend to learn Go?", "score": 35, "views": 2536, "answer_pids": [89098, 89167, 90400, 90734, 90738, 91016], "question_author": "Jin"} {"qid": 1903, "query": "Can instants be played after I decide not to block?", "score": 34, "views": 104107, "answer_pids": [94101, 94103, 94115, 94960, 97630], "question_author": "joshdick"} {"qid": 1904, "query": "If you pass GO, do you collect 200 at the end of the turn or as you pass GO?", "score": 34, "views": 40344, "answer_pids": [93808, 93825, 103228, 107430], "question_author": ""} {"qid": 1905, "query": "What order do triggered abilities happen in if multiple things trigger at the same time?", "score": 34, "views": 39860, "answer_pids": [92977], "question_author": ""} {"qid": 1906, "query": "Why are there fewer board games with a triangular grid?", "score": 34, "views": 8485, "answer_pids": [89432, 89439, 89442, 89466, 89467, 89498], "question_author": ""} {"qid": 1907, "query": "In Monopoly, Is it OK for a third party to make a trade with a player who is about to lose?", "score": 33, "views": 51103, "answer_pids": [92492, 92493, 92495, 92520, 92586, 95302, 95658, 97531, 100092], "question_author": ""} {"qid": 1908, "query": "What are the most busted cards in Agricola?", "score": 33, "views": 21901, "answer_pids": [89870, 92446, 93156, 98308], "question_author": ""} {"qid": 1909, "query": "How do you play the Master Spy in Guillotine?", "score": 33, "views": 16048, "answer_pids": [90362, 90363, 91418, 92069, 92676], "question_author": ""} {"qid": 1910, "query": "What is a safe and accurate technique for shuffling cards?", "score": 33, "views": 8837, "answer_pids": [90507, 90508, 90658], "question_author": "Parasaran"} {"qid": 1911, "query": "Pandemic and Forbidden Island - How similar are they?", "score": 33, "views": 7718, "answer_pids": [89941, 89942], "question_author": "PiHalbe"} {"qid": 1912, "query": "Why do mana abilities not go on the stack?", "score": 33, "views": 5920, "answer_pids": [95405, 95406, 95412], "question_author": ""} {"qid": 1913, "query": "How do game designers come up with suggested (or minimum) age requirements for games?", "score": 33, "views": 5125, "answer_pids": [92768, 92769], "question_author": ""} {"qid": 1914, "query": "How should I approach teaching Magic:The Gathering to a new player?", "score": 33, "views": 4981, "answer_pids": [90398, 90545, 91148, 91552, 91554], "question_author": "Johannes Setiabudi"} {"qid": 1915, "query": "How do you determine the winner of a hand?", "score": 32, "views": 193108, "answer_pids": [94159, 94160, 108613], "question_author": ""} {"qid": 1916, "query": "Good games for playing in a bar", "score": 32, "views": 26941, "answer_pids": [89363, 89364, 89366, 89367, 89368, 89371, 89374, 89380, 89383, 89386, 89485, 89486, 89495, 89583, 89588, 89595, 89617, 89839, 89889, 90395, 90806, 90813, 91832], "question_author": ""} {"qid": 1917, "query": "Should I get $200 for passing Go if I then get a card that says dont collect $200?", "score": 32, "views": 23056, "answer_pids": [101580, 101583, 101587], "question_author": ""} {"qid": 1918, "query": "In Ticket to Ride, how do you overcome bad initial tickets?", "score": 32, "views": 7689, "answer_pids": [92237, 92241, 92243, 93635, 93810, 99262, 103825], "question_author": ""} {"qid": 1919, "query": "Can you throw animals into the stewpot if you dont have enough pasture room for any of them?", "score": 32, "views": 7543, "answer_pids": [93869], "question_author": ""} {"qid": 1920, "query": "Can a player choose to not pay {0}?", "score": 32, "views": 5770, "answer_pids": [107592], "question_author": "RunnerRick"} {"qid": 1921, "query": "Can there be two winners at the same time in Catan?", "score": 32, "views": 3576, "answer_pids": [99492, 99779], "question_author": ""} {"qid": 1922, "query": "How many lands do I need for a Commander deck?", "score": 31, "views": 201314, "answer_pids": [92092, 103800, 107697], "question_author": "Mark Norgren"} {"qid": 1923, "query": "How to play a quicker game of Monopoly?", "score": 31, "views": 98753, "answer_pids": [102714, 102716, 102755], "question_author": "misuba"} {"qid": 1924, "query": "What is the maximum score in Ticket to Ride USA, and how to aim towards it?", "score": 31, "views": 37720, "answer_pids": [94622, 97205, 98456, 103838, 106044], "question_author": ""} {"qid": 1925, "query": "Does anyone have a way to modify Ticket to Ride to accommodate six people?", "score": 31, "views": 17644, "answer_pids": [90542, 90547, 92189, 97036, 99450, 100556], "question_author": ""} {"qid": 1926, "query": "How Unethical Is A Purely Blocking Move In Ticket To Ride?", "score": 31, "views": 16872, "answer_pids": [90261, 90262, 90265, 90267, 90856, 90857, 90859, 91312, 94577, 94793, 95127], "question_author": ""} {"qid": 1927, "query": "What brand of card sleeves are most reliable for Magic the Gathering?", "score": 31, "views": 12102, "answer_pids": [93185, 93186, 96847], "question_author": ""} {"qid": 1928, "query": "Is adding a parallel word that only intersects the end letter of an existing word allowed in Scrabble?", "score": 31, "views": 11291, "answer_pids": [105048], "question_author": "Sarah Haren"} {"qid": 1929, "query": "Why doesnt Undying trigger if a creature with +1/+1 counters dies from -1/-1 counters?", "score": 31, "views": 8558, "answer_pids": [92444], "question_author": ""} {"qid": 1930, "query": "Why is real-money-betting necessary for poker to be fun?", "score": 31, "views": 7632, "answer_pids": [90238, 90241, 90242, 90517, 90596, 90602, 91581, 92376, 92379], "question_author": "Gordon Gustafson"} {"qid": 1931, "query": "When I put the robber on someone in Settlers of Catan, does the resource I take HAVE to be random?", "score": 31, "views": 4289, "answer_pids": [90714, 90717, 90727, 93104], "question_author": "Shabbyrobe"} {"qid": 1932, "query": "Comparisons Betwen Deck-Building Games", "score": 31, "views": 3251, "answer_pids": [89663, 89664, 94961, 97109], "question_author": null} {"qid": 1933, "query": "What makes a cooperative game require multiple players?", "score": 31, "views": 2649, "answer_pids": [97438, 97439, 97440, 98295], "question_author": "dcharles"} {"qid": 1934, "query": "How to eliminate advantages gained from the seating position of inferior players?", "score": 31, "views": 988, "answer_pids": [89638, 89639, 89640, 89642, 89659, 89736, 89738, 91363, 98866], "question_author": ""} {"qid": 1935, "query": "What is a good opening in Blokus?", "score": 30, "views": 117301, "answer_pids": [89909, 89910, 89911, 89917, 89946, 92037, 96494], "question_author": ""} {"qid": 1936, "query": "Where do you go for the nearest railroad in Monopoly?", "score": 30, "views": 50112, "answer_pids": [102932, 102936, 107766], "question_author": "Sev"} {"qid": 1937, "query": "Why do people refer to Blue color as U in Magic the Gathering?", "score": 30, "views": 45736, "answer_pids": [94904, 94905], "question_author": ""} {"qid": 1938, "query": "What are effective strategies for passing cards in Hearts?", "score": 30, "views": 33780, "answer_pids": [90521, 90523, 90524, 90527, 90528, 93139, 94509, 101275, 104725], "question_author": "Roddy"} {"qid": 1939, "query": "Are general 3-to-1 or specific 2-to-1 ports better in Settlers of Catan?", "score": 30, "views": 13545, "answer_pids": [89195, 89201, 89296, 89542, 89919, 90106, 90367, 90948, 90956], "question_author": "Gordon Gustafson"} {"qid": 1940, "query": "Is it possible to play 2-player, 2-character Arkham Horror successfully?", "score": 30, "views": 11167, "answer_pids": [89140, 90693, 96597], "question_author": "Petteri H"} {"qid": 1941, "query": "Catan: when to buy development cards?", "score": 30, "views": 10703, "answer_pids": [89316, 89317, 89318, 89355, 89372, 90111, 94826, 95296], "question_author": ""} {"qid": 1942, "query": "When you Kings Court a Kings Court, when must you pick the three target cards?", "score": 30, "views": 9645, "answer_pids": [89597, 89608, 94134], "question_author": "LeguRi"} {"qid": 1943, "query": "Does Scrabble allow playing words which contain diacritics in their original spelling?", "score": 30, "views": 7281, "answer_pids": [103588, 103589], "question_author": ""} {"qid": 1944, "query": "Why is Agricola named as such?", "score": 30, "views": 6556, "answer_pids": [105865], "question_author": "GWLlosa"} {"qid": 1945, "query": "Two players would gain control of a creature after it dies \u2014 who does?", "score": 30, "views": 4655, "answer_pids": [104396], "question_author": "AJ Finch"} {"qid": 1946, "query": "Is castling still done in the openings in modern chess?", "score": 30, "views": 2472, "answer_pids": [89069, 91494, 92169], "question_author": "Gordon Gustafson"} {"qid": 1947, "query": "Why is it of high importance to control the center of the board in Chess?", "score": 30, "views": 1853, "answer_pids": [90789, 90790, 93316], "question_author": "Gordon Gustafson"} {"qid": 1948, "query": "Seating orders effect on Puerto Rico", "score": 30, "views": 1665, "answer_pids": [89308, 95429], "question_author": "LeguRi"} {"qid": 1949, "query": "What is the converted mana cost of a spell with {X} when cast with the Miracle mechanic?", "score": 29, "views": 97746, "answer_pids": [92962, 92963, 92985], "question_author": ""} {"qid": 1950, "query": "Why does a run of 4 in Cribbage score only 4?", "score": 29, "views": 63364, "answer_pids": [90425], "question_author": ""} {"qid": 1951, "query": "In plain English, how does casting spells and using creature abilities work with the stack and priority?", "score": 29, "views": 38135, "answer_pids": [95186, 95187, 95188, 95191], "question_author": ""} {"qid": 1952, "query": "Why is Skullclamp banned?", "score": 29, "views": 32251, "answer_pids": [91293, 91294, 95944, 96051, 100468, 106607], "question_author": ""} {"qid": 1953, "query": "What are good strategies against counterspell decks?", "score": 29, "views": 30417, "answer_pids": [92078, 92081, 92082, 99804, 109005], "question_author": ""} {"qid": 1954, "query": "Can someone explain the Damage on the Stack rule change?", "score": 29, "views": 25549, "answer_pids": [90809, 90810, 90812, 92708], "question_author": "rrenaud"} {"qid": 1955, "query": "Why is Homelands considered among the worst Magic expansions?", "score": 29, "views": 22732, "answer_pids": [92403, 92404, 97862], "question_author": "AJ Finch"} {"qid": 1956, "query": "Is Pandemic harder with fewer players?", "score": 29, "views": 22300, "answer_pids": [95430, 95434, 95438], "question_author": ""} {"qid": 1957, "query": "What happens when you need to tear-down a hotel but no houses are in the bank in Monopoly?", "score": 29, "views": 14018, "answer_pids": [89591], "question_author": "ire_and_curses"} {"qid": 1958, "query": "What are the differences between Chinese and Japanese rules in Go?", "score": 29, "views": 11675, "answer_pids": [90124, 104957], "question_author": "Jadasc"} {"qid": 1959, "query": "What is offensive about the card Stone-Throwing Devils?", "score": 29, "views": 11644, "answer_pids": [109012, 109015, 109016, 109019], "question_author": "cmcculloh"} {"qid": 1960, "query": "What is the most balanced/competitive character set in Citadels?", "score": 29, "views": 11351, "answer_pids": [89915, 89921, 94641, 97322, 99400, 101183, 103846], "question_author": "Darius"} {"qid": 1961, "query": "Why Does Magic Use The Paris Mulligan Rule?", "score": 29, "views": 8436, "answer_pids": [90500, 90501, 90525, 90526, 90624, 91297, 98118, 99678, 109041], "question_author": ""} {"qid": 1962, "query": "How can we play a shorter but satisfying game of Arkham Horror?", "score": 29, "views": 6565, "answer_pids": [93746, 93747, 96595], "question_author": "Darius"} {"qid": 1963, "query": "Why does Dryad of the Ilysian Grove grant all basic land types even if it loses all abilities?", "score": 29, "views": 5483, "answer_pids": [107664], "question_author": "T.Rob"} {"qid": 1964, "query": "How important is memorizing chess openings in casual play?", "score": 29, "views": 5230, "answer_pids": [89618, 89620, 89629, 89634, 89646, 91062, 91794], "question_author": ""} {"qid": 1965, "query": "What can I do to avoid being stuck being able to do nothing in Settlers of Catan?", "score": 29, "views": 5196, "answer_pids": [93217, 93218, 93219, 93220, 93625, 93631, 93636], "question_author": ""} {"qid": 1966, "query": "Was it ever the case that Scrabble tiles had to be placed contiguously?", "score": 29, "views": 4307, "answer_pids": [108914, 108915], "question_author": "StasM"} {"qid": 1967, "query": "Interesting boardgames to play with children", "score": 29, "views": 3851, "answer_pids": [89229, 89230, 89239, 89243, 89244, 89248, 89259, 89264, 89286, 89289, 89319, 89323, 89325, 89326, 89525, 89526, 89535, 89586, 89590, 89603, 89604, 89607, 89611, 89628, 89714], "question_author": "erikric"} {"qid": 1968, "query": "What makes a cooperative board game actually a game?", "score": 29, "views": 3364, "answer_pids": [92998, 92999, 93001, 93036, 93038, 102761], "question_author": ""} {"qid": 1969, "query": "Good General Strategies for Agricola?", "score": 28, "views": 57172, "answer_pids": [89425, 89426, 89554, 89866, 96272], "question_author": ""} {"qid": 1970, "query": "I have the Dominion base set only, in what order should I buy the expansions?", "score": 28, "views": 47749, "answer_pids": [90172, 90225, 90233, 96244], "question_author": ""} {"qid": 1971, "query": "How can I play Dominion with 5+ players without expansions?", "score": 28, "views": 39911, "answer_pids": [90136, 90137, 90138, 92773], "question_author": "user771"} {"qid": 1972, "query": "Are there any ways to lose in a You cant lose the game game?", "score": 28, "views": 14768, "answer_pids": [107502, 107505, 107512], "question_author": null} {"qid": 1973, "query": "Who are Johnny, Timmy, and Spike and what do they do?", "score": 28, "views": 14729, "answer_pids": [103470, 103471, 103472, 103473, 105898], "question_author": ""} {"qid": 1974, "query": "In Pandemic, why take the extra step of eradicating a disease after youve cured it?", "score": 28, "views": 8700, "answer_pids": [106417], "question_author": "jfklein13"} {"qid": 1975, "query": "Does anybody need the 6th epidemic card in Pandemic?", "score": 28, "views": 7992, "answer_pids": [89077, 89497, 89544, 91496, 91497, 96627, 106717], "question_author": "mga911"} {"qid": 1976, "query": "Can I show my resource cards to other players in Settlers of Catan?", "score": 28, "views": 7504, "answer_pids": [91683], "question_author": "Brisbe42"} {"qid": 1977, "query": "What is the best strategy to avoid large farms in Carcassonne?", "score": 28, "views": 5138, "answer_pids": [92104, 92105, 92106, 92360], "question_author": "Lance Roberts"} {"qid": 1978, "query": "What is this weird d12 for?", "score": 28, "views": 4839, "answer_pids": [106292], "question_author": "Lee"} {"qid": 1979, "query": "How do I avoid the endless trade negotiations in Settlers of Catan?", "score": 28, "views": 3323, "answer_pids": [89392, 89393, 89394, 89397, 90107, 90113], "question_author": "Darius"} {"qid": 1980, "query": "What is a good strategy for engaging children in playing go?", "score": 28, "views": 2654, "answer_pids": [91964, 91965, 91966, 92198], "question_author": "Mateen Ulhaq"} {"qid": 1981, "query": "If I buy a Mint, do I still get to use the coins from the treasures I played that turn to buy a Grand Market?", "score": 28, "views": 895, "answer_pids": [91367], "question_author": ""} {"qid": 1982, "query": "How quickly does a player have to say Uno to avoid drawing extra cards?", "score": 27, "views": 257433, "answer_pids": [94650], "question_author": ""} {"qid": 1983, "query": "What is the best way to print cards for my game?", "score": 27, "views": 119138, "answer_pids": [91353, 91380, 91382, 91924, 91929, 92552, 95431], "question_author": ""} {"qid": 1984, "query": "Where can I find good tile layouts for Settlers of Catan?", "score": 27, "views": 84108, "answer_pids": [94096, 94097, 94098, 94099, 94100, 94109, 94247, 96700, 98684, 105510], "question_author": "Gordon Gustafson"} {"qid": 1985, "query": "How does the double strike mechanic interact with other mechanics like lifelink or trample?", "score": 27, "views": 53860, "answer_pids": [92205], "question_author": "aslum"} {"qid": 1986, "query": "Differences between Dixit versions?", "score": 27, "views": 51458, "answer_pids": [95128, 95129, 98644, 98931], "question_author": "Chris"} {"qid": 1987, "query": "What Are The Official Rules for Carcassonne: The River?", "score": 27, "views": 48198, "answer_pids": [90051], "question_author": ""} {"qid": 1988, "query": "What are the differences between the four editions of The Settlers of Catan base game?", "score": 27, "views": 38099, "answer_pids": [89981, 89983, 89984, 89985, 94621], "question_author": "spong"} {"qid": 1989, "query": "How does Intrigue change your general Dominion strategy?", "score": 27, "views": 21617, "answer_pids": [89986, 89988, 89994, 90041, 93313], "question_author": "Casebash"} {"qid": 1990, "query": "What happens if you roll doubles 3 times then land on Go to jail?", "score": 27, "views": 20493, "answer_pids": [106040, 106041], "question_author": ""} {"qid": 1991, "query": "Recommendations for 6-player board game", "score": 27, "views": 19701, "answer_pids": [89224, 89226, 89231, 89233, 89234, 89236, 89237, 89251, 89252, 89265, 89268, 89269, 89293, 89506, 89519, 89922, 89937, 90105, 91272], "question_author": ""} {"qid": 1992, "query": "What are good standard tactics for Dominion?", "score": 27, "views": 18607, "answer_pids": [89305, 89306, 89307, 89310, 89344, 89540, 89582, 89754, 89800, 89802, 89823, 90773, 90780], "question_author": null} {"qid": 1993, "query": "Help! Im A Cylon!", "score": 27, "views": 13927, "answer_pids": [92814, 92818, 92819, 92827, 92830], "question_author": ""} {"qid": 1994, "query": "Can non-creature permanents have +1/+1 counters?", "score": 27, "views": 12547, "answer_pids": [96687, 96706], "question_author": ""} {"qid": 1995, "query": "What are good strategies for booster drafts?", "score": 27, "views": 11413, "answer_pids": [91631, 91645, 91833, 106419], "question_author": ""} {"qid": 1996, "query": "How should I ship cards?", "score": 27, "views": 9531, "answer_pids": [105803, 105808, 105813, 105814, 105826], "question_author": "Gordon Gustafson"} {"qid": 1997, "query": "If I cant win the game in multiplayer, do my opponents still lose?", "score": 27, "views": 7143, "answer_pids": [102613, 102615, 102616, 102628, 102630], "question_author": "jfklein13"} {"qid": 1998, "query": "Why is Pandemics Bio-terrorist Challenge not recommended with five players?", "score": 27, "views": 5542, "answer_pids": [90002], "question_author": "Badmike"} {"qid": 1999, "query": "Is untapping my permanents during the untap step optional?", "score": 27, "views": 5439, "answer_pids": [92800, 92801], "question_author": ""} {"qid": 2000, "query": "In Magic: the Gathering, what is being gathered?", "score": 27, "views": 5371, "answer_pids": [95884], "question_author": "PearsonArtPhoto"} {"qid": 2001, "query": "Does priority matter in Magic?", "score": 27, "views": 4912, "answer_pids": [97748, 101857, 107742], "question_author": ""}